FP 513 practice questions

अब Quizwiz के साथ अपने होमवर्क और परीक्षाओं को एस करें!

Identify which of these is NOT a characteristic of a normal yield curve.

As the maturity date of bonds lengthens, the corresponding bond yield decreases.

If an American investor holds foreign securities and is concerned about exchange rate risk, that investor may hedge by

B) buying a futures contract for delivery of dollars. The answer is buying a futures contract for delivery of dollars. If an investor holds securities denominated in euros, for example, he is concerned about the dollar appreciating versus the euro when he converts his holdings back to dollars. He can hedge against the dollar appreciating (which would be negative when holding foreign securities) by buying a futures contract for delivery of dollars.

Jana believes that the allocation to emerging market equities in her portfolio has become overvalued, so she trims the holdings in that asset class and reinvests the proceeds in other asset classes that she believes are undervalued. Jana's approach to asset allocation can best be described as

B) tactical asset allocation. Tactical asset allocation continuously adjusts the asset allocation and class mix in an attempt to take advantage of changing market conditions and overall investor sentiment.

Which of the following methods can be used in determining the basis in a mutual fund when the shares were acquired at different times? Specific identification First in, first out (FIFO) Average cost method

D) I, II, and III

Which of the following statements regarding Eurodollar CDs is CORRECT? Eurodollar CDs are obligations of non-U.S. banks. Eurodollar CDs are more liquid than domestic CDs. Eurodollar CDs offer a slightly higher yield than domestic CDs. Eurodollar CDs are only used to settle transactions in the U.S.

Eurodollar CDs are obligations of non-U.S. banks. Eurodollar CDs offer a slightly higher yield than domestic CDs.

In general, rising interest rates result in which of the following combinations?

Falling stock and bond prices

Which of the following is not a type of money market mutual fund?

Federally insured money market mutual fund

Your clients have invested in a variety of mutual funds, including foreign country funds. You expect the U.S. dollar to strengthen against the Japanese yen over the next year. Which of these actions would be most appropriate?

Hedge by taking a short position in yen futures.

Which of the following characteristics are associated with the market anomaly known as the neglected-firm effect? Low price-to-earnings (P/E) ratio stocks outperform high P/E stocks. Stocks of foreign companies outperform known domestic stocks. Neglected-firm stocks underperform large capitalization stocks. Stocks not frequently followed by analysts outperform widely followed stocks.

IV Neglected firms are those that are neglected (not followed) by many financial analysts. When such companies can be found, the market for those companies may not be efficient, and investors who can take the time to analyze these companies may be able to take advantage of undervalued situations. LO 8.3.1

Which of the following correctly describes the efficient frontier in portfolio theory?

It indicates the highest returns for given levels of risk. At any point along the efficient frontier the highest return for a given level of risk is present. The optimal portfolio is the point on the efficient frontier that intersects with the investor's indifference curve.

Janice, who is in the 35% marginal income tax bracket, would like to purchase a bond for her investment portfolio. Assuming all of the bonds are of similar investment quality, which would produce the highest after-tax yield?

Janice should purchase the municipal bond based on the following after-tax yield calculations: U.S. Treasury bond [2.75% × (1 − 0.35)] 1.79% Corporate bond [5.25% × (1 − 0.35)] 3.41% Municipal bond (tax-free) 3.55% U.S Treasury note [2.25% × (1 − 0.35)] 1.46%

Which of the following statements describes a limitation of Monte Carlo simulation?

Outcomes of a simulation can only be as accurate as the inputs to the model.

While managing his portfolio, James's investment adviser attempts to take advantage of perceived market inefficiencies. His investment adviser is not concerned with James's long-term goals; rather, the interest lies in continuously changing the investment mix to take advantage of overall investor sentiment. Based on this information, choose the type of portfolio management style that the investment adviser is using to manage James's money.

Tactical asset allocation The answer is tactical asset allocation. Tactical asset allocation continuously adjusts the asset allocation in an attempt to take advantage of changing market conditions.

On January 1, 20XX, one U.S. dollar could buy 105 Japanese yen; on December 31, 20XX, one U.S. dollar could buy 121 Japanese yen. Which one of these statements best illustrates what has happened to the U.S. dollar during year 20XX?

The U.S. dollar was revalued. The answer is the U.S. dollar was revalued. A revaluation occurs to a U.S. investor when the U.S. dollar can buy more of a foreign currency today than it could buy last year. A devaluation occurs when the U.S. dollar can buy less of a foreign currency today than it could buy last year.

Ronald owns a Hydro Industries 7% convertible bond. The bond is convertible into 30 shares of Hydro Industries, which is currently trading at $43 per share. The investment value of the bond is $980, and the current market price of the bond is $1,433. What is the conversion value of Larry's bond?

The answer is $1,290. Conversion value = conversion ratio × market price of common stock, or 30 × $43 = $1,290.

Your client, Ralph, has $15,000 of capital gains and $20,000 of capital losses in the current tax year. How much unused loss may Ralph carry forward to the following tax year?

The answer is $2,000. After netting capital gain and losses, the client has a net capital loss of $5,000. Because $3,000 of net losses can be deducted during any one tax year, the client will carry over the remaining $2,000 capital loss.

Jack is interested in purchasing LFM stock. LFM has an estimated free cash flow to equity (FCFE) for the next year of $2.75 per share, which is expected to grow at a constant rate of 3.5% per year. Jack's required rate of return is 12%. Using the FCFE valuation model, calculate the intrinsic value of LFM stock.

The answer is $32.35. Using the model V = FCFE1 ÷ (r - g), the intrinsic value of the stock is $32.35 [$2.75 ÷ (0.12 - 0.035)]. LO 4.3.1

Consider CPM stock with a current dividend of $1.05 per share and a market price of $46.65 per share. The current dividend is expected to grow for three years at a rate of 2% and then 3% thereafter. Assume the required rate of return is 6%. Using the multistage growth dividend discount model, calculate the intrinsic value of CPM stock. (Round all numbers to the nearest cent.)

The answer is $34.82. Compute the value of each future dividend until the growth rate stabilizes (years 1 through 3). D1 = $1.05 × 1.02 = $1.07 D2 = $1.07 × 1.02 = $1.09 D3 = $1.09 × 1.02 = $1.11 Use the constant growth dividend discount model to compute the remaining intrinsic value of the stock at the beginning of the year when the dividend growth rate stabilizes (year 4). D4 = $1.11 × 1.03 = $1.14 V = $1.14 ÷ (0.06 - 0.03) = $38.00 Use the uneven cash flow method to solve for the net present (intrinsic) value of the stock. CF0 = $0 CF1 = $1.07 CF2 = $1.09 CF3 = $1.11 + $38.00 = $39.11 I/YR = 6% Solve for NPV = 34.8170, or $34.82

An investor has purchased $500 of additional shares in a mutual fund each month for the past five months at the following prices: MonthPrice Per ShareInvestmentJan.$35$500Feb.$37$500Mar.$38$500Apr.$32$500May$33$500

The answer is $34.85.

An investor buys 100 shares of stock at $75 per share, with a 60% initial margin requirement and 40% maintenance margin requirement. Assuming the stock quickly falls to $40 per share, calculate the additional capital that the investor must provide to cover a margin call.

The answer is $600. The current market value of the 100 shares is $4,000. The maintenance margin requires an equity of $4,000 × 0.40, or $1,600. The investor's equity in the account ($1,000) is the market value ($4,000) minus the loan amount ($3,000). A margin call for $600 ($1,600 - $1,000) will be ordered.

Norma owns ABC Corporation bonds of AA rated quality that mature in seven years, pay semiannual interest, and have a coupon of 8%. Similar bonds (AA rated, seven years to maturity) yield 9%. The ABC Corporation bonds are convertible into common stock at $26 per share, and the current market price of ABC common stock is $23. What is the conversion value of an ABC Corporation bond?

The answer is $884.61. The conversion value = conversion ratio × market price of common stock. Therefore, the conversion value equals ($1,000 ÷ $26) × $23 = $884.61.

ABC Corporation has a P/E ratio of 5.00 and an expected growth rate in earnings for the next year of 9.5%. Assuming an investor's required rate of return is 12%, calculate the firm's PEG ratio.

The answer is 0.5263. Calculate the firm's PEG ratio as follows: 5.00 ÷ (0.095 × 100) = 0.5263. After calculating this ratio, it then would be compared to ABC Corporation's peers to determine whether a purchase is warranted.

Brenda is interested in calculating the inflation-adjusted rate of return of a recent investment. Assuming the after-tax return on her investment is 6.25% and the inflation rate is 5%, calculate the inflation-adjusted rate of return.

The answer is 1.19%. The inflation-adjusted rate of return is calculated as follows: [(1.0625 ÷ 1.05) − 1] × 100 = 1.19%

Rex, Ltd., has assets of $400 million and liabilities of $200 million. Last year, the company earned $45 million and paid out $15 million in dividends. Using the formula g = return on equity × retention rate, what is the growth rate for Rex, Ltd.?

The answer is 15.0%. $400,000,000 - $200,000,000 = $200,000,000 of equity; $45,000,000 earnings ÷ $200,000,000 equity = 0.225, or 22.50% ROE $45,000,000 earnings - $15,000,000 paid out in dividends = $30,000,000 of retained earnings; $30,000,000 ÷ $45,000,000 = 0.6667 retention rate g = ROE × RR g = 0.2250 × 0.6667 = 0.15, or 15%

VUL stock has a current market price of $25.65 and sales per share of $1.67. Calculate the price-to-sales ratio for this stock.

The answer is 15.36. The formula for the price-to-sales (P/S) ratio: P/S = market price per share ÷ sales per share P/S = $25.65 ÷ $1.67 = 15.3592, or 15.36 This ratio would then be compared to its industry peers to determine whether the stock appears to be overvalued or undervalued. LO 4.3.2

Stock TTY has a mean return of 11% and a standard deviation of 5%. What is the probability of a return of less than 6%, assuming a normal distribution of returns?

The answer is 16%. The probability of Stock TTY returning more than 11% is 50%. Additionally, the probability that the stock's return lies within one standard deviation of the mean is 68%. Because the distribution is symmetric, there is a 34% chance that returns will be between 6% and 11%. Therefore, the probability of a return less than 6% is 16% (100% − 50% − 34%). LO 6.2.2

JEM Technologies, Inc. has assets of $500 million and $50 million in liabilities. For the past year the company earned $125 million, and paid out $50 million in dividends. Calculate the company's return on equity (ROE)

The answer is 28%. $500,000,000 - $50,000,000 = $450,000,000 in equity. $125,000,000 profit ÷ $450,000,000 equity = 0.2778, or 28% ROE.

Robinson owns a municipal bond with a coupon rate of 2.75%. He is currently in the 32% federal marginal income tax bracket and resides in a state that does not impose a state income tax. Calculate his municipal bond's taxable equivalent yield (TEY).

The answer is 4.04%. The bond's TEY is calculated as follows: 2.75% ÷ (1 - 0.32). LO 7.4.1

Nancy bought $4,000 worth of common stock two years ago. She received dividends of $30 each quarter for the first year and $35 each quarter for the second year. The stock currently is worth $4,100. The graph below summarizes inflows and outflows. What is the internal rate of return that the stock has earned?

The answer is 4.45%. Keystrokes: HP 10bII+(set for 4 P/YR)4000,+/-, CFj30,CFj, 4, SHIFT, Nj35,CFj, 3, SHIFT, Nj4100,+, 35, =, CFjSHIFT, IRR/YR

Steve has an AA rated bond with an annual coupon rate of 4.35% that is currently trading for $965. Calculate the bond's current yield.

The answer is 4.51%. The bond's current yield is calculated as $43.50 ÷ $965. Annual interest payment as a percent of par equals $43.50 ($1,000 × 4.35%) divided by the current market price of $965.

XYZ Corporation issues a 20-year callable bond paying a 6% coupon (semiannual payments) selling at par ($1,000). XYZ Corporation has the option to call the bonds in five years for 105% of par value. Calculate the bond's nominal yield.

The answer is 6.00%. The nominal yield (coupon rate) is 6%. The nominal yield is stated as a percentage of the par value of the bond.

An analysis of the monthly returns for the past year of a mutual fund portfolio consisting of two funds revealed these statistics: Fund AFund BTotal return18%11%Standard deviation23%16%Percentage of portfolio35%65%Correlation coefficient (R)0.25 What is the coefficient of determination (R2) of Fund A and Fund B?

The answer is 6.25%. The coefficient of determination is the square of the correlation coefficient (0.25)2 = 0.25 × 0.25 = 0.0625, or 6.25%.

A $1,000 U.S. Treasury note maturing in eight years is selling for $938.12. The semiannual coupon payment is $35. What is the yield to maturity (YTM) for the note?

The answer is 8.06%. The note's YTM is computed as follows: END Mode, 2 P/YR PV = −938.12 PMT = 35 FV = 1,000 8, DOWNSHIFT, N = 16 Solve for I/YR = 8.06%

What is the covariance between OPC and NIR stocks with a standard deviation of 9.13% and 11%, respectively, and a correlation coefficient of 0.85?

The answer is 85.37. The covariance between the two stocks is 85.37 (9.13 × 11 × 0.85). Covariance measures the extent to which two variables move together, either positively (together) or negatively (opposite).

Identify which of these statements regarding revenue bonds is NOT correct. They are secured by a specific pledge or property. They are a type of full faith and credit bond. Their interest is tax-exempt at the federal level. They are analyzed by the project's ability to generate earnings.

The answer is I and II. Revenue bonds are not secured by property and are not a type of general obligation bond. They are only secured by user fees.

Companies A and B have exactly the same dollar amount of assets and net income. Company A has a capitalization structure of 70% equity and 30% debt; Company B has a capitalization structure of 40% equity and 60% debt. Which one of these statements is CORRECT? Company B has a higher ROE than Company A. Company B has a higher ROA than Company A. Company A has a higher debt-to-equity ratio than Company B.

The answer is I only. All else being equal, a profitable company with a higher debt level will have a higher return on equity. If income is the same for both companies, then the only difference is the percentage of equity. With a lower equity, Company B will have a higher return on equity. Company B has a higher debt-to-equity ratio.

Which of the following would be held in a money market portfolio? Treasury bill Negotiable CDs Commericial paper

The answer is I, II, and III. All of these financial instruments would be held in a money market portfolio. LO 1.3.1

Top-down analysis includes which of these? Economic and market factors Industry analysis Company analysis

The answer is I, II, and III. Top-down analysis includes all these elements. Top-down analysis allows investors to look at the big picture first and then work their way down to the details.

Connie, 45, has an extensive amount of assets in a separately managed account. The account is mostly compromised of small-cap growth companies. What is the best benchmark to analyze the performance of her account?

The answer is Russell 2000 Index. Connie should use the Russell 2000 Index to compare her investment portfolio's performance to a benchmark. This index is used to benchmark small-capitalization companies.

Brian and Kellie purchased savings bonds for their children's future college education expenses. What series of bonds did they choose if the savings bonds are inflation-indexed and may be used to pay for higher education costs on a tax-favored basis?

The answer is Series I. Series I savings bonds are an inflation-indexed debt security issued by the U.S. government. The accrued interest on Series I bonds may be completely excluded from income tax if the bond proceeds are used to pay for qualified higher education costs.

When a company issues an option to buy its stock at a specified price within a specified time period, it is known as a

The answer is a warrant. A warrant can only be created by corporations and is an option to buy its stock at a specified price within a specified time period.

The performance of two growth and income mutual funds is displayed below: ABC FundXYZ FundAverage annual rate of return8.65%6.78%Standard deviation of returns5.86%9.98%Beta0.751.00 Assuming a risk-free rate of return of 5%, which of these statements is CORRECT?

The answer is all of the statements are correct. When compared with another investment, the higher the Treynor ratio, the better the risk-adjusted performance of the asset. Therefore, ABC Fund with a Treynor ratio of 0.0487 has a better risk-adjusted performance than XYZ Fund. XYZ Fund has a beta greater than ABC Fund, indicating a higher level of systematic risk. Calculations: Treynor ratio for ABC Fund is 0.0487 [(0.0865 - 0.05) ÷ 0.75] Treynor ratio for XYZ Fund is 0.0178 [(0.0678 - 0.05) ÷ 1.00] Sharpe ratio for ABC Fund is 0.6229 [(0.0865 - 0.05) ÷ 0.0586] Sharpe ratio for XYZ Fund is 0.1784 [(0.0678 - 0.05) ÷ 0.0998] LO 7.4.1

ich of the following is a risk involved in an investment in undeveloped land?

The answer is all of these. An investment in undeveloped land may be subject to all of these risks.

Which of the following statements regarding certificates of deposit (CDs) is CORRECT? CDs are deposits made with a bank or savings and loan for a specified period, commonly one month to five years. Negotiable CDs are deposits of $100,000 or more placed with commercial banks at a specified interest rate for a term of up to one year.

The answer is both I and II. Both of these statements accurately describe CDs.

An investor fearing a bear market would hedge his or her position b

The answer is buying a put. Buying a put is bearish. All the other choices are bullish.

he duration of a bond is inversely related to its

The answer is coupon rate. Duration is directly related to term to maturity and inversely related to coupon rate and yield to maturity. LO 4.1.2

Elayne recently purchased a municipal bond through her stockbroker. The broker told her that the bond is backed by the full faith and credit of the issuer. What type of bond did Elayne buy for her portfolio?

The answer is general obligation bond (GO). GOs are municipal bonds that are issued to finance capital improvements for the benefit of the entire community. Because taxes back most GOs, municipalities may require a taxpayer vote to approve new issues.

All of the following are risks associated with hedge funds except

The answer is long selling. Short selling is a risk associated with hedge funds because losses can be incurred in unlimited amounts.

Which of the following statements regarding fundamental and technical analysis is CORRECT?

The answer is technical analysis is not considered valid under the efficient market hypothesis because this type of analysis is attempting to predict future prices based on past price movement.

All of the following statements correctly explain the core and satellite approach to investing except

The answer is the core portion of the portfolio uses an active investment philosophy to achieve above-market returns. The core portion of the portfolio uses a passive investment philosophy to achieve market-based returns, while the satellite portion of the portfolio uses active investment management in an attempt to achieve above-market level returns.

Which statement regarding portfolio theory is NOT correct?

The answer is the optimal portfolio will always lie above the efficient frontier. The optimal portfolio is found at the point of tangency of the investor's indifference curve and the efficient frontier.

Which one of these statements is CORRECT regarding exchange-traded funds (ETFs)?

The answer is they also have lower turnover of assets than mutual funds and are, as a result, more tax efficient. In-kind exchanges are done in blocks of 50,000 shares or more. An ETF that buys all the securities in an index is called a replicate index-based ETF. Futures contracts are taxed at year-end on appreciation at rates of 60% for long-term gains and 40% for short-term gains. ETFs can and do offer currency ETFs.

Jensen's alpha is an absolute measurement. What does it tell you?

The percentage a manager over- or underperformed based on the amount of risk taken The answer is the percentage a manager over- or underperformed based on the amount or risk taken. Jensen's alpha is a measure of the risk-adjusted value added by a portfolio manager. Specifically, alpha is measured as the portfolio's actual or realized return in excess of (or deficient to) the expected return calculated by the capital asset pricing model (CAPM).

Which of these best describes the concept of asset allocation?

The process of apportioning assets available for investment among various investment classes

Which of the following statements is CORRECT regarding hedge funds?

They often make extensive use of derivatives. Hedge funds may employ a variety of investment strategies in an attempt to achieve a superior return for investors and, as such, can be conservative investments or very aggressive investments.

Which statement concerning an active portfolio management strategy is CORRECT?

Which statement concerning an active portfolio management strategy is CORRECT?

The segment of the security trading marketplace that allows for institutional investors to trade with other institutional investors outside of normal trading hours is known as

the fourth market.

Debbie owns a five-year AAA rated municipal bond with a coupon rate of 3.50% (paid semiannually). If the comparable yield for this quality bond is currently 2.75%, what is the present value of her bond?

$1,034.81 The answer is $1,034.81. The present value of her bond is $1,034.81, calculated as follows: END Mode, 2 P/YR 5, DOWNSHIFT, N = 10 I/YR = 2.75 PMT = 17.50 (3.50% × 1,000 ÷ 2) FV = 1,000 Solve for PV = -1,034.81, or $1,034.81. LO 4.1.1

Which of these is NOT a characteristic of negotiable certificates of deposit (CDs)?

They are used as short-term drafts drawn to finance imports and exports.

The Sharpe ratio

does not assume the portfolio is well diversified.

The daily limit of a commodity futures contract is the maximum

price increase or decrease relative to the settlement price the previous day.

The interest rate theory that long-term rates consist of many short-term rates and that long-term rates will be the average of short-term rates is known as

unbiased expectations theory.

LAC Corporation stock is currently trading for $180 per share. If the company institutes a 3-for-2 stock split, calculate the company's stock price following the stock split.

The answer is $120. The company's new stock price will be $120, calculated as follows: $180 ÷ 3 × 2 = $120.

Element Corp had these annual returns over the past four years: +12%, +6%, -8%, and +20%. What is the standard deviation for Element Corp. over the past four years?

The answer is 11.8%. HP 10bII+ Keystrokes: 12, ∑+ 6, ∑+ 8, +/-, ∑+ 20, ∑+ SHIFT, Sx,Sy (8 key) for standard deviation LO 6.2.2

Brantley recently purchased a new video game system on his credit card. Assuming the nominal annual percentage rate (APR) is 14.95% (compounded daily), calculate the effective annual rate (EAR).

The answer is 16.12%. The effective annual rate on his credit card is 16.12%, calculated as follows: EAR = [1 + (0.1495 ÷ 365)]365 - 1 = 0.1612, or 16.12%.

Harry has an investment that has produced the following returns: Year 1: 10%, Year 2: 5%, Year 3: -7%, Year 4: -3%, Year 5: 12%. Calculate the arithmetic mean return on this investment.

The answer is 3.40%. The arithmetic mean is calculated by dividing the sum of the periodic returns by the total number of periods being evaluated. Therefore, Harry earned an average of 3.40% [(10% + 5% - 7% - 3% + 12%) ÷ 5] per year on his investment.

The Gemini Fund has a correlation coefficient of 0.80 with the S&P 500 Index. How much of the price movement of the Gemini Fund can be explained by the S&P 500 Index?

The answer is 64%. The correlation coefficient (R) has been given, so it needs to be squared (R2) in order to come up with the coefficient of determination (0.802 = 0.64).

ABC Mutual Fund has a correlation coefficient of 0.93 with the S&P 500 Index. How much of the price movement of the fund can be explained by the S&P 500 Index?

The answer is 86%. The correlation coefficient (R) has been given, so it needs to be squared (R2) in order to come up with the coefficient of determination. (0.932 = 0.8649, or 86%)

Which of these is a correct justification for use of an investment in a client's portfolio? Blue chip common stocks because they provide a hedge against inflation FNMA (Federal National Mortgage Association) securities because they are backed by the full faith and credit of the U.S. government Aggressive growth stocks because they perform better during economic contractions

The answer is I only. Stocks generally are considered an inflation hedge; in periods of hyper-inflation, this may not be true, but the question does not ask about periods of hyper-inflation. FNMA securities are not backed by the full faith of the government (the government did step in as a result of the credit crisis of 2008, but there has not been a commitment to permanently back FNMAs in the same way that GNMAs (Government National Mortgage Association) have historically been backed). LO 3.1.1

Exchange-traded funds (ETFs) generally offer which of these? Tax efficiency Low expense ratios Active professional management Marketability

The answer is I, II, and IV. Tax efficiency, low expenses, and marketability are all characteristics of ETFs. Professional management is incorrect because virtually all ETFs are, in effect, index funds, which are passively managed.

Identify which of the following statements regarding money market deposit accounts (MMDAs) are NOT correct. They are FDIC insured. They offer unlimited check writing privileges. They are primarily offered by open-end investment companies. They require a minimum balance.

The answer is II and III. MMDAs provide limited check writing privileges and are offered by banks and savings and loans. MMDAs require a minimum balance. Unlike money market mutual funds, MMDAs are FDIC insured.

Which of the following statements regarding the efficient market hypothesis (EMH) is CORRECT? The EMH suggests that active management should be used. Under the EMH, all publicly known information is incorporated into security

The answer is II only. The EMH suggests that passive, not active, management should be used.

identify which of these methods may be used to trade exchange-traded funds (ETFs). Investors can buy or redeem shares from the fund family in lots of 1,000. Investors can trade ETFs in the secondary market by using a broker. ETFs can be purchased on margin. ETFs may be sold short.

The answer is II, III, and IV. Some ETFs may be redeemed through the fund family but usually in lots of 50,000 shares or more.

Which statement regarding the concepts of modern portfolio theory (MPT) is NOT correct?

The answer is Markowitz used risk (as measured by beta) and expected return as the basis for determining appropriate assets or portfolios. Harry Markowitz's theory uses standard deviation as a measure of portfolio risk.

Jerry Warner would like to increase his portfolio of common stock. He requires a 13% rate of return on common stock investments. He is considering purchasing one of these stocks: Stock 1:Dividends are currently $3.00 annually and are expected to increase 7% annually; market price = $45Stock 2:Dividends are currently $2.25 annually and are expected to increase 8% annually; market price = $50 Which stock is most appropriate to purchase in this situation, and why?

The answer is Stock 1, because the return on investment is greater than Jerry's required rate of return. The intrinsic value of Stock 1 = $53.50 [($3 × 1.07) ÷ (0.13 - 0.07)]. Because $53.50 is more than $45, the stock is undervalued and would return more than his required return. The intrinsic value of Stock 2 = $33.50 [($2.25 × 1.08) ÷ (0.13 - 0.08)]. Because $48.60 is less than $50, the stock is overvalued and would return less than his required return.

The top 10 shareholders of publicly held Emax, Inc., are selling 10 million shares they personally own through a public offering. The investment banker will offer these shares as part of which of the following offerings?

The answer is a secondary offering. If a company has already issued shares but wants to raise additional capital through the sale of more stock, it does so by what is called a secondary or seasoned offering.

Which of the following financial intermediaries would a business look to for startup capital?

The answer is investment bankers. Startup capital is typically available at commercial banks and from investment bankers.

IN a positively skewed distribution, what is the order (from lowest value to highest) for the distribution's mode, mean, and median values?

The answer is mode, median, mean. In a positively skewed distribution, the mode is less than the median, which is less than the mean.

Jim, 32, and Caren, 30, have been married for seven years. Both Jim and Caren work and have no children, so they have a large amount of disposable income. They live in the suburbs and are planning to purchase a condominium downtown as a weekend getaway. They would like to invest their money in a safe place for the down payment during the six months they spend searching for the perfect location and amenities. Which of the following investments would you recommend to Jim and Caren to accomplish this goal?

The answer is money market fund. Jim and Caren are preparing to make a significant purchase within a relatively short period of time. They will require a highly liquid investment to keep their money safe. Therefore, the money market fund best matches their investment objective.

Which of the following statements regarding the arbitrage pricing theory (APT) is CORRECT?

The answer is multiple factors affect the return of a security. The APT determines returns based on multiple factors. These factors might include inflation, growth in GDP, major political upheavals, or changes in interest rates.

As a result of a market correction, your client's portfolio mix has changed substantially from the desired allocation. What portfolio management technique could be used to bring the allocation back to the desired mix?

The answer is portfolio rebalancing. Rebalancing refers to adjusting the weights of securities in a portfolio to their target weights after price changes have affected the weights. LO 8.6.1

FER stock has a current dividend of $0.75 per share that has been growing at a rate of 1.25% per year. If an investor's required rate of return is 15% and the stock is currently selling for $6.34 per share, determine whether the investor should purchase the stock.

) No, the stock is overvalued based on the constant growth dividend discount model. The answer is no, the stock is overvalued based on the constant growth dividend discount model. Based on the constant growth dividend discount model, the intrinsic value of the stock is $5.52, calculated as follows: [0.75 × (1 + 0.0125)] ÷ (0.15 - 0.0125) = 0.7594 ÷ 0.1375 = 5.5227, or $5.52. Because FER is currently trading at a price of $6.34 per share, it is overvalued, and the investor should not buy the stock.

Which of the following statements regarding investment theory is NOT correct?

) The beta coefficient may be used to help select a portfolio that is consistent with an investor's willingness to assume unsystematic risk. Beta is a measure of systematic risk, not unsystematic risk. The beta coefficient may be used to help select a portfolio that is consistent with an investor's willingness to assume systematic risk.

Beverly owns two stocks with a correlation coefficient of zero. Which of these is CORRECT?

) These stocks will move independently of each other.\ The answer is these stocks will move independently of each other. A correlation coefficient of zero means that the two stocks will move independently. Because most stocks are positively correlated, a correlation coefficient of zero should provide more diversification benefits than most pairs of stocks.

All of the following statements regarding modern portfolio theory are correct except

) the optimal portfolio will always lie above the efficient frontier. The answer is the optimal portfolio will always lie above the efficient frontier. The optimal portfolio will always lie on the investor's efficient frontier. This portfolio is found at the point of tangency of the investor's indifference curve and the efficient frontier.

Henry is constructing an asset allocation portfolio for his financial planning client, Gretchen, based on information obtained from completing a risk-profile questionnaire. Which of the following are considerations for Henry to keep in mind when developing portfolio alternatives for Gretchen?

All of these are considerations.

Select the CORRECT statement regarding foreign investments.

American depositary receipts (ADRs) pay dividends in U.S. currency.

Which of the following statements regarding the capital market line (CML) is CORRECT?

B) Provides a direct relationship between the risk and return for a well-diversified portfolio The answer is provides a direct relationship between the risk and return for a well-diversified portfolio. The capital market line (CML) graphically depicts the relationship of risk and return for efficient well-diversified portfolios. The CML uses standard deviation as a risk measure.

Which of the following best illustrates characteristics of mortgage and equity real estate investment trusts (REITs)? Mortgage REITs Equity REITs A Generates capital gains Generates interest income B Generates rental income Generates capital gains C Similar to a bond fund Similar to a stock fund D Also known as a FREIT Also known as a hybrid REIT

Equity REITs own property and receive all income from the property rentals, making them more like a stock fund; they also are more volatile than mortgage REITs, and they provide more capital gain opportunity then do mortgage REITs. Mortgage REITs own the mortgages used to finance real estate properties, making them more like a bond fund. LO 5.1.1

Andy owns a yen-denominated bond that matures in 15 years. Andy's bond is subject to which one of these combinations of systematic risk?

Exchange rate risk and reinvestment rate risk

Which of the following risks is specific to international investing?

Exchange rate risk pertains to foreign investments and is the risk for a U.S. investor that the exchange rates between a foreign currency and the U.S. dollar change adversely; that is, when the U.S. investor converts the foreign currency into U.S. dollars, she will get fewer dollars than previously.

Jay has recently learned that foreign investments are a good way to diversify a portfolio. Select which of these statements regarding the risks and benefits of foreign investments, specifically American depositary receipts (ADRs), is CORRECT.

Foreign taxes paid on income earned from an ADR are eligible for the foreign tax credit. The answer is foreign taxes paid on income earned from an ADR are eligible for the foreign tax credit. Although ADRs are dollar denominated, they are still subject to exchange rate risk because the dividends are declared in the local currency and converted to U.S. dollars. ADRs are receipts for shares of a foreign company, not shares of a mutual fund. Because ADRs are traded in the secondary market, they are relatively liquid and marketable investments. Foreign taxes paid are eligible for the foreign income tax credit.

Consider the following information regarding Stock A and Stock B. The market's standard deviation is 15. Stock A Stock B Correlation coefficient with market 0.20 0.80 Standard deviation 20 10 Which of the following statements are true and why? The beta of Stock A is lower than the beta of Stock B due to the impact of the correlation coefficients. The beta of Stock A is higher than the beta of Stock B because the standard deviation of Stock A is twice the standard deviation of Stock B. The ratio of Stock A's correlation coefficient to Stock B's correlation coefficient indicates that Stock B's beta is four times Stock A's beta. The correlation coefficient of Stock A suggests that the price movements of the market are likely to have little relationship with the price movements of Stock A.

I and IV According to the formula for beta, Stock A's beta is (20 ÷ 15) × 0.20 = 0.27 and Stock B's beta is (10 ÷ 15) × 0.80 = 0.53. Statement II is incorrect because it does not take into account the relative correlation coefficients. Statement III is incorrect because it does not take into account the relative standard deviations.

Consider the yield curves below. Assume yield curve 1 (YC1) changed to yield curve 2 (YC2) over a period of time. Which of the following can be interpreted from these yield curves? The Fed has been decreasing the money supply. Duration has decreased. Interest rates for all time periods have decreased. The yield curve was positive at the time of YC1.

III & IV Option I is incorrect because the yield curve rises when the Fed tightens the money supply. Option II is incorrect because duration and interest rates are inversely related; when rates fall, duration rises. LO 2.4.1

A major source of systematic risk when investing in foreign securities is exchange rate risk. To minimize the effect of exchange rate risk, an investor can hedge with currency futures. Which of the following illustrate appropriate times for an investor who owns a foreign security to hedge? The foreign security is expected to rise in price and the dollar is expected to fall relative to the foreign currency. The foreign security is expected to remain relatively stable in price and the dollar is expected to fall relative to the foreign currency. The foreign security is expected to fall in price and the dollar is expected to rise relative to the foreign security. The foreign security is expected to fall in price and the dollar is expected to remain relatively stable relative to the foreign security.

III only Explanation A hedge should be initiated if the U.S. dollar is expected to rise; it is not needed if the dollar is expected to fall or remain relatively stable.

Which of the following is a written document that sets forth a client's objectives, sets limitations on the portfolio manager, provides guidance to the portfolio manager, and provides a means for evaluating performance?

Investment policy statement The answer is investment policy statement. The investment policy statement sets forth a client's objectives, sets limitations on the portfolio manager, provides guidance to the portfolio manager, and provides a means for evaluating performance.

Which one of these statements regarding tangible or collectible assets is INCORRECT?

Investors in politically stable countries, such as the United States, cannot benefit from owning gold. The answer is investors in politically stable countries, such as the United States, cannot benefit from owning gold. Gold is considered by many to be an investment product held in times of political uncertainty in emerging market economies. If political unrest does occur and gold demand increases, U.S. investors can benefit from the price appreciation likely to occur.

Which of the following statements regarding performance measures is CORRECT?

Jensen's alpha may be used by itself to judge an investment. Beta is the risk measure for alpha, but Sharpe uses standard deviation as its risk measure. Therefore, the reliability of beta is relevant for alpha. Jensen's alpha can be used by itself to judge an investment; the Sharpe ratio must be used in comparison with another Sharpe ratio in judging an investment. A negative alpha indicates the investment did not perform as well as expected given the risk taken. For example, an alpha of -1 means the investment underperformed by 1% compared to what it was expected to return. Accordingly, a negative alpha does not necessarily mean the investment lost money. LO 7.2.1

Identify the types of bonds that are subject to the most default risk.

Junk bonds, sometimes referred to as high-yield bonds, are subjected to the most default risk. Obligations of the U.S. government are free from default risk. AA rated bonds are not free from default risk, but they are less likely to default than junk bonds.

Ophelia is considering the purchase of the Quest Mutual Fund, which has a beta of 1.2, a standard deviation of 15, and a return of 11%. The current risk-free rate is 4%, and the market risk premium is 7%. Should Ophelia purchase the fund?

No, because of the fund's Sharpe ratio. This question can be answered with or without a calculation. We have nothing to compare Quest Mutual Fund to, and both Sharpe and Treynor are comparative measures, so neither can be used to evaluate the fund. The answer "Yes, because the fund has an 11% return" just restates the return of the fund without taking risk into account. The calculation would be as follows: a=¯¯¯rp−[¯¯¯rf+(¯¯¯¯rm−¯¯¯rf)βp]a=11−[4+7(1.2)]a=11−[12.40]a=−1.40a=rp¯−[rf¯+(rm¯−rf¯)βp]a=11−[4+7(1.2)]a=11−[12.40]a=−1.40 The fund has a negative alpha of 1.40, meaning the fund manager has not obtained the return she should for the amount of risk taken.

PQR stock has a current dividend of $1.75 that has been growing at a constant rate of 8% per year. If the stock is currently selling for $100 and your required rate of return is 10%, would you buy the stock at today's price?

No, because the stock is overvalued on the basis of the constant growth dividend discount model. The answer is no, because the stock is overvalued on the basis of the constant growth dividend discount model. On the basis of the constant growth dividend discount model, the intrinsic value of XYZ stock is $94.50, calculated as follows: D0(1 + g) ÷ (r - g) = $1.75(1.08) ÷ (0.10 - 0.08) = $94.50. Because XYZ stock is currently selling for $100 per share, it is overvalued in the market and the investor should not buy the stock.

Which of these combinations of events is likely to cause the largest loss on a transaction for a German investor who invests in a Japanese stock? Gain/Loss on Japanese StockGain/Loss on Euro vs. YenA5% gain10% devaluation of euro vs. yenB5% loss10% revaluation of euro vs. yenC5% gain10% revaluation of euro vs. yenD5% loss10% devaluation of euro vs. yen

Option B Explanation The answer is Option B. One of the options with the 5% loss on the stock is likely to provide the largest loss if there also is a loss on the currency translation. When a German investor buys a Japanese stock, he or she wants the euro to depreciate against the yen; therefore, a revaluation of the euro vs. the yen will cause a currency loss in addition to the stock loss, providing the greatest overall loss.

All of these statements correctly describe the price-to-earnings divided by growth (PEG) ratio except

PEG is an indication of how much an investor is paying for a specific revenue stream. The answer is PEG is an indication of how much an investor is paying for a specific revenue stream. Companies with a lower PEG ratio have a higher expected rate of return and vice versa. The PEG ratio is a measure of relative valuation that can be used to compare companies with different growth rates. Proponents of the PEG ratio believe that companies with a low PEG ratio will have higher rates of return. The price-to-sales ratio is an indication of how much an investor is paying for a specific revenue stream.

Which statement regarding indifference curves is CORRECT?

Represent all points where an investor is equally satisfied with the risk/return trade-off The answer is represent all points where an investor is equally satisfied with the risk/return trade-off. Indifference curves express an individual's preference regarding two items: risk and return. The more risk averse an investor is, the steeper the slope is for that investor's indifference curve. Conversely, the less risk averse an investor is, the flatter the slope for that investor's indifference curve.

Lloyd is a dealer in government securities. He has purchased government securities from another dealer, Fred, and has agreed to sell them back at a later date. From Lloyd's perspective, which transaction has been executed?

Reverse repurchase agreement Lloyd, as the buyer, has entered into a reverse repurchase agreement, and Fred, as the seller, has entered into a repurchase agreement.

Calculate the intrinsic value of a call option that trades at $4, with an exercise price of $35, and has a current underlying stock price of $33.

The answer is $0. The intrinsic value of the call option equals $0, because $33 is less than $35. The call premium is $400 ($4 × 100 shares); therefore, the market feels the stock price will appreciate before the call expires.

Calculate the present value of a five-year bond with a coupon rate of 5.50% (paid semiannually) if similar quality bonds are currently yielding 4.35%.

The answer is $1,051.18. The present value of the bond is $1,051.18, calculated as follows: END Mode, 2 P/YR, 5, DOWNSHIFT, N = 10; I/YR = 4.35; PMT = 27.50 (5.50% × 1,000 ÷ 2); FV = 1,000; solve for PV = -1,051.18, or $1,051.18. LO 4.1.1

ABC Corporation issued bonds with a 10-year maturity, $1,000 par value, and 7% coupon rate (paid semiannually). Two years after issue, interest rates on similar bonds fell to 5.75%. What price should ABC Corporation bonds sell for in the secondary market?

The answer is $1,079.26. This answer is calculated as follows: END Mode, 2 P/YR FV = 1000 PMT = 7% x 1000 ÷ 2 = 35 8, DOWNSHIFT, N = 16 I/YR = 5.75 Solve for PV = -1,079.26, or $1,079.26 LO 4.1.1

XYZ Corporation issued bonds with a 25-year maturity, $1,000 par value, and 5.75% coupon (paid semiannually). Five years after issue, interest rates on similar bonds fell to 4.75%. Calculate the price that XYZ bonds should sell for in the marketplace.

The answer is $1,128.20. XYZ bonds should sell for $1,128.20, calculated as follows: END Mode, 2 P/YR FV = 1,000 PMT = 28.75 (57.50 ÷ 2) 20, DOWNSHIFT, N = 40 I/YR =4.75 Solve for PV = -1,128.1979, or $1,128.20 LO 4.1.1

Tim began purchasing BLT, Inc., mutual fund shares several years ago. He has followed a dollar-cost averaging approach by investing $1,000 each year for five years. The following data depicts Tim's purchases: YearInvestmentShare Price1$1,000$1202$1,000$1003$1,000$1184$1,000$975$1,000$130 What is Tim's average cost per share?

The answer is $111.58. Calculate the number of shares purchased each year, then total the shares purchased. The total investment is then divided by the total number of shares purchased ($5,000 ÷ 44.809 = $111.58).

LJM stock has a current annual dividend of $2.50 per share that is expected to remain constant. Using the perpetuity dividend discount model, what is LJM's market price if an investor's required rate of return is 10%?

The answer is $25.00. Using the no-growth (perpetuity) dividend discount model, V = D ÷ r, the intrinsic value of the stock is $25.00 ($2.50 ÷ 0.10).

CCC stock paid a dividend this year of $3, and this dividend is expected to grow at a rate of 10% for the next two years and at a rate of 5% thereafter. Assuming Jackie expects to sell the stock in three years, and her required rate of return is 12%, what is the price she should be willing to pay for CCC stock?

The answer is $49.25. Use the multistage growth dividend discount model to calculate the stock's intrinsic value. Step 1: Compute the value of each future dividend until the growth rate stabilizes (Years 1-2). D1 = $3.00 × 1.10 = $3.30 D2 = $3.30 × 1.10 = $3.63 Step 2: Use the constant growth dividend discount model to compute the remaining intrinsic value of the stock at the beginning of the year when the dividend growth rate stabilizes (Year 3). D3 = $3.63 × 1.05 = $3.81 V = $3.81 ÷ (0.12 - 0.05) = $54.45 Step 3: Use the uneven cash flow method to solve for the net present (intrinsic) value of the stock. CF0 = $0 CF1 = $3.30 CF2 = $3.63 + $54.45 = $58.08 I/YR = 12% Solve for NPV = $49.25 The intrinsic value of the stock is $49.25.

LFM Corporation has an estimated free cash flow to equity (FCFE) of $2.50 per share in the current year. Moreover, its FCFE is expected to grow at a constant rate of 2% per year. Assuming an institutional investor has a required rate of return of 6.5%, calculate the intrinsic value of LFM stock.

The answer is $56.67. The formula for the discounted free cash-flow model: V = FCFE1 ÷ (r - g) = ($2.50 × 1.02) ÷ (0.065 - 0.02) = 2.55 ÷ 0.045 = 56.6667, or $56.67

BC Corporation pays a current annual dividend of $0.75 per share. This dividend is expected to grow at a 30% rate per year during Years 1 and 2. After Year 2, the company's dividend is expected to grow at a constant rate of 8%. What is the value of the stock today, assuming a required rate of return of 10%?

The answer is $58.50. Using the multistage growth dividend discount model, the value of the stock equals $58.50. Compute the value of each future dividend until the growth rate stabilizes (Years 1-2). D1 = $0.75 × 1.30 = $0.9750 D2 = $0.9750 × 1.30 = $1.2675 Use the constant growth dividend discount model to compute the remaining intrinsic value of the stock at the beginning of the year when the dividend growth rate stabilizes (Year 3). D3 = $1.2675 × 1.08 = $1.3689 V = $1.3689 ÷ (0.10 - 0.08) = $68.4450 Use the uneven cash flow method to solve for the net present (intrinsic) value of the stock. CF0 = $0 CF1 = $0.9750 CF2 = $1.2675 + $68.4450 = $69.7125 I/YR = 10% Solve for NPV = $58.50 LO 4.3.1

ase Study Question The Pratts are considering purchasing JKL stock based on a stock tip from their neighbor, Alicia. Alicia has provided them with the following information: Current dividend: $2.50 Expected dividend growth rate for years 1 and 2: 8% Expected dividend growth rate for years 3+: 10% Based on this information, what is the intrinsic value of JKL stock if the Pratts have a required rate of return for this investment of 14%?

The answer is $66.37. Using the multistage growth dividend discount model, calculate JKL stock's intrinsic value: Step 1: Compute the value of each future dividend until the growth rate stabilizes (Years 1-2). D1 = $2.50 × 1.08 = $2.70 D2 = $2.70 × 1.08 = $2.92 Step 2: Use the constant growth dividend discount model to compute the remaining intrinsic value of the stock at the beginning of the year when the dividend growth rate stabilizes (Year 3). D3 = $2.92 × 1.10 = $3.21 V = $3.21 ÷ (0.14 - 0.10) = $80.25 Step 3: Use the uneven cash flow method to solve for the net present (intrinsic) value of the stock. CF0= $0 CF1= $2.70 CF2 = $2.92 + $80.25 I/YR= 14% Solve for NPV = $66.37 LO 9.2.2

JEM Corporation always pays a dividend of $5.50 per share. Calculate the intrinsic value of the stock assuming a required rate of return of 8% and a risk-free rate of return of 4%.

The answer is $68.75. The intrinsic value of the stock is $68.75 ($5.50 ÷ 0.08) using the no-growth (perpetuity) dividend discount model.

LJM Corporation has a bond issue with a coupon rate of 8% and seven years remaining until maturity. Assuming a par value of $1,000 and semiannual coupon payments, calculate the intrinsic value of the bond if current market conditions justify a 10% required rate of return.

The answer is $901.01. The intrinsic value of LJM's bond is $901.01, calculated as follows: END Mode, 2 P/YR 7, DOWNSHIFT, N = 14 10 = I/YR 8% × $1,000 ÷ 2 = 40 = PMT 1000 = FV Solve for PV = -901.01, or $901.01. LO 4.1.1

Henry owns a 10-year bond with a coupon rate of 4.85% (paid semiannually). Assuming the comparable yield for this quality bond is currently 5.5%, calculate the intrinsic value of his bond.

The answer is $950.51. The intrinsic value of his bond is $950.51, calculated as follows: END Mode, 2 P/YR, 10, DOWNSHIFT, N = 20; I/YR = 5.5; PMT = 24.25 (4.85% × 1,000 ÷ 2); FV = 1,000; solve for PV = -950.51, or $950.51.

Lauren's bond has a current market value of $987.56 and Macaulay duration of 3.2. Assuming the bond's yield to maturity (YTM) changes from 6.5% to 6%, calculate the estimated percent change in the price of the bond and the new expected market price of the bond.

The answer is +1.5%, $1,002.37. The formula for determining the change in the price of the bond: ΔP/P = −D[Δy ÷ (1 + y)] ΔP/P = −3.2[(0.06 − 0.065) ÷ (1 + 0.065)] ΔP/P = −3.2(-0.004695) = 0.01502, or 1.5% This means that the bond's price should increase by 1.5% and sell for $1,002.37 (a slight premium to par) in the secondary market.

An analysis of the monthly returns for the past year of a mutual fund portfolio consisting of two funds revealed these statistics Fund AFund BTotal return12%15%Standard deviation9%26%Percentage of portfolio35%65%Correlation coefficient (R)0.32 What is the coefficient of determination (R2) of Fund A and Fund B?

The answer is 0.10. The coefficient of determination is the square of the correlation coefficient (0.32)2 = 0.32 × 0.32 = 0.1024, or 10%.

An investor is researching a mutual fund. Last year this fund had a total return of 12% when the stock market had a 10% return. This fund has a beta of 1.2 and a standard deviation of 14%. The risk-free rate of return is 5%. What is the Sharpe ratio for this fund?

The answer is 0.50. The Sharpe ratio is (the investment return - the risk-free return) divided by the investment's standard deviation. Therefore, (0.12 - 0.05) ÷ 14 = 0.07÷0.14 = 0.50. LO 7.2.1

A client has a $1.2 million portfolio consisting of these four stocks: 1.$300,000ABC @1.1 beta2.$225,000RTR @0.7 beta3.$405,000XYZ @0.3 beta4.$270,000PDQ @1.3 beta What is the beta of the portfolio as a whole?

The answer is 0.8. This is the weighted average of beta of components, which is calculated as follows: $300,000 ÷ 1,200,000 = 0.25 weighting × 1.1 beta =0.2750$225,000 ÷ 1,200,000 = 0.1875 weighting × 0.7 beta =0.1313$405,000 ÷ 1,200,000 = 0.3375 weighting × 0.3 beta =0.1013$270,000 ÷ 1,200,000 = 0.225 weighting × 1.3 beta =0.2925Weighted Average Beta:0.8001 LO 6.2.1

Keegan is an analyst for Global Growth and Income Mutual Fund. For the past five years, the fund has returned -20%, 17%, 5%, 15%, and -7%, respectively. Calculate the geometric mean of these returns.

The answer is 1.0018%. The geometric mean return is calculated as follows: PV = -1 FV = (1 - 0.20)(1 + 0.17)(1 + 0.05)(1 + 0.15)(1 - 0.07) = 1.0511 N = 5 Solve for I/YR = 1.0018, or 1.0018%

A stock that you are researching has an expected return of 22%, a beta of 1.2, a correlation coefficient of 0.65 with the Russell 2000, an R2 of 0.38 with the S&P 500, and a standard deviation of 28%. Which one of these is the stock's coefficient of variation?

The answer is 1.27. CV = standard deviation of asset ÷ expected return of asset, 28% ÷ 22% = 1.27.

Juan has an investment portfolio consisting of 30% MIJ stock with a beta of 1.76, 40% ABC stock with a beta of 0.98, and 30% LFM stock with a beta of 2.09. What is the weighted beta for Juan's portfolio?

The answer is 1.547. The portfolio's weighted beta is calculated as follows: (0.30 × 1.76) + (0.40 × 0.98) + (0.30 × 2.09) = (0.528 + 0.392 + 0.627) = 1.547 With a beta higher than 1.0, the portfolio is much more volatile than the overall market.

Mark owns a corporate bond with a coupon rate of 6.78%. Assume the annual inflation rate is 2.5% and he is in the 35% federal marginal income tax bracket. Calculate his after-tax, inflation-adjusted rate of return on this bond.

The answer is 1.86%. First, calculate Mark's after-tax rate of return on the corporate bond [0.0678 × (1 - 0.35)] = 0.04407, or 4.41%. Next, calculate the after-tax, inflation-adjusted rate of return {[(1 + 0.0441) ÷ (1 + 0.025)] - 1} × 100 = 1.8634, or 1.86%.

Assume an investor has purchased a bond with the following characteristics: Seven years to maturity $1,000 face value 6% annual coupon (paid semiannually) 8.43% current yield $711.74 current market price Which of the following is the bond's yield to maturity?

The answer is 12.25%. The bond's yield to maturity is calculated as follows: END Mode, 2 P/YR PV = -711.74 FV = 1000 PMT = 6% × 1,000 ÷ 2 = 30 7, DOWNSHIFT, N = 14 I/YR = 12.2508, or 12.25% LO 7.4.1 Previous NextGo to Summary

Assume an investor purchased $10,000 of Fund ABC at the beginning of Year 1. Subsequently, he made investments at the beginning of Years 2, 3, and 4 of $1,000, $5,000, and $8,000, respectively. At the beginning of Year 5, the fund was worth $33,000. What was the internal rate of return (IRR) on this fund?

The answer is 12.79%. This problem involves calculating the IRR/YR for uneven cash flows per the following inputs. Using the HP 10bII+: (10,000) CFj, (1,000) CFj (5,000) CFj (8,000) CFj 33,000 CFj SHIFT, IRR/YR = 12.79%

Brett bought 500 shares of WCA stock at $27 per share on margin (50% initial margin percentage) with an annual margin interest rate of 5.25%. After one year, he sold the shares for $44 per share. The stock did not pay dividends during his holding period. Calculate Brett's holding period rate of return using margin.

The answer is 120.68%. Brett's holding period rate of return using margin is 120.68% [($22,000 - $13,500 - $354.38) ÷ $6,750]. With a margin account, Brett's initial investment will be 50% of the total purchase price of $13,500. Margin interest for the year is $354.38 ($6,750 × 0.0525).

Michael purchased 800 shares of ABC stock for $75 per share. The stock paid a $1.20 dividend per share at the end of the year, and there was a 2-for-1 stock split during the year. Assuming the value of his investment at the end of the year was $66,000, calculate the holding period return for the investment.

The answer is 13.2%. The investment's holding period return is calculated as [($66,000 − $60,000) + ($1.20 × 800 × 2)] ÷ $60,000 = 13.2%. The dividend is based on 1,600 shares because of the 2-for-1 stock split.

Based on the following information, which of the following is the expected rate of return for Softco Corporation? Stock's beta0.80Forecasted market rate of return15%Risk-free rate of return6.5%

The answer is 13.30%. Using the capital asset pricing model, the expected rate of return is 13.30% [6.5% + (15% - 6.5%)0.80].

Allison purchases 200 shares of ADM stock for $23 per share and makes subsequent purchases at the end of the following years: Year 1:50 sharesat$26/shareYear 2:75 sharesat$29/shareYear 3:25 sharesat$36/share At the end of Year 4, ADM is trading for $41 per share. There have been no dividends paid during the holding period. Calculate the annualized dollar-weighted return on Allison's investment for this four-year period.

The answer is 16.05%. To use the uneven cash flow method to solve for the internal rate of return: CF0 (4,600) (200 × 23) CF1 (1,300) (50 × 26) CF2 (2,175) (75 × 29) CF3 (900) (25 × 36) CF4 14,350 (350 × 41) Solve for IRR/YR = 16.0531, or 16.05%

Mike, a stock analyst, has determined several factors affecting the expected return on a stock. In addition, he has estimated their sensitivity coefficients and associated risk premiums. FactorSensitivity CoefficientRisk PremiumUnemployment0.76%Inflation0.84%Demand1.25% Assuming the current risk-free rate of return is 3%, calculate the expected return of the stock using the arbitrage pricing theory (APT).

The answer is 16.40%. The stock has an expected return of 16.40%, calculated as follows: using APT, ri = 0.03 + (0.7 × 0.06) + (0.8 × 0.04) + (1.2 × 0.05) = 0.1640, or 16.40%.

Case Study Question Given the following historical return information for ABC stock: YearAnnual Rate of Return112%218%3-10%4-14%523% Which of the following is the standard deviation for this series of returns?

The answer is 16.77%. The standard deviation for this historical series of return for ABC stock is 16.77%. Using the HP 10bII+: KeystrokesDisplay[SHIFT] [C ALL]0.000012 [Ʃ+]1.000018 [Ʃ+]2.000010 [+/-][Ʃ+]3.000014 [+/-][Ʃ+]4.000023 [Ʃ+]5.0000[SHIFT] [Sx,Sy]16.7690, or 16.77%

Jonathan purchased 500 shares of CPM stock for $12 per share. At the end of the first year, he made another purchase of 500 shares at a stock price of $12 per share. At the end of the third year, he sold all of the stock for $17 per share. In addition, the stock paid a dividend of 0.35 per share at the end of each year. Calculate the dollar-weighted return to Jonathan over the three-year period.

The answer is 17.46%. Jonathan earned a dollar-weighted rate of return of 17.46% on CPM stock over the three-year period, calculated as follows: CF0 = −12 × 500 = −6,000 CF1 = (−12 × 500) + (0.35 × 500) = −5,825 CF2 = (0.35 × 1,000) = 350 CF3 = (0.35 × 1,000) + (17 × 1,000) = 17,350 Solve for the internal rate of return (IRR/YR) = 17.4626, or 17.46%

Calculate the expected rate of return for a stock with a beta of 2.0 when the risk-free rate is 6% and the return on the market is 12%.

The answer is 18%. Using the capital asset pricing model (CAPM), the expected rate of return is 18% [6% + (12% - 6%)2.0].

Carolyn owns a corporate bond with a coupon rate of 3.60%. Currently, the inflation rate is 1.50%. Calculate the real rate of return on this bond.

The answer is 2.07%. The real rate of return on this bond is calculated as follows: [((1 + 0.036) ÷ (1 + 0.015)) - 1] × 100 = 0.0207 × 100 = 2.07%

Johnny owns a municipal bond with a coupon rate of 4.25%. Assuming the annual inflation rate is 1.65%, calculate Johnny's real rate of return on his bond.

The answer is 2.56%. Johnny realized a real rate of return of 2.56%. Real rate of return = {[(1 + 0.0425) ÷ (1 + 0.0165)] - 1} × 100 = 2.5578, or 2.56%.

Assume a 3-year, $1,000 par value corporate bond is currently trading for $959.53. The bond has a coupon rate of 4% (paid once per year) and a yield to maturity of 5.50%. Calculate the duration for this bond.

The answer is 2.8835 years. The duration for this bond is 2.8835 years, calculated as follows: YearCash Flow(CF)Present Value(PV) of CFPV × Year140.0037.9137.91240.0035.9471.8831,040.00885.682,657.04959.532,766.83 To solve for the PV of a given CF (example Year 1): FV = 40, N = 1, PMT = 0, 5.5 = I/YR, solve for PV. Divide the sum in the last column (2,766.83) by the total PV/market price of the bond (959.53) to derive the duration of 2.8835 years.

Nancy bought 50 shares of ABC stock for $50 per share. She made additional purchases at the end of each of the following years: Year 1: 10 shares at $52 per share Year 2: 10 shares at $53 per share Year 3: 10 shares at $45 per share ABC stock has not paid any dividends during her holding period. At the end of Year 4, the stock is trading for $55 per share. Which of the following is Nancy's return over the past four years on ABC stock (assume she sells the stock for the current trading price)?

The answer is 2.95%. The dollar-weighted return is calculated as follows: CF0 (2,500) [50 × $50] CF1 (520) [10 × $52] CF2 (530) [10 × $53] CF3 (450) [10 × $45] CF4 4,400 [80 × $55] Solve for IRR/YR = 2.95% LO 7.1.1

Strahan Corporation's current annual common stock dividend is $3 and is expected to grow by 15% during the next year. The stock's current market price is $35 per share. If Strahan stock had a $30 market price one year ago, calculate the stock's total return.

The answer is 26.67%. A stock's total return (TR) = (the dividend received during a given period + the change in the stock's price during the same period) ÷ by the stock's current market value at the beginning of the period. Therefore, TR = ($3 + $5) ÷ $30 = 26.67%.

Your client, Jackson, is considering adding XYZ Mutual Fund to his portfolio. The fund has a correlation coefficient of 0.55 with the S&P 500, and he wants to know how much systematic risk the fund has when compared to this benchmark. You would advise him that the percentage of systematic risk is

The answer is 30%. You have been provided with the correlation coefficient (R) and what you need is the coefficient of determination (R2). Keystrokes are 0.55, DOWNSHIFT, "+" key = 0.3025. This means that 30% of the price movement of the fund is explained by the S&P 500, and the other 70% is not. Stated another way, there is 30% systematic risk, and 70% unsystematic risk.

A wash sale for tax purposes occurs when a person sells a security and repurchases it within ___ days before or after the sale.

The answer is 30. A wash sale occurs if the taxpayer sells or exchanges stock or securities for a loss and, within 30 days before or after the date of the sale or exchange, acquires similar securities. If this event occurs, the basis of the new stock or securities will include the unrecovered portion of the basis of the formerly held stock or securities.

Leslie purchased a 10-year bond with a coupon rate of 4.75% paid semiannually. The bond has a current market price of $1,035. Calculate the yield to maturity (YTM) for Leslie's bond.

The answer is 4.3154%. The bond's YTM is calculated as follows: END Mode, 2 P/YR PV = −1035 FV = 1000 PMT = 4.75% × 1000 ÷ 2 = 23.75 10, DOWNSHIFT, N = 20 Solve for I/YR = 4.3154% The YTM for Leslie's bond is lower than its coupon rate because the bond is trading at a premium. LO 7.4.1

Eight years ago, ABC Company issued a 20-year bond with a 4% coupon rate. Due to a recent decline in market interest rates, the company decided to call the bonds for 103% of par value. Calculate the rate of return for an investor who purchased the bond at issue for par and surrendered it today for the call price.

The answer is 4.32%. The yield to call on this issue is calculated as follows: END Mode, 2 P/YR PV = -1000 FV = 1030 PMT = 4% x 1000 ÷ 2 = 20 8, DOWNSHIFT, N = 16 Solve for I/YR = 4.32% LO 7.4.1

Assume the nominal return on 30-year U.S. T-bonds is 6.5%, and the inflation rate is 1.75%. Which of the following is the real rate of return on the T-bonds?

The answer is 4.67%. The real rate of return is 4.67%, calculated as follows: {[(1 + 0.065) ÷ (1 + 0.0175)] -1} × 100 = 0.0467, or 4.67%.

Crowder made an investment that paid him an 8% nominal rate of return for the year in which he held the investment. During that year, the inflation rate was 3%. Based on this information, calculate Crowder's inflation-adjusted return (real return). A) 3.08%

The answer is 4.85%. The inflation-adjusted return (IAR) is computed as: IAR = [((1 + nominal rate of return) ÷ (1 + inflation rate)) − 1] × 100 = ((1.08 ÷ 1.03) − 1) × 100 = 4.8544, or 4.85% LO 7.1.1

Jefferson originally purchased 100 shares of XYZ stock for $45 per share. The stock is currently trading at $60 per share. The stock paid dividends of $2 per share in year 1 and $2.30 per share in year 2 (all paid at year end). If Jefferson has held the stock for two years, what is his holding period return?

The answer is 42.9%. This is calculated as: [($60 - $45) + $2 + $2.30] ÷ $45 = 42.9%.

Billie purchased a 10-year U.S. Treasury bond with a 6.5% coupon paid semiannually. Assuming the bond is currently trading at $1,075, calculate its yield to maturity (YTM).

The answer is 5.51%. END Mode, 2 P/YR PV = -1,075 FV = 1,000 PMT = 32.50 (6.5% × 1,000 ÷ 2) 10, DOWNSHIFT, N = 20 Solve for I/YR = 5.51%. The YTM on the bond is 5.51%, which is lower than the coupon rate of 6.5%, further validating that the bond is trading at a premium.

Five years ago, XYZ Company issued a 20-year bond with a 4.75% coupon paid semiannually. The bond may be called at 104% of par, 10 years after issue. Assuming the bond is currently selling for $990, calculate the bond's yield to call.

The answer is 5.68%. The bond's yield to call is calculated as follows: Note: XYZ Company has the option to call the issue in five years. END Mode, 2 P/YR PV = -990 5, DOWNSHIFT, N = 10 PMT = 23.75 (4.75% x 1000 = 47.50 ÷ 2) FV = 1,040 (1,000 × 1.04) Solve for I/YR = 5.68% LO 7.4.1

Security A has a standard deviation of 12% and the market has a standard deviation of 16%. The correlation coefficient between Security A and the market is 0.75. What percent of the change in Security A's price can be explained by changes in the market?

The answer is 56%. Because the correlation coefficient is 0.75, the coefficient of determination (R2) is 0.5625, or 56%. Therefore, only 56% of investment returns can be explained by changes in the market (i.e., systematic risk represents 56%).

Seven years ago, KLO Industries issued a 15-year bond with a 6% coupon rate. The bonds are currently rated BB+. Due a decline in interest rates, the company decided to call the bonds for 106% of par value. Calculate the rate of return for an investor who purchased the bond at issue for par and surrendered it today for the call price.

The answer is 6.69%. The yield to call on this issue is 6.69%, calculated as follows: END Mode, 2 P/YR FV = 1,060 PV = −1,000 PMT = 30 (6% x 1000 = 60 ÷ 2) 7, DOWNSHIFT, N = 14 Solve for I/YR = 6.69% LO 7.2.1

Portfolio A has a standard deviation of 55%, and the market has a standard deviation of 40%. The correlation coefficient between Portfolio A and the market is 0.50. Calculate the percentage of total risk that is unsystematic.

The answer is 75%. The coefficient of determination explains the percentage of change in the dependent variable that can be explained by changes in the independent variable. Therefore, 25% (0.50 × 0.50) of returns are explained by changes in the market. To determine the percentage of returns that are explained by unsystematic risk, subtract the systematic risk from 1. Therefore, the return explained by unsystematic risk is (1 − 0.25) = 0.75, or 75%.

If the market interest rate is 7.27%, the current yield of a bond with a 9% coupon, $1,000 par, selling for $1,120, and maturing in 10 years is

The answer is 8.04%. The current yield is the coupon payment divided by the market price of the bond: ($90 ÷ $1,120) x 100 = 8.04%. LO 7.4.1

The current yield of an 8% coupon bond, maturing in five years, and selling currently for $850 is

The answer is 9.41%. Current yield = annual interest payment ÷ current market price = $80 ÷ $850 = 0.09412, or 9.41%.

STU Corporation stock has an average rate of return of 24% and a standard deviation of 10%. The risk-free rate of return is 4%. Assuming the historical returns for STU stock are normally distributed, calculate the probability that this stock will have a return in excess of the risk-free rate of return

The answer is 97.5%. The probability of a return above 24% is 50%. The probability of a return between 4% and 24% is 47.5% (95% ÷ 2). Therefore, the probability of a return above 4% is 97.5% (50% + 47.5%). LO 6.2.2

Acme Electric Company announces a cash dividend of $0.50 per share on August 5, to be paid on September 20, the payable date. The company also announces that the record date will be August 25. Bob Johnson purchases 100 shares of Acme on August 24. Based on this information, choose the CORRECT statement regarding the dividend payment.

The answer is Bob will not receive the dividend, because he did not purchase the shares before the ex-dividend date. In order to receive the dividend, Bob must purchase the shares before the ex-dividend date. The ex-dividend date is one business day before the record date. To receive the dividend, Bob must have purchased the shares by August 23.

DIV Corporation's current market value is $50 million, with 2 million shares outstanding. The board of directors votes to pay a stock dividend of 10%. Which of the following statements is correct?

The answer is DIV Corporation's per-share stock price will be $22.73 following the stock dividend. DIV Corporation's overall market value will remain unchanged at $50 million, with 2.2 million shares outstanding. The stock price per share will be adjusted downward to $22.73 ($50,000,000 ÷ 2,200,000). LO 3.2.1

You are choosing between two investments: Mutual Fund A with a return of 12% and a standard deviation of 18%, and Mutual Fund B with a return of 8% with a standard deviation of 12%. If the risk-free rate is 3%, which fund should you choose based upon one of the risk-adjusted return measurements?

The answer is Fund A. Enough information is given to calculate the Sharpe ratio for each investment. For Fund A (12 - 3) ÷ 18 = 0.50. For Fund B (8 - 3) ÷ 12 = 0.42. The higher the ratio, the better, so Fund A is the best choice.

You have narrowed your choice down to these three mutual funds, which have these annual returns. Which fund should you choose based upon risk and return? Fund XFund YFund ZYear 1+15%+7%+12%Year 2+9%+13%+2%Year 3+5%+8%+11%

The answer is Fund Y. This is a problem that can be solved using the coefficient of variation. With the annual returns calculate the standard deviation and mean return for each of the three funds: Fund XFund YFund ZStandard deviation5.03323.21465.5076Mean return9.66679.33338.3333Coefficient of variation0.52070.34440.6609 The lowest CV is the correct answer. Sample keystrokes for Fund X (repeat for the other funds): 15 ∑+ 9 ∑+ 5 ∑+ SHIFT, Sx,Sy (8 key) = 5.0332 For the mean return: SHIFT, x,y (7 key) = 9.6667 CV = standard deviation ÷ mean CV = 5.0332 ÷ 9.6667 = 0.5207

Which of these statements concerning portfolio diversification is CORRECT? By increasing the number of securities in a portfolio, the total risk would be expected to fall at a decreasing rate. Total risk is reduced as diversification is increased. The benefits of diversification are not realized until at least 30 individual securities are included in the portfolio. Diversification reduces the portfolio's expected return because diversification reduces a portfolio's total risk.

The answer is I and II. Studies have shown that an investor only needs about 15-20 assets to fully diversify a portfolio. The main attraction of diversification is the reduction of risk without an accompanying loss of return.

Which of these statements are CORRECT of mutual fund dividend distributions? The fund pays dividends from net investment income. A single taxpayer may exclude $100 worth of dividend income from taxes annually. An investor is liable for taxes on distributions whether a dividend is a cash distribution or is reinvested in the fund. An investor is not liable for taxes if he or she automatically reinvests distributions.

The answer is I and III. Mutual funds pay dividends from net investment income, and shareholders are liable for taxes on all distributions, whether reinvested or taken in cash.

Which of the following are reasons why a client's preferences, understanding, and experience may influence the management of his or her portfolio? These factors affect investment strategies that might be used by the planner. Consideration of these factors relieves the planner of any fiduciary duty. Consideration of these factors eliminates the need for due diligence of securities. Investment choices must be checked for suitability, with these factors kept in mind.

The answer is I and IV. Investment strategies used by a planner should tie closely to the client's goals, risk tolerance, investment preferences, and other intangible factors. Clients with little understanding may not tolerate complicated investment strategies and products. Unsuitability lawsuits are very common in the investment universe. By matching as closely as possible a client's suitability factors with investment products, an advisor may be able to defeat claims of unsuitability.

Identify which of these statements regarding unit investment trusts (UITs) is CORRECT. Units are sold at net asset value plus a commission for the broker executing the transaction. Like stocks, UITs are traded on the major exchanges. During the term of the trust, unit holders are taxed in the same manner as owners of variable annuities. Upon maturity, the securities are generally liquidated and the proceeds distributed to the investor or trust beneficiaries.

The answer is I and IV. Statements II and III are incorrect. UITs are sold in the secondary market, but not on the major exchanges. During the term of the trust, unit holders are taxed in the same manner as shareholders of mutual funds with capital gains earned by the trust passed through and taxed to the unit holders. Dividends are taxed as ordinary income in the year earned.

Identify the CORRECT statements regarding warrants. Warrants give the owner the right to purchase a specified number of shares for a specified period at a specified price. Warrants are typically written with a maturity date of nine months. Warrants must include standardized terms required by the Options Clearing Corporation. Warrants are issued by a corporation rather than written by an individual.

The answer is I and IV. Warrants typically have a maturity date of several years, not months, and are customized to fit the needs of the issuing corporation.

A client of yours, George, wants to maximize his return on an intermediate-term bond that he plans to hold until maturity. You have gathered information on the following two bonds, both of which have a $1,000 par value. Bond 1: A rated; coupon rate of 6%; matures in 6 years and pays interest semiannually; currently selling for $850; duration is 5.16 years. Bond 2: A rated; coupon rate of 10%; matures in 8 years and pays interest semiannually; currently selling for $1,100; duration is 7.15 years. Which of these bonds would you recommend to George and why? Bond 1 because it has a higher yield to maturity than Bond 2 Bond 2 because its higher coupon rate gives it a superior total return to Bond 1 Bond 2 because it has a higher duration than Bond 1

The answer is I only. Bond 1's YTM (9.32%) is higher than Bond 2's YTM (8.26%) For both calculations, END Mode, 2 P/YR. Bond 1: PV = -850 FV = 1000 PMT = 30 (6% × $1,000 ÷ 2) 6, DOWNSHIFT, N = 12 Solve for I/YR = 9.32% Bond 2: PV = -1100 FV = 1000 PMT = 50 (10% × 1,000 ÷ 2) 8, DOWNSHIFT, N = 16 Solve for I/YR = 8.26%. LO 7.4.1

Distributions of dividend and capital gains in cash to mutual fund investors are fully taxable to the investor. are added to the tax basis of the shares once taxes on the distributions are paid. decrease the taxable gain or increase the loss on sale of the shares after taxes are paid. decrease the cost basis of the shares whether or not taxes are paid

The answer is I only. If the dividends and capital gains are reinvested, the individual receives an increased tax basis. If the distributions are made in cash, there is no increase in the tax basis of the underlying securities.

Which of the following are correctly defined bond classifications? Short-term - maturities up to 5 years Intermediate-term - maturities of 5 to 7 years Long-term - maturities longer than 7 years A) I, II, and III

The answer is I only. Intermediate-term bonds have maturities 5 to 12 years. Long-term bonds have maturities of longer than 12 years.

Which of the following are considered bond classifications for multisector bond funds? Foreign bonds High-dividend-paying common stocks Commodities

The answer is I only. Multisector bond funds typically purchase three types of bonds: U.S. government bonds, high-yield corporate bonds, and foreign bonds. LO 3.3.1

Which of the following are characteristics of the Sharpe ratio? The ratio adjusts the return for variability by using standard deviation as the measure of risk. The ratio assumes that the portfolio being evaluated is well diversified. Both alpha and beta appear in the formula for the ratio. The ratio indicates by how much the realized return differs from the return expected by the capital asset pricing model.

The answer is I only. Statements II, III, and IV are true of Jensen's alpha. Statement II is true of the Treynor ratio.

You have recommended a growth mutual fund to a new client. The client considered your recommendation and asked why he should not invest in another fund that he had been following, which appeared to have a better performance over the past three years. You explained the concept of risk-adjusted performance and obtained this information about the two funds: Your FundClient FundThree-year total return13.5%14.75%Average P/E ratio20%24%Standard deviation19%23%Beta1.031.24 Which fund would you recommend based on each fund's relationship between risk and return? Your fund, because its coefficient of variation is 1.41, compared to the client's coefficient of variation of 1.56 Client fund, because its higher beta dictates that its return should also be higher, which in fact occurred Client fund, because standard deviations and betas change over time and the statistics are close enough so that the fund with the better return should be chosen

The answer is I only. The standard deviation is divided by the total return to obtain the coefficient of variation. A beta is higher does not mean that any higher return is acceptable. The client's fund has higher risk as measured by both standard deviation and beta, but taking this higher risk does not provide sufficient return based on the coefficient of variation calculation.

Analyzing financial statements of a company and performing industry analysis would be beneficial under which of the following forms of the efficient market hypothesis (EMH)? Weak Semistrong Strong Semiweak

The answer is I only. The weak form of the efficient market hypothesis (EMH) states that market prices incorporate all historical price information. However, fundamental analysis, such as analyzing financial statements, may be beneficial under the weak form of the EMH. Semiweak is not a form of the EMH.

XYZ Company issued a series of bonds 20 years ago. The bonds originally sold at par, have a 6.25% coupon rate (paid semiannually), and mature in 30 years. Five years after issue, the prevailing market interest rate for similar type bonds was 5.15%. Based on this information, identify which of the following statements are CORRECT. The bond's yield to maturity at issue was 6.25%. The bond's price five years after issue was $1,153.68. The current yield of the bond five years after issue was 5.42%. The bond was selling at a discount in the secondary market five years after issue.

The answer is I, II, and III. Because the bond was selling for a price exceeding the par value, the bond was trading at a premium. A bond selling for par at issue will have a yield to maturity equal to the annual coupon rate. Statement II: END Mode, 2 P/YR FV =1,000 PMT = 62.50 ÷ 2 = 31.25 25, DOWNSHIFT, N = 50 I/YR = 5.15 Solve for PV = 1,153.68, or $1,153.68 Statement III: Current yield = $62.50 ÷ $1,153.68 (found in Statement II) Current yield = 0.0542, or 5.42% LO 7.4.1

Which of these statements concerning the principles of real estate investing is CORRECT? Investors in undeveloped land are seeking returns primarily from capital appreciation. Effective real estate investing almost always involves a careful but extensive use of leverage. Investors in real estate are usually seeking tax benefits for mortgage interest, property taxes, and depreciation. Generally, investors in developed land experience significant taxable income in the early years of ownership.

The answer is I, II, and III. Only Statement IV is incorrect. Investors in real estate experience little taxable income in the early years due to depreciation.

Which of these statements correctly explains dollar cost averaging as a portfolio management technique? This technique involves investing a specific amount into an investment vehicle, regardless of whether the recent trend in the investment has been up or down. If prices decline, the fixed investment amount will purchase a greater quantity of the security. For the long-term investor, the presumption is that prices will eventually rise, so a lower average price translates into greater profits. If prices rise, the fixed investment amount will purchase a greater amount of the security.

The answer is I, II, and III. Only statement IV is incorrect. If prices rise, the fixed investment amount will purchase a lower amount of the security. The effect of dollar cost averaging is to increase the number of shares gradually over a long period. Because more shares are acquired when the price of the stock or mutual fund declines, the average cost per share is reduced.

Indifference curves, which represent the risk-reward trade-off that the investor is willing to make, will cross the efficient frontier in two locations. lie tangent to the efficient frontier. will not intersect the efficient frontier.

The answer is I, II, and III. The portfolio that lies at the point of tangency of an indifference curve and the efficient frontier is the optimal portfolio for the investor.

Select the CORRECT statements concerning the analysis of portfolio risk. Investors estimate the risk of a portfolio on the basis of the variability of returns. Markowitz used risk and expected return as the basis for determining efficient combinations of assets. For a given level of risk, investors prefer lower returns to higher returns. Investors base decisions solely on expected return and risk.

The answer is I, II, and IV. Only statement III is incorrect. For a given level of risk, investors prefer higher returns to lower returns.

Which of the following statements regarding the capital asset pricing model (CAPM) are CORRECT? Standard deviation is used as the measure of risk on the security market line. The capital asset pricing model formula defines the security market line. Superior performance opportunity exists if a fund's position is above the security market line. Both portfolio risk and return decrease when investors substitute risky securities for risk-free assets.

The answer is II and III. Beta is used as the measure of risk on the security market line. If risky securities are added to the portfolio, both risk and return will increase. The capital market line slopes upward indicating that as more risk is undertaken, more return should be achieved.

Which of the following statements about the efficient market hypothesis (EMH) and associated anomalies are CORRECT? An investor purchasing a high price-to-earnings (P/E) ratio is exploiting the P/E effect anomaly. An investor studying annual reports and analysts' reports in his stock selection process believes that markets are weak-form efficient. An investor who buys the securities of firms that are not followed by many analysts is trying to benefit from the neglected-firm effect. An investor who befriends the chauffeur of a firm's CEO to solicit information about the firm's plans before making investment decisions believes the markets are strong-form efficient.

The answer is II and III. The P/E effect suggests that portfolios consisting of stocks with low price-to-earnings ratios have higher average returns than do portfolios consisting of stocks with high P/E ratios. Strong-form market efficiency suggests that all public and private information is included in market prices. A person who solicits private information believes that it is possible to profit by making trading decisions based on private information and does not believe that the markets are efficient in the strong form. Weak-form efficiency suggests that all historical price and volume information is included in stock prices but that gains may be made by analyzing other publicly available information. An investor studying annual reports and analysts' reports to make stock selections indicates that the person is conducting fundamental analysis, because the investor believes that the markets are weak-form efficient.

Which of these describe differences between preferred stock and long-term bonds? Preferred stock usually has a shorter maturity than long-term bonds. Corporations receive more favorable tax treatment when investing in preferred stock than when investing in long-term bonds. Preferred stock dividends are a stronger legal obligation to the firm than interest payments on long-term bonds. The market price of preferred stock tends to fluctuate more than the market price of long-term bonds.

The answer is II and IV. Corporations receive preferential tax treatment when investing in preferred stock. The market price of preferred stocks is more volatile than long-term bonds when interest rates fluctuate.

An investment portfolio has the following three stocks: StockInvestmentBetaStock 1$8,0000.6Stock 2$22,0001.3Stock 3$12,0000.9 Which of the following are CORRECT statements about this portfolio? The weighted beta for the portfolio is 0.93. The weighted beta for the portfolio is 1.05. The portfolio is less risky than the market. The portfolio is riskier than the market.

The answer is II and IV. The portfolio weighted beta is computed as follows: [(8 ÷ 42) × 0.6] + [(22 ÷ 42) × 1.3] + [(12 ÷ 42) × 0.9] = 0.114 + 0.681 + 0.257 = 1.052, or 1.05. Because the portfolio beta is more than 1.0, the portfolio is considered riskier than the market, which has a portfolio beta of exactly 1.0.

elect which of these statements regarding Treasury notes and Treasury bonds is CORRECT. Both Treasury notes' and bonds' interest payments are income tax free at the state and federal level. Treasury notes and bonds are considered default risk free. If held for more than one year, interest paid on Treasury bonds is eligible for long-term capital gain treatment. Both Treasury notes and bonds are not traded in the secondary market.

The answer is II only. Both Treasury notes and bonds are considered default risk free. Both obligations trade in the secondary market and pay interest, which is income tax free at both the state and local level, but taxed as ordinary income in the year earned at the federal level. LO 2.1.1

Greg buys a call option on LMN Corporation's stock for an option premium of $1.50. Which of these statements is CORRECT? Greg hopes that the price of LMN stock will decline. Greg's maximum loss on the option is $150.

The answer is II only. Statement I is incorrect. An investor who buys a call option hopes the price of the underlying stock will increase. Any time an investor buys an option (either a call or a put) the maximum loss is the amount of premium paid. In this case, Greg paid an option premium of $150 ($1.50 × 100 shares). Each option contract represents 100 shares of the underlying stock.

TLP stock sells for $40 a share and pays an annual dividend of $2.75, which is expected to increase 5% annually. An investor has a required rate of return of 13%. Which of the following conclusions about TLP stock can be drawn? It is undervalued. It has an intrinsic value of $36.09. It is overvalued. It has an expected rate of return of 12.2%.

The answer is II, III, and IV. The constant growth dividend discount model may be used as follows: V = D1 ÷ (r - g) = (2.75 × 1.05) ÷ (0.13 - 0.05) = 2.8875 ÷ 0.08 = 36.09, or $36.09 With an intrinsic value that is less than the current market price, the stock would be considered overvalued and not worthy of a purchase. With option IV, the expected return is lower than the required return. In order to calculate the expected return, we can solve for it using the following steps: $40 = 2.8875 ÷ (x - 0.05) $40 (x - 0.05) = 2.8875 $40x - 2 = 2.8875 $40x = 4.8875 x = 4.8874 ÷ $40 = 0.1222, or 12.2% LO 4.3.2

Which of these statements regarding unit investment trusts (UITs) are CORRECT? A bond UIT has a yield to maturity. UIT sponsors must make a secondary market in the UITs they create. UITs have management fees lower than mutual funds. A bond UIT does not replace bonds that are called.

The answer is III and IV. A bond UIT has an estimated return but cannot offer a yield to maturity because various bonds in the UIT have different maturities and some of the bond issues might be called before maturity. UIT sponsors are not required to make a secondary market in the UITs they create. UITs are not managed so their management fees are typically lower than those of mutual funds. If a bond issue is in a UIT and subsequently called, that issue is not replaced.

Identify which of these statements concerning technical analysis is CORRECT. Technical analysis is focused on the process by which stock prices rapidly adjust to new information. Technical analysis is based on the underlying fundamentals of a stock's value. The focus of technical analysis is market timing with an emphasis on price changes. Technicians concentrate on past stock price movements to forecast future stock price movements.

The answer is III and IV. Statements I and II are incorrect. Fundamental analysis is based on the underlying fundamentals of a stock's value. Technicians concentrate on the short run, looking for short-term price movements. The focus of technical analysis is the gradual process whereby stock prices adjust to new information. Technical analysis involves analyzing past stock prices to forecast future prices.

An investor is deciding whether to make an investment in LKJ stock, which has a standard deviation of 4.3% and an expected return of 13%, or FDS stock, which has a standard deviation of 5.6% and an expected return of 10.5%. Using the coefficient of variation (CV), which of the following is the preferable investment choice?

The answer is LKJ, with a CV of 0.3308. The CV is calculated by dividing the standard deviation of an asset by its expected return. For LKJ, the CV is 0.3308 (0.043 ÷ 0.13). For FDS, the CV is 0.5333 (0.056 ÷ 0.105). Using only this information, the investor may choose to select the asset with the lower CV, which in this case, is the LKJ stock.

During his next meeting with his financial advisor, Zachary would like to compare the performance of his international investments against a benchmark. Select the appropriate benchmark to use for this comparison.

The answer is MSCI EAFE Index. The MSCI EAFE Index is used as a measure of the international securities markets. The other choices are not used to measure international investments, rather, they are used as benchmarks for domestic issues.

The current risk-free rate of return is 4%, and the market risk premium is 5%. One stock under consideration for investment has a beta of 1.3. Calculate the expected rate of return for both the market portfolio and the individual stock.

The answer is Market = 9.00%; Stock = 10.50%. The expected rate of return for the market portfolio is 9%, calculated as follows: Using the capital asset pricing model (CAPM), rp = 4% + (5% × 1.0). The market has a beta of 1.0 and the market risk premium, rather than the market return, is given. The expected rate of return for the stock is 10.50%, calculated as follows: Using CAPM, ri = 4% + (5% × 1.3) = 10.50%.

The Galaxy Fund has a standard deviation of 15, and a mean return of 9%. The Universe Fund has a standard deviation of 22, and a mean return of 13%. The Milky Way Fund has a standard deviation of 18, and a mean return of 11%. Which fund should you choose in order to minimize the risk per unit of return?

The answer is Milky Way Fund. The Galaxy Fund has a coefficient of variation of 1.67, the Universe Fund has a coefficient of variation of 1.69, and the Milky Way Fund has a coefficient of variation of 1.64. Coefficient of variation = standard deviation ÷ mean return, select the lowest number.

Mysterious Company stock has a mean return of 9% and a standard deviation of 3%. Based on this information, which of the following statements is CORRECT?

The answer is Mysterious Company is unlikely to experience a negative return. With a normal probability distribution, 68% of the returns fall within one standard deviation of the mean, 95% within two standard deviations, and 99% within three standard deviations. Therefore, Mysterious Company has less than a 0.5% chance of experiencing a negative return.

Sam holds a considerable amount of both Series EE and Series HH savings bonds. He is nearing retirement and likes the fact that his Series HH bonds pay interest semiannually and would like to exchange most of his Series EE bonds for Series HH bonds to increase his cash flow. Choose which of these statements regarding such an exchange is CORRECT.

The answer is Series EE bonds may no longer be exchanged for Series HH bonds. Until September 2004 (when Series HH bonds were no longer issued by the Treasury), the exchange of EE bonds for HH bonds was a popular way of continuing the income tax deferral on the accrued interest portion of the EE bonds. Such changes are no longer possible.

Adam is trying to evaluate the performance of his portfolio on a risk-adjusted basis. He has a nondiversified portfolio of large-cap stocks. He knows there are different measures of risk-adjusted performance and is not sure which one to use. Which of the following is the most appropriate measure to use?

The answer is Sharpe, because when a portfolio represents the entire investment fund, standard deviation is a better measure of risk. The Sharpe ratio must be computed for a benchmark, which is then compared to the performance of a portfolio. The Treynor ratio is used when the performance of a subportfolio is measured. Alpha is computed using the Jensen performance measure. LO 7.2.1

Lynn and Stuart Wagman are a middle-aged couple who would like to add an equity investment to their portfolio. They require a 12% rate of return and are considering the purchase of one of these common stocks Stock 1: Dividends currently are $1.50 annually and are expected to increase 8% annually; market price = $35. Stock 2: Dividends currently are $2.25 annually and are expected to increase 7% annually; market price = $50. Using the dividend growth model, determine which stock would be more appropriate for the Wagmans to purchase at this time?

The answer is Stock 1, because the expected return on investment is great than the Wagmans' required return. Compute the intrinsic value and expected return first, then determine which stock should be purchased. The intrinsic value of Stock 1 is $40.50; of Stock 2, $48.15. The expected return on investment of Stock 1 is 12.6%; of Stock 2, 11.8%. LO 4.3.1

Stock A has an expected mean return of 15% and a standard deviation of 22%; Stock B has an expected mean return of 11% and a standard deviation of 13%; and Stock C has an expected mean return of 18% and a standard deviation of 24%. You want to recommend one of these stocks to a client who is most interested in owning stocks that are more likely to deliver the expected mean return. Which stock should you recommend to meet this client's requirement?

The answer is Stock B. The coefficient of variation is a measure of the degree of variation of returns compared with the expected mean return. The security with the lowest coefficient of variation is the one most likely to deliver periodic returns closest to its expected return. The coefficients of variation of the three securities are 1.47 for Stock A, 1.18 for Stock B, and 1.33 for Stock C. Stock B should be recommended.

Consider this information regarding two possible investments: Stocks J and K. Stock J: Expected return: 11.5% Standard deviation: 8% Stock K: Expected return: 8.2% Standard deviation: 6% Identify which of these investments you would prefer and why.

The answer is Stock J because it has the lowest coefficient of variation. The stock with the lower coefficient of variation (CV) provides the least amount of risk for a given level of return. CV = standard deviation of asset ÷ expected return of asset. Stock J: CV = 0.08 ÷ 0.115 = 0.6957 Stock K: CV = 0.06 ÷ 0.082 = 0.7317 LO 6.2.3

Which of the following statements regarding futures contracts is correct?

The answer is a futures contract is an agreement between two parties to make or take delivery of a specified amount of a commodity or financial asset at a future time, place, and unit price. To complete a futures contract, delivery of the commodity or asset may be made. But more often, the buyer (or holder) simply purchases an offsetting contract and cancels the original position. Hedging involves purchasing a futures contract in the opposite position of that which is currently held. Futures contracts are traded on an exchange such as the Chicago Mercantile Exchange.

Which of the following statements regarding modern portfolio theory is correct?

The answer is a portfolio that lies at the point where the indifference curve is tangent to the efficient frontier is the optimal portfolio. Statements III and IV are not correct. A portfolio that lies below the SML is overvalued because its expected return is lower than the required return plotted on the SML. A portfolio that lies below the efficient frontier is inferior to one that lies on the efficient frontier. Each portfolio on the efficient frontier offers the highest possible return for a particular level of risk.

A firm declares a $3.00 cash dividend to its shareholders. The firm has issued dividends of only $0.07 per share for each of the last 15 quarters, and market analysts anticipate a similar dividend this quarter. In an efficient market, one would expect

The answer is a price change upon the announcement. In an efficient market, the price of the stock will represent all public information. Because the increase in the dividend was not public knowledge until it was declared, no price change would take place before the announcement. A price change, representing the increase in dividends, would be expected immediately after the information became public.

A transaction whereby an investor sells a bond for a loss, in order to reduce capital gains, while investing the proceeds of the sale in a bond of similar quality and maturity is considered

The answer is a tax swap. A tax swap results in a tax savings generated by the realized loss.

Select the CORRECT statement regarding security market indexes and averages.

The answer is all of these statements are correct. Averages and indexes are constructed to inform investors about changes in the market. They also serve as benchmarks for the performance of investors' portfolios and the performance of money managers. LO 7.3.1

Which of the following is a risk associated with corporate bonds?

The answer is all of these. All bonds are subject to systematic risks. However, government bonds are not subject to default risk.

Which of these statements describing the primary market is correct?

The answer is all of these. In the primary market, underwriters assist with the sale of the issue, evaluate the firm's financial needs, and determine the best investment vehicle to achieve the capital goal.

Which of the following is a disadvantage of investing in convertible bonds?

The answer is all of these. These bonds are doubly cursed during times of high interest rates and low stock prices because convertibles typically have lower coupon rates than comparable nonconvertibles. Thus, they experience greater price volatility.

Which of the following would cause the risk premium an investor expects to earn on a stock to increase when using the capital asset pricing model (CAPM)?

The answer is an increase in beta. All other factors remaining equal, an increase in beta will cause the risk premium to increase. In addition, the resultant expected rate of return will increase.

When analyzing various investment alternatives, investors would generally choose which of these?

The answer is an investment exhibiting a high positive skewness and a leptokurtic distribution. Investments exhibiting high positive skewness have a larger than average number of positive price movements. Also, investments exhibiting a leptokurtic distribution have more observations clustered closely around the mean, resulting in a lower variance. Investors prefer a large number of positive returns with low risk.

Case Study Question Larry's 401(k) plan has experienced returns over the past five years of -10%, 15.55%, -5.87%, 12.75%, and 14.85%. What are both the arithmetic and geometric means for this series of returns?

The answer is arithmetic mean = 5.45%; geometric mean = 4.85%. Arithmetic mean: [(-0.10 + 0.155 - 0.0587 + 0.1275 + 0.1485) ÷ 5] = 0.0545, or 5.45%. Geometric mean: PV = -1; FV = (1 - 0.10)(1 + 0.155)(1 - 0.0587)(1 + 0.1275)(1 + 0.1485) = 1.2671; N = 5; solve for I/YR = 4.8485, or 4.85%.

Brandon owns ABC mutual fund that has produced the following returns over the past three years: Year 1: 4.7% Year 2: −10.0% Year 3: 6.5% Based on this information, calculate both the arithmetic mean (AM) and geometric mean (GM) returns for this series.

The answer is arithmetic mean: 0.40%; geometric mean: 0.1182%. The arithmetic mean is calculated as follows: (4.7% - 10.0% + 6.5%) ÷ 3. The geometric mean is calculated as follows: PV = -1, FV = (1.047)(0.90)(1.065), PMT = 0, N = 3, solve for I/YR = 0.1182.

Which of the following statements correctly identify the factor relationships of the Black-Scholes option valuation model?

The answer is as the amount of time until expiration increases, the price of the call option increases. As the volatility of the underlying stock increases, the price of the call option will increase. The price of a call option will increase when the risk-free rate increases. If the market price of the underlying stock increases, the price of the call option increases.

Which of these statements regarding unit investment trusts (UITs) is CORRECT?

The answer is at the maturity date of the portfolio, the securities are generally liquidated, and the proceeds are distributed to the investors or trust beneficiaries. UITs are considered unmanaged or passively managed because the initial securities (usually bonds) included in the portfolio are typically held until maturity. UIT units are sold at NAV plus a commission. Units are sold in the secondary market but not on the major exchanges. LO 3.3.1

jack has $50,000 to invest in a portfolio of bonds. He decides to invest $25,000 into bonds with a two-year maturity and $25,000 into bonds with a 10-year maturity. His portfolio illustrates what type of bond strategy?

The answer is bond barbells. Bond barbells is a strategy for investing in both short-term and long-term bond issues

Select the INCORRECT statement regarding a company's book value. A) Book value per share may be derived by dividing the stockholder's equity portion by the total number of common shares outstanding. B) Book value is determined by subtracting company liabilities from company assets. C) Different inventory accounting methods may yield a different value for company assets, thereby affecting the book value of the corporation. D) Book value represents an accurate measure of the fair market value of the company.

The answer is book value represents an accurate measure of the fair market value of the company. Generally, book value does not represent an accurate measure of the fair market value of the company because this value is determined using historical costs.

Which of the following statements regarding asset allocation is CORRECT? Asset allocation is the main determinant of a portfolio's total return. The purpose of strategic asset allocation is to determine an appropriate allocation based on the long-term financial goals of the client.

The answer is both I and II. Both of these statements are correct.

Which of the following statements regarding wash sales is CORRECT? A wash sale occurs if the taxpayer sells or exchanges stock or securities for a loss and, within 30 days before or after the date of the sale or exchange, acquires similar securities. The wash sale rules are easily avoided in the case of fixed-income securities by substituting a bond with the same or similar characteristics as long as it is issued by a different company.

The answer is both I and II. Both of these statements describe characteristics of wash sales.

Rhett recently purchased a bond with attached warrants that afford him the opportunity to participate in the appreciation of the underlying stock. Which of the following statements correctly describes warrants? Warrants are customized to fit the needs of the issuing corporation. Warrants typically have a maturity date of several years.

The answer is both I and II. If corporations issue warrants, they usually do so in conjunction with new bond issues or preferred stock issues. These warrants give the bond or stock purchaser a sweetener or equity kicker, making the issue more attractive to buyers.

Which of these statements regarding a real estate mortgage investment conduit (REMIC) is CORRECT? A REMIC is a self-liquidating, flow-through entity that invests exclusively in real estate mortgages or mortgage-backed securities. A REMIC terminates when the mortgages that constitute the investment of the REMIC are repaid.

The answer is both I and II. REMICs are often structured to offer classes of bonds that mature over a period of 3-30 years. Accordingly, investors may invest in bond classes that match their investment time horizon and avoid the uncertainty of bond principal repayment.

Identify which of these statements regarding rights and warrants is CORRECT. Rights provide current common stockholders with the ability to retain their ownership percentage when new shares of stock are issued. Warrants are typically attached to new bond issues to attract investors.

The answer is both I and II. Rights provide current stockholders with the ability to maintain their percentage ownership interest in the corporation when new stock is issued. Warrants give the bond purchasers a sweetener, making the issue more attractive to buyers.

Identify which of the following statements regarding the risk premiums associated with the capital asset pricing model (CAPM) are CORRECT. The stock risk premium is the inducement necessary to entice the individual to invest in a given stock. The market risk premium is the incentive required for the individual to invest in the securities market.

The answer is both I and II. Statements I and II are both correct. The stock risk premium is the inducement necessary to entice the individual to invest in a particular stock, whereas the market risk premium is the incentive required for the individual to invest in the securities market in general.

Mike expects a certain stock to significantly rise in value in the near future. He is expecting a bond to mature in two months and does not want to miss out on any appreciation on the stock while waiting for the funds to become available. Which of these option strategies should be recommended to Mike?

The answer is buy a call option. Mike can lock in the price of the stock by purchasing a call option with an expiration date exceeding two months.

Case Study Question The Pratts are concerned that ABC stock may substantially decline in price in the near future. Which of the following option strategies would best protect the Pratts' position in ABC stock? (Assume they own 100 shares currently valued at $130 per share, and all options will expire within 60 days.)

The answer is buy one put option with an exercise price of $120 for $3.00 per share. If the Pratts are bearish, they will either sell a call or buy a put. In this particular case, the Pratts should purchase one put option with an exercise price of $120 for $3.00 per share. The total cost of the put option, excluding transaction costs, is $300 ($3 × 100). Each put option contract gives the holder the right to sell 100 shares of the underlying stock at the exercise price within a specified time frame.

Which of these statements concerning portfolio diversification is CORRECT?

The answer is by increasing the number of securities in a portfolio, the total risk would be expected to fall at a decreasing rate. As more and more securities are added to a portfolio, diversification benefits begin to diminish. The main attraction of diversification is the reduction of risk without an accompanying loss of return. LO 6.1.1

A financial reporter notices that the quoted price of one investment company's shares is at a 22% discount from the NAV. From this information, it can be deduced that the company is likely which of the following?

The answer is closed-end investment company. If the selling price of an investment company is less than the NAV, the fund is likely a closed-end investment company. LO 3.3.1

All of these statements concerning the use of the correlation coefficient in reducing portfolio risk are CORRECT except

The answer is combining two securities with perfect negative correlation provides no portfolio risk reduction. Combining two securities with perfect negative correlation could eliminate risk altogether. This is the principle behind hedging strategies.

ABC Corporation is a manufacturer of electronic devices used in the manufacturing of airplanes. Five years ago, the corporation floated a $100 million bond issue that would be used to finance improvements at its main manufacturing and distribution center. However, orders for its products have dropped dramatically due to much lower than anticipated demand. The company believes it may miss paying the coupon payment on the bond issue in the upcoming fiscal year. Identify which of these risks the owners of ABC Corporation bonds may be subject to by holding the bonds.

The answer is default risk. Default risk is the risk that a business will be unable to service its debt obligations.

ll of the following statements describing diversification are correct except

The answer is diversification is not enhanced by the addition of foreign securities to a portfolio because they are always highly correlated with domestic equity. Diversification is enhanced by the addition of foreign securities to a portfolio because they are usually not highly correlated with domestic equity. Each of these asset classes, as a whole, responds differently to different types of risk; therefore, diversifying or allocating investment resources among these classes is a proven way to reduce risk overall, dampen volatility, and improve the performance of one's portfolio.

The duration of a zero-coupon bond is

The answer is equal to the bond's maturity. The duration of a zero-coupon bond is equal to the bond's maturity.

Which of the following statements about the importance of risk and return in the investment objective is least accurate?

The answer is expressing investment goals in terms of risk is more appropriate than expressing goals in terms of return. Expressing investment goals in terms of risk is not more appropriate than expressing goals in terms of return. The investment objectives should be stated in terms of both risk and return. Risk tolerance will likely help determine what level of expected return is feasible.

Which of the following statements regarding hedge funds is CORRECT?

The answer is hedge funds are usually structured as a partnership. The partnership is with the general partner as the investment manager and the investors as limited partners. Hedge funds are actively and aggressively managed, seek superior returns, and are best suited for wealthy, sophisticated investors. Hedge fund managers are largely compensated for performance, not assets under management.

Which of the following statements regarding common stock is CORRECT?

The answer is if Jacob sells his XYZ stock to Darlene the day before the record date, Jacob will receive the dividend for that period. The person who is the owner of the stock on the second business day before the record date is issued the dividend for that period. Jacob sold the stock to Darlene after the ex-dividend date; therefore, he is still the owner of record because the trade will not settle for two business days following the sale. Darlene purchased the stock after the ex-dividend date. Stock owned in street name is held by the brokerage firm; the corporation does not issue a certificate to the investor. Growth stocks typically do not pay dividends. The value of the stock is expected to grow—or appreciate—with the reinvestment of earnings. Defensive stocks typically perform better than cyclical stocks in a recessionary economy.

An individual who is a proponent of the efficient market hypothesis (EMH) will likely invest in which of the following?

The answer is index funds. An individual who believes in the EMH will likely invest in index funds. Inherent in this strategy is a belief that an investor cannot outperform the market with active portfolio management techniques. The remaining choices all incorporate an active portfolio management philosophy.

Identify the CORRECT statement concerning international investing.

The answer is information is not as readily available on foreign investments. Foreign markets offer economies of scale and growth opportunities. Investors may earn higher returns in foreign markets, particularly if they are less efficient than U.S. markets. Including foreign securities in an investment portfolio may lower risk through greater diversification.

Which of the following is a characteristic of American depositary receipts (ADRs)?

The answer is information regarding the foreign company is often more easily attainable with ADRs because the entity holding the security generally has access to that information. ADRs are trust receipts issued by a U.S. bank for shares of a foreign company purchased and held by a foreign branch of the bank. They are an alternative to investing directly in foreign companies or foreign mutual funds. Dividends are declared in the local currency, so exchange-rate risk is not completely eliminated.

Which of the following is NOT an advantage of the buy-and-hold strategy?

The answer is investors or managers time their security purchases while staying fully or predominantly invested in the market, thereby maximizing profit potential. Market timing is an attempt to predict the overall direction of the securities market and take advantage of changes in the prices of those securities, whether those prices go up or down. Studies have shown that, by and large, market timing techniques do not work over the long term.

All of the following illustrate a characteristic of a Monte Carlo simulation except

The answer is large changes in the projected rate of return will make small differences in the outcome. Small changes in the projected rate of return will make large differences in the outcome.

Tangible assets might be suitable as an investment in the portfolio of an investor looking for

The answer is long-term capital gains. Tangible assets tend to have high markups and take a relatively long time to recover the costs. They tend to do well during periods of inflation.

Which of the following combinations will result in a bond with the greatest price volatility?

The answer is low coupon and long maturity. Price volatility is measured by duration. Duration is inversely related to the bond's coupon rate and directly related to the bond's term to maturity.

Which one of these is NOT a typical key element that separates hedge funds from mutual funds?

The answer is many hedge funds are broadly diversified. Hedge funds are private investment vehicles that tend to be more heavily concentrated than mutual funds. They often use leverage, derivatives, employ narrow investment strategies, and invest in nonpublic and illiquid securities.

Which of the following correctly explains a disadvantage of investing in money market instruments?

The answer is money market mutual funds pay a low rate of interest. An investor in CDs is willing to accept a lower return to maintain a high degree of safety. Certificates of deposit are subject to penalties for withdrawals made prior to maturity. However, by accepting a low rate of return, the investor is subject to purchasing power (inflation) risk. Money market instruments are subject to purchasing power (inflation) risk. LO 1.3.1

Choose the REITs that are used to finance real estate ventures that develop property or finance construction.

The answer is mortgage REITs. Mortgage REITs are in the business of financing real estate ventures. They make loans to develop property or finance construction. LO 5.1.1

Which of the following statements concerning preferred stock is correct?

The answer is preferred stock dividends are not legally binding but must be voted on each period by the corporation's board of directors. Preferred stockholders are paid after bondholders in terms of priority of payment of income and in case of corporate liquidation. If the issuer of a cumulative preferred stock fails to pay the dividend in any year, the unpaid dividend(s) will have to be paid in the future before common stock dividends can be paid. LO 2.5.1

Which of the following is NOT likely to be an advantage of a valid investment policy statement?

The answer is provides for short-term strategy shifts in response to short-term dramatic value declines. The investment policy statement does not provide for shifts in strategy due to short-term value declines. LO 8.4.1

Xavier is concerned that his investment portfolio is too concentrated in only a few stocks. He meets with his financial advisor for advice. The advisor recommends that Xavier diversify his portfolio among many different types of issues. Which of the following is the primary goal of this strategy?

The answer is reduce portfolio risk. Diversification aims to reduce unsystematic risk, not necessarily increase portfolio returns.

Which of the following actions is consistent with a belief in the efficient market hypothesis (EMH)?

The answer is selecting a random set of stocks for a portfolio. The fundamental assumption of the EMH is that current stock prices reflect all available information and that prices rapidly (or immediately) adjust to reflect any new information. In addition, any new information must be unexpected; therefore, any changes in stock prices resulting from this new information will be random (i.e., the random walk theory). LO 8.3.2

Assume that the yield curve currently is shaped as shown in YC1, and you anticipate it will be shaped as shown in YC2 one year from now. Assuming you want to maximize the opportunity for capital appreciation, which of the following investment strategies would you recommend for clients, based on the current and anticipated shapes of the yield curves?

The answer is sell short-term bonds and buy long-term bonds. When a yield curve is inverted, a financial planner should consider purchasing long-term securities to lock in rates for a long period of time—in anticipation of lower yields in the future. Such a move might also result in a capital gain on the investment as bond prices rise.

Identify the efficient market hypothesis that suggests an investor can achieve above-market returns by only utilizing insider information.

The answer is semistrong. The semistrong form suggests that fundamental analysis is of no value and only through the use of insider information can an investor achieve above-market returns.

An investor who reallocates her portfolio frequently to take advantage of perceived opportunities in other market sectors is using which one of the following types of asset allocation?

The answer is tactical. Reallocating a portfolio frequently to take advantage of perceived under- or overvaluations in a particular market is tactical asset allocation, which is akin to market timing. Strategic asset allocation involves determining the best risk/return portfolio for an investor and then rebalancing to that optimum mix when percentages change due to market movements. The dynamic strategy is used by institutions, and involves increasing risky assets as the portfolio value rises, and decreasing risky assets as the portfolio value declines.

All of the following affect an investor's risk tolerance except

The answer is tax bracket. Tax concerns play an important role in investment planning; however, these constitute an investment constraint, not an investment objective (i.e., risk tolerance).

All of the following correctly explain advantages of investing in real estate investment trusts (REITs) except

The answer is tax deferral. Tax deferral is not an advantage of investing in REITs. REITs are professionally managed assets investing in a diversified portfolio of real estate holdings. Many REITs trade on the exchanges and over the counter, thereby providing investors with liquidity.

Which of the following is the primary difference between the underlying assumptions of technical analysis and those of fundamental analysis?

The answer is technical analysts believe the trend in security prices is determined solely by the interaction of economic supply and demand. Stock prices move in trends, and therefore, market price adjustments occur gradually over time (e.g., company financial statements play a much less important role).

A beta coefficient of 1.3 indicates that a stock

The answer is that the stock is more volatile than the market. A beta that is higher than 1.0 indicates that the stock's volatility and risk are higher than that of the market.

According to the arbitrage pricing theory (APT), the return on a stock represents which of the following?

The answer is the APT depends on the stock's responsiveness to unexpected changes. The APT is related to the expected return on the stock. The APT is not reduced by the construction of diversified portfolios. The APT does not take into consideration the market return if the expected rate of inflation is realized.

The Allegro Mutual Fund has an alpha of +1.50, a Sharpe ratio of 0.44, and an R2 with the Russell 2000 of 0.45. The Moderato Mutual Fund has an alpha of +0.40, a Sharpe ratio of 0.48, and an R2 with the Russell 2000 of 0.52. Which fund should be chosen and why?

The answer is the Moderato Fund because has a higher Sharpe ratio. In order to use alpha (which uses beta in the formula), beta needs to be a reliable number. This can be determined by R2, which gives the level of systematic risk. The R2 for both funds is low (0.45 and 0.52), meaning beta, and formulas using beta, should not be used. You want an R2 of 0.70 or higher in order to use beta. This then leaves you with Sharpe, and the Moderato Fund has the highest Sharpe ratio of the two.

Which of the following statements regarding security market indexes is CORRECT?

The answer is the S&P 500 Index automatically adjusts for stock splits and dividends by focusing on market value instead of price. The Russell 2000 Index is a well-known index used to benchmark small capitalization companies. A market average, not a market index, reflects the average price behavior of a group of stocks at a given point in time. An index measures the current price behavior of a group of stocks in relation to a base value. The Wilshire 5000 index is used as a measure of the U.S. broad market.

f an investment has a correlation coefficient of 0.80 with the market, which of the following performance measures is the best measure of risk?

The answer is the Sharpe ratio. Because the correlation coefficient is 0.80, the coefficient of determination (R squared) is 0.64. Therefore, only 64% of the returns from the investment can be explained by the market (i.e., systematic risk represents 64%). Beta only measures systematic risk, which means that 36% of outcomes will not be captured by beta. Thus, Treynor and Jensen are not appropriate because they use beta. Sharpe is always an appropriate performance measure because the ratio is calculated using standard deviation.

You are about to choose a new mutual fund to add to client portfolios. As you review the Morningstar reports for the funds you are considering, you have focused on each fund's alpha as reported by Morningstar. Alpha tells you

The answer is the difference between a fund's realized return and its risk-adjusted expected return. Alpha does not compare directly to the S&P 500, but rather to the fund's expected return, which is risk-adjusted for the fund's beta. The total return, not just the capital appreciation component, is used in the Jensen formula. The risk-adjusted required return is the risk-free rate plus the risk premium multiplied by the fund's beta.

Which of the following statements regarding market efficiency is correct?

The answer is the efficient market hypothesis (EMH) is the proposition that the securities markets are efficient, with the prices of securities reflecting their current economic value. Investors who accept the EMH usually adopt a passive investment strategy. Investors who do not accept the EMH pursue an active investment strategy. The fundamental assumption of the theory is that current stock prices reflect all available information for a company, and prices rapidly adjust to reflect any new information. Any new information must be unexpected; therefore, any changes in the stock price resulting from this new information will be random.

Which of the following statements concerning technical analysis as an approach to selecting securities is correct?

The answer is the emphasis of technical analysis is on internal factors that help to detect supply and demand conditions in the market.Technical analysis can be applied to the aggregate market as well as individual stocks. The rationale for technical analysis is that stock prices require time to adjust to changes in supply and demand. Company financial statements are used by fundamental analysts.

When using a security market index to represent a market's performance, the performance of that market over time is best represented by

The answer is the percent change in the index value. Percentage changes in the value of a security market index over time represent the performance of the market, segment, or asset class from which the securities are chosen. LO 7.3.1

If a security has an average return of 14.2% and a standard deviation of 8.4%, then

The answer is the security's returns can be expected to be between 5.8% and 22.6% approximately 68% of the time. This security can be expected to have a return that does not range beyond one standard deviation on either side of its average return approximately 68% of the time.

What is one disadvantage of investing in convertible bonds?

The answer is the yield to maturity tends to be lower than that of similar nonconvertible bonds. A disadvantage of investing in a convertible bond is that its yield to maturity tends to be lower than a similar nonconvertible bond due to the conversion feature.

Trey is considering the purchase of American depositary receipts (ADRs). He is looking to further diversify his portfolio. Identify which of these is NOT a feature of this type of investment vehicle.

The answer is they are not subject to exchange rate, or currency, risk. Even though ADRs are denominated in U.S. dollars, they are subject to exchange rate, or currency, risk.

To evaluate the performance of a portfolio manager, you should calculate the portfolio's

The answer is time-weighted return. Because portfolio managers have no control over the deposits and withdrawals made by clients, the time-weighted return is a more appropriate measure of performance.

To measure the performance of an investment manager, which of the following methods of computing returns should be used?

The answer is time-weighted return. The time-weighted return should be used to measure the performance of an investment manager.

All of the following are primary factors in the arbitrage pricing theory (APT) except

The answer is unemployment rate. These factors might include inflation, growth in GDP, major political upheavals, or changes in interest rates.

Diversification reduces

The answer is unsystematic risk. Unsystematic risk can be diversified away by investing in approximately 10-15 large company stocks in different industries and 25-30 small company stocks in different industries. Systematic risk cannot be reduced by diversification.

The issuer-specific component of the variability in a stock's total return that is unrelated to overall market variability is known as

The answer is unsystematic risk. Unsystematic risk is unique to a single security, business, industry, or country and may be reduced by diversification.

Mimi, a client of Osborne Capital, Inc., believes her portfolio should be adjusted. She supports her claim by stating that she just won the lottery and wants to retire 10 years earlier than originally planned. Does she have a valid claim?

The answer is yes, her wealth and time horizon have changed. Changes in wealth, time horizon, and liquidity requirements all dictate the need to rebalance. Taxes, laws, regulations, and unique circumstances also play into this decision. LO 8.5.2

CDE Inc. bonds have the following characteristics: 10% coupon $1,000 par value Current price of $1,136.92 Eight years to maturity Callable in five years at $1,100 Calculate the bond's yield to maturity (YTM) and yield to call (YTC).

YTM: 7.68%; YTC: 8.26% The answer is YTM: 7.68%; YTC: 8.26% Yield to maturity is calculated as: END Mode, 2 P/YR PMT = 50 FV = 1,000 PV = -1,136.92 8, DOWNSHIFT, N = 16 I/YR (YTM) = 7.68, or 7.68% Yield to call is calculated as: PMT = 50 FV = 1,100 PV = -1,136.92 5, DOWNSHIFT, N =10 I/YR (YTC) = 8.26, or 8.26%

Wayne owns shares in the Big Value mutual fund. Last year, Big Value had a return of 18%, the S&P 500 Index had a return of 21%, and six-month Treasury bills averaged a return of 5%. Big Value's standard deviation was 14, the standard deviation of the S&P 500 index was 12, and Big Value's beta was 0.9. Which of the following is the Treynor ratio for Big Value last year?

he answer is +14.44. The Treynor ratio for Big Value: (18 - 5) ÷ 0.9 = 14.44

An analysis of the monthly returns for the past year of a mutual fund portfolio consisting of two funds revealed the following statistics: Fund AFund BTotal return18%11%Standard deviation24%17%Percentage of portfolio35%65%Correlation coefficient (R).27 What is the standard deviation of the portfolio?

he answer is 15.58%. This requires using the standard deviation of a portfolio formula: σP=√W2iσ2i+W2jσ2j+2WiWj[COVij]σP=Wi2σi2+Wj2σj2+2WiWj[COVij] Break this formula down into three sections: Fund A: (0.35)2(24)2 = (0.1225)(576) = 70.56 Fund B: (0.65)2(17)2 = (0.4225)(289) = 122.1025 Covariance: 2(0.35)(0.65)(24)(17)(0.27) = 50.1228 σp = (70.56 + 122.1025 + 50.1228)0.5 σp = (242.7853)0.5 = 15.5816, or 15.58% LO 6.2.4

Lauren invested $15,000 in a growth and income fund four years ago. She received a dividend of $800 the first year and $900 each in the second, third, and fourth years. Today, her investment has a total value of $27,234.56. Calculate the approximate internal rate of return (IRR) on Lauren's investment. (Round to the nearest percent.)

he answer is 21%. IRR is the discount rate that equates the present value of all the cash inflows with the present value of the cash outflows. Keystrokes for the HP 10bII+: 15,000 +/- CFj, 800 CFj, 900 CFj, 900 CFj, 28,134.56 CFj, gold IRR/YR = 20.80 (rounded to 21%). Note the final cash flow consists of both the dividend ($900) and the ending value ($27,234.56).

Myles purchased 1,000 shares of XYZ growth fund for $15 per share. At the end of the two years, he sold all of the shares for $22 per share. At the end of each year, the fund paid a dividend of $0.50 per share. Calculate the fund's time-weighted return over the two-year period.

he answer is 24.15%. The fund produced a 24.15% time-weighted rate of return over the two-year period, calculated as follows: CF0 = -15 × 1,000 = −15,000 CF1 = 0.50 × 1,000 = 500 CF2 = (0.50 × 1,000) + (22 × 1,000) = 22,500 Solve for the internal rate of return (IRR/YR) = 24.1525% (rounded to 24.15%) LO 7.1.1

Which of the following would be an appropriate index to track an investment in an international developed markets mutual fund?

he answer is MSCI EAFE. The MSCI EAFE Index tracks markets in Europe, Australasia, and the Far East (primarily Japan).

When an analyst divides the P/E ratio by the earnings growth rate, which one of these ratios is being used?

he answer is PEG ratio. This identifies the P/E to Growth (PEG) ratio. P/B is price to book; P/E is price to earnings; P/S is price to sales; DDM is dividend discount model.

The semistrong form of the efficient market hypothesis states that current market prices reflect all available information on the history of prices. all publicly available information concerning a company.

he answer is both I and II. The semistrong form states that all publicly available information, including past stock price history, is reflected in current stock prices.

You currently own 100 shares of a stock that has increased in value from $15 to $35 per share. You do not want to sell the stock but want to protect yourself against a large downturn in the market. Which of the following is an effective action on your part?

he answer is buy a $30 put option on the stock. The purchase of a put option on the stock will provide downside protection without limiting the investor's upside.

Which of the following are attributes of an economy that is coming out of recession?

he answer is cyclical stocks will begin to move up in price. Once the economy starts to recover, the unemployment rate will decline, interest rates will fall, and personal income will grow. Then cyclical stocks will begin to move up in price.

Asante stipulated that certain assumptions must be present for the capital asset pricing model (CAPM) to be used. Which of the following statements is NOT one of these assumptions?

he answer is investment expenses, such as taxes and transaction costs, are relevant in investment decision making. The CAPM does not consider taxes or transaction costs. LO 8.2.1

Identify which of these is NOT an unsystematic risk.

he answer is market risk. Unsystematic risk is the risk that affects only one company, country, or sector and its securities. Market risk is an example of a systematic risk.

Linda and Ralph Stewart have never invested in the stock market, but they would like to begin an investment program to cover college expenses for their two young children. The Stewarts' required rate of return is 11%. They are considering the purchase of one of these two stocks: Stock 1:Dividends are currently $1.85 annually and are expected to increase 9% annually; market price = $59Stock 2:Dividends are currently $1.58 annually and are expected to increase 6% annually; market price = $37 Which stock would be most appropriate for the Stewarts to purchase at this time, and why?

he answer is stock 1, because the return on investment is greater than the Stewarts' required rate of return. The intrinsic value of Stock 1 = $100.83 [($1.85 × 1.09) ÷ (0.11 - 0.09)]. Because $100.83 is more than $59, the stock is undervalued and would return more than their required return. The intrinsic value of Stock 2 = $33.50 [($1.58 × 1.06) ÷ (0.11 - 0.06)]. Because $33.50 is less than $37, the stock is overvalued and would return less than their required return.

Capitalization-weighted indexes are

the preferred type of index to use in modern portfolio theory applications. The answer is the preferred type of index to use in modern portfolio theory applications. Capitalization weighted indexes are the most prevalent type of index and are best suited for modern portfolio theory applications. In a price-weighted index, higher-priced stocks within this index have more influence on the overall movement of this index than lower-priced stocks.

To be eligible for preferential dividend tax rates

the stock must be held for more than 60 days during the 121-day period beginning 60 days BEFORE the ex-dividend date.

Which of these statements regarding basic option positions is CORRECT?

If an investor sells a put, the maximum loss is the exercise price less the amount of premium received The answer is if an investor sells a put, the maximum loss is the exercise price less the amount of premium received. If an investor buys a put, the maximum gain is the exercise price less the premium paid. If an investor sells a naked call, the maximum loss is unlimited. If an investor buys a call, the maximum loss is the premium paid..

An active approach to portfolio management is more likely to reward investors in which of the following asset classes? Emerging market equities U.S. large-cap stocks U.S. small-cap stocks European large-cap stocks

B) I and III investors who follow an active approach to portfolio management will benefit most from a portfolio containing emerging market equities and U.S. small-cap stocks, which tend to be more volatile investments.

During the past year, the stock market had a return of 8%, while the risk-free rate of return was 3%. Fund B had a realized return of 12%, a standard deviation of 15, and a beta of 1.20. Jensen's alpha for the fund is

The answer is 3.00%. Alpha = 12% - [3% + (8% - 3%)1.20] = 12% - 9% = 3%

Al Jenkins owns a corporate bond that currently sells for $1,175. The coupon rate is 9%, interest is paid semiannually, and the bond matures in 20 years. The bond is callable in 11 years at $1,050. What is the yield to call on this bond?

The answer is 7.00%. The yield to call is calculated as follows: END Mode, 2 P/YR 11, DOWNSHIFT, N = 22 PV = -1175 FV= 1000 PMT = 9% x 1000 / 2 = 45 Solve for I/YR = 7.00% LO 7.4.1

To be on a corporation's books as holder-of-record (and thus have a right to the next dividend payment), the investor must purchase stock

The answer is before the ex-dividend date. Ex-dividend means "without dividend." Therefore, all purchases made on or after the ex-dividend date are not entitled to the next dividend to be paid. The ex-dividend date is one trading day before the record date.

Grant calls his broker and tells her to sell his XYZ stock if it falls to $20, but he does not want less than $19.75 for his shares. Select the type of order that his broker should place to sell the stock.

The answer is stop limit order. The stop limit order turns into a limit order when triggered (both the stop order price and the limit order price are specified). However, this type of order will not guarantee execution if the stock leapfrogs past the $19.75 mark.

All of the following statements correctly assess the determination of cost basis when a taxpayer sells publicly traded securities such as stocks or bonds except

if the taxpayer cannot specifically identify the shares sold, basis will be calculated using the average cost of all shares or units owned.

Distributions of dividend and capital gains in cash to mutual fund investors

he answer is that distributions of dividend and capital gains in cash to mutual fund investors are fully taxable to the investor. If the dividends and capital gains are reinvested, the individual receives an increased tax basis. If the distributions are made in cash, there is no increase in the tax basis of the underlying securities.

When investment bankers absorb the loss on an initial public offering, which one of the following terms represents this type of offering?

Firm commitment

If a $100 par value preferred stock pays an annual dividend of $5 and comparable yields are 10%, the price of the preferred stock will be

$50 $5 ÷ 0.10 = $50 for preferred stocks, the zero-growth model is used because the preferred stock's dividend is fixed. The formula is V = D0/r, where D0 is the dividend and r is the required return.

You own a small-cap fund and are trying to compare its performance to an appropriate benchmark. Which of the following benchmarks would be the best to use?

Russell 2000 Explanation The Russell 2000 is generally considered the benchmark for small-cap funds.

Gary Stevens would like to know the weighted beta for his portfolio. He owns 100 shares of ACE common stock with a beta of 1.1 and total current market value of $5,000; 400 shares of BDF common stock with a beta of 0.70 and total current market value of $8,000; and 200 shares of GIK common stock with a beta of 1.5 and total current market value of $10,000. What is the overall weighted beta coefficient for Gary's portfolio?

The answer is 1.13, calculated as follows: 1.1INPUT (ENTER)5Σ+0.7INPUT (ENTER)8Σ+1.5INPUT (ENTER)10Σ+SHIFT, xw,b = 1.13

The yield to maturity on a zero-coupon bond ($1,000 par value) currently selling at $677 and maturing in four years is approximately

The answer is 10.00%. Solve for yield to maturity: END Mode, 2 P/YR PV = −677 FV = 1,000 PMT = 0 4, DOWNSHIFT, N = 8 Solve for I/YR = 10% (rounded). LO 7.4.1

The annual returns of the ABC fund have been +12%. -4%, and +7%. What is the standard deviation of the fund's returns?

The answer is 8.19%. HP 10bII+ keystrokes: 12, ∑+ 4, +/-, ∑+ 7, ∑+ SHIFT, Sx,Sy (8 key) for standard deviation LO 6.2.2

Angela purchased a corporate bond currently selling for $925 in the secondary market. The bond has a coupon rate of 7.75% and matures in 12 years. Which of the following is the yield to maturity on this bond?

The answer is 8.77%. The yield to maturity on this bond is as follows: END Mode, 2 P/YR PV = -925 PMT = 7.75% × 1,000 ÷ 2 = 38.75 FV = 1000 12, DOWNSHIFT, N = 24 Solve for I/YR = 8.77% LO 7.4.1

Two mutual funds have these performance statistics: Fund EFund FThree-year total return16.5%17.2%Standard deviation18.116.4R-squared81%87%Sharpe ratio.58.68Alpha1.11.6 Which one of the two funds has the better risk-adjusted performance, and why?

The answer is Fund F, because its alpha is higher. In this case, the investor should choose the fund with the higher alpha. With an alpha of 1.6, Fund F exhibits the best risk-adjusted performance.

Advantages of unit investment trusts include which of these? Stable periodic income Diversification Active management of the portfolio

The answer is I and II. Because the portfolio is fixed, the income stream is predictable. The number of different bonds in the portfolio provides investors with a diversified portfolio.

oose the CORRECT statements regarding option contracts. The buyer of a call option has the potential for unlimited gain. The seller of a put option is bullish. The writer of a naked call is exposed to an unlimited loss. If the writer of a call option owns the underlying stock, the option is considered covered.

The answer is I, II, III, and IV. All of these statements are correct regarding option contracts.

the basic purposes of the investment policy statement include all of the following except

The answer is determining the specific assets to include. This is not one of the basic purposes of the investment policy statement.

In the Markowitz framework, an investor should most appropriately evaluate a potential investment based on its

The answer is effect on portfolio risk and return. Modern portfolio theory concludes that an investor should evaluate potential investments from a portfolio perspective and consider how the investment will affect the risk and return characteristics of an investor's entire portfolio.

The risk associated with the amount of debt a company has issued is

The answer is financial risk. Financial risk is the risk related to the amount of debt a company has. Business risk is the risk associated with the nature of the business. Interest rate risk is the risk that as interest rates increase, bond prices decrease. Systematic risk is the risk associated with all factors affecting all comparable investments.

When working with anxious clients who are concerned about widely fluctuating stock market prices, what is the most important service you can provide to them?

The answer is managing their expectations. Unrealistic expectations can be a function of a client's ignorance about financial markets and securities. By using your knowledge, you can educate clients about markets and securities so that expectations are realistic. Doing so also strengthens the client/planner relationship and allows you to demonstrate your expertise. LO 8.6.1

Which of the following is NOT considered a constraint when developing an investment policy statement?

The answer is market conditions. An investment policy statement is a written document that sets forth a client's objectives and establishes certain limitations on the investment manager. The following are constraints of the investment policy statement: time horizon, liquidity needs, taxes, laws and regulations, and unique circumstances/preferences. Market conditions are not a constraint when developing an investment policy statement.

Yadira owns 20 stocks across several sectors of the economy that are part of the S&P 500 index, typically known as "blue chip" stocks. Her investments are exposed to which of the following risks?

The answer is market risk. Market risk is the risk inherent in the overall securities marketplace and is sometimes simply referred to as volatility.

Which one of these is a general characteristic of hedge funds?

The answer is may sell short a variety of securities beyond the standard stocks and bonds. Hedge funds charge both a management fee and a carried interest fee. Hedge funds are characterized by a lack of marketability, significant use of leverage, and limited transparency.

Francis and William would like to place $5,000 into an account that would be used primarily for emergencies. Which of the following investment choices should be recommended for an emergency fund?

The answer is money market mutual funds. Money market mutual funds are used by many investors as their emergency fund because the funds are extremely liquid.

Which of the following stock market anomaly descriptions is CORRECT?

The answer is neglected firm effect: buy stocks followed by few analysts. Buying stocks in December and selling them in January is the January effect. Buying stocks with high book value to market value is BVMV. Small-company stocks outperforming large-company stocks over time is the size effect.

Choose the form of the efficient market hypothesis that supports technical analysis A) Strong B) Weak C) None of these D) Semistrong

The answer is none of these. The efficient market hypothesis is in direct contradiction to technical analysis because the efficient market hypothesis is founded on the notion that all historical price and volume data, which is used by technical analysts, is already accounted for in the current stock price.

Select the investment that gives the shareholder a short-term opportunity to buy new shares of the new stock issue, thereby maintaining the shareholder's respective overall percentage ownership in the corporation.

The answer is rights. Rights are a purchase option for stock that allows a shareholder the opportunity to buy shares of the new stock issue, thereby maintaining the overall percentage ownership in the corporation.

What is the coefficient of variation for an investment with a standard deviation of 8.65%, an expected return of 11.5%, and a beta of 1.25?

The answer is 0.7522. CV = standard deviation of asset ÷ expected return of asset, 8.65% ÷ 11.5% = 0.7522.

An investor selects an appropriate portfolio by choosing the portfolio

at the point of tangency between the indifference curve and the efficient frontier. The answer is at the point of tangency between the indifference curve and the efficient frontier. The investor will choose a portfolio represented by the highest point attainable on the indifference curve

Patrice Patterson began investing last year in the Apex Fund. She is investing $500 every quarter and wants to know what her average cost per share (basis) has been. These are the prices of the Apex fund at the end of each quarter when she made her purchases: $35.50, $38.90, $65.70, $72.50, and $89.00. What is her average cost per share?

$2,500 ÷ 47.063 shares = $53.12

Which of the following factors is NOT necessary for calculating the measure of risk that is used in Markowitz's efficient frontier?

) Beta for each asset class ) Beta for each asset class

An investor analyzing a particular security is generally concerned with

) expected return. The answer is expected return. Expected return is what determines an asset's value. The expected return must be greater than the investor's required return to induce the investor to make the investment. Historical return is only important to the extent that it may impact future return.

Terri has been an active investor for many years, and her present portfolio consists of listed stocks, penny stocks, and options. She earns approximately $175,000 annually, has $35,000 in cash to invest, and is in the 32% marginal income tax bracket. Terri is interested in accumulating wealth for the future and does NOT need current income. Which one of these fixed-income securities best suits Terri's needs at this time?

A rated, discount, long-term, tax-free, municipal revenue bond Because Terri is in a high income tax bracket, an investment-grade municipal bond would be the best choice. The long-term nature of this bond may afford Terri a higher net interest payment than the other bonds. LO 2.3.2

Select which of these is NOT a primary risk associated with a coupon-paying bond.

A) Debenture risk

The use of financial leverage affects all of these except

A) monetary policy.

Select a benefit of investing in foreign markets.

Diversification

Which of these is a disadvantage of a REIT investment?

No flow-through of tax benefits

What is the highest long-term capital gains rate on collectibles?

The answer is 28%. The long-term capital gains tax rate on collectibles (tangible assets other than real estate) is capped at 28%. LO 1.4.1

Max bought 100 shares of PET Corporation stock 10 years ago. He paid $10 per share for the stock. The stock currently has a fair market value of $50 per share. Choose the CORRECT statement regarding Max's stock.

f Max does not sell the stock this year, he will not have to pay taxes on the difference between the amount he paid for the stock and the current market price this year.

Carol sells her AA rated 5% YTM bond for $940 and buys a BB-rated 6% bond for $900. Both bonds mature in four years. This transaction illustrates which of these swaps?

he answer is a pure yield pickup swap. In a pure yield pickup swap, a lower YTM bond is substituted for a higher YTM bond.

All of the following statements correctly illustrate bond relationships except

higher-rated bonds have more price volatility than lower rated bonds.

Long-term bond funds have

more interest rate risk than short-term bonds.

Financial leverage affects

return on equity, earnings per share, and risk to stockholders.

Tripp is an investor in the 32% marginal tax bracket. If he invests in a 4.75% municipal bond, his taxable equivalent yield (TEY) would be

taxable equivalent yield = tax-exempt yield ÷ (1 − marginal tax rate) = 4.75% ÷ (1 − 0.32) = 6.99%

In a financial market

that is efficient, the prices of securities will not differ from their justified economic values for any length of time.

All of the following statements concerning market efficiency are correct except

The answer is investors usually react slowly to new and random information pertaining to securities markets. Another condition that guarantees an efficient market is investors reacting quickly to new information.

Which of the following statements correctly explains separately managed accounts and their relation to mutual funds?

The answer is separately managed accounts are individual investment accounts offered by financial consultants who provide advisory services and are managed by independent money managers using an asset-based fee structure.

A mutual fund with an investment objective of growth and income has an alpha of +4, a beta of 1.5, and a Sharpe ratio of 1.15. The fund

The answer is should be purchased, because the rate of return is high in relation to risk. A positive alpha indicates the fund performed better than it should have on a risk-adjusted basis. Also, an alpha of +4, which is very high, means it performed 4% better than expected.

Select the CORRECT statement regarding investors who only purchase high-beta stocks.

The answer is they prefer stocks with high risk and high positive skewness. Generally, these types of investors would prefer stocks with high risk (high beta) and high positive skewness that provide the opportunity for high rates of return. Stocks exhibiting high positive skewness have a larger than average number of positive price movements. LO 6.2.2

Select the INCORRECT statement regarding futures contracts

The answer is to complete a futures contract, the delivery of the commodity is required. To complete the futures contract, delivery of the commodity may be made, but usually the holder will purchase an offsetting contract and cancel the original position.

You are considering buying a stock that has a mean return of 14% and a standard deviation of 20. You can expect the return to fall within what range 95% of the time?

-0.26 to 0.54 A stock with a standard deviation of 20 will deviate from the mean by one standard deviation 68% of the time, two standard deviations 95% of the time, and three standard deviations 99% of the time. So for this stock, plus or minus 40 from the mean of 14% would be -26% and +54%.

Assuming JHG and DSA stocks have standard deviations of 6.23% and 10.78%, respectively, and a correlation coefficient of 0.17, calculate the covariance between the two stocks.

The answer is 11.42. The covariance between the two stocks is a positive 11.42 (6.23 × 10.78 × 0.17). Covariance measures the extent to which two variables move together, either positively (together) or negatively (opposite).

Caralla Foods produced before-tax earnings of $15 million last year. A large institutional investor has determined an appropriate capitalization for valuing this company is 8%. In addition, the risk-free rate of return is 5%. Based on this information, calculate the value of Caralla Foods using the discounted earnings model.

The answer is $187,500,000. The formula for the discounted earnings model: V = E ÷ RD = 15,000,000 ÷ 0.08 = 187,500,000 Using the discounted earnings method produces a current corporate valuation of $187,500,000. LO 4.3.1

John Hawkins began purchasing VNB stock two years ago. He has followed a dollar cost averaging approach by investing $1,500 every six months for the last two years. The following data depicts John's purchases:

The answer is $23.44. John purchased 256 shares (60 + 68 + 72 + 56) and invested $6,000 over this period. Divide the total dollars invested by the number of shares purchased to obtain the correct answer.

George has a five-year bond with a coupon rate of 3.65% (paid semiannually). Assuming the comparable yield for this quality bond is 4.85%, calculate the intrinsic value of his bond.

The answer is $947.28. The intrinsic value of George's bond equals $947.28, indicating the bond would be trading at a discount to par. END Mode, 2 P/YR FV = 1,000 5, DOWNSHIFT, N = 10 I/YR = 4.85 PMT = 3.65% × 1,000 ÷ 2 = 18.25 Solve for PV = -947.2835, or $947.28 LO 4.1.1

A bond has a current yield to maturity (YTM) of 3.80%. Market interest rates are expected to rise to 4.50% soon. The bond has a duration of 5.5 years. Calculate the estimated price change of the bond.

The answer is -3.71%. The market price of the bond is expected to decrease by 3.71%, calculated as follows: ΔP/P = −5.5[(0.045 − 0.038) ÷ (1 + 0.038)] ΔP/P = −5.5(0.007 ÷ 1.038) ΔP/P = −5.5(0.006745) ΔP/P = −0.0371, or −3.71% LO 4.1.2

Phil is considering adding a tax-free municipal bond to his extensive investment portfolio. He is in the 37% federal marginal income tax bracket and lives in a state that does not impose a state income tax. His broker has offered him the opportunity to purchase a AAA-rated general obligation bond. Assuming the bond has a coupon rate of 3.35%, what is its taxable equivalent yield?

The answer is 5.32%. This bond's taxable equivalent yield is calculated as follows: 3.35% ÷ (1 - 0.37).

Ashley purchased a five-year corporate bond with a 6.25% coupon paid semiannually. The bond is callable after three years for a price of $1,025. Assuming the bond is currently trading at $1,045, calculate its yield to call.

The answer is 5.38%. Yield to call: END Mode, 2 P/YR PV = -1,045 FV = 1,025 PMT = 31.25 (6.25% × 1,000 ÷ 2) 3, DOWNSHIFT, N = 6 Solve for I/YR = 5.38%. The yield to call is 5.38%, which is lower than the coupon rate of 6.25%, further validating that the bond is trading at a premium. LO 7.4.1

Michael owns a municipal bond, trading at par, with a 4.25% coupon rate and is in the 32% federal marginal income tax bracket. Calculate the taxable equivalent yield (TEY) for this bond.

The answer is 6.25%. TEY = tax-exempt yield ÷ (1 − marginal tax rate) = 0.0425 ÷ (1 − 0.32) = 0.0625, or 6.25%. Therefore, the TEY for Michael's bond is 6.25%. Michael would require this rate of return or higher for an equivalent taxable bond.

Portfolio immunization is designed to protect bondholders from which of the following risks? Interest rate risk Reinvestment rate risk Default risk

The answer is I and II. Portfolio immunization protects bondholders from fluctuations in interest rates and from reinvestment rate risk but does not protect against default risk.

Choose the statement regarding the correlation coefficient that is NOT correct.

The answer is combining assets with less than perfect positive correlation will not reduce the total risk or the portfolio. Combining assets with less than perfect positive correlation can reduce the total risk of the portfolio. The further the correlation coefficient between the two assets is away from +1.0, the greater the diversification benefits that may be attained.

Advantages of investing in exchange-traded funds (ETFs) include all of the following except

The answer is daily pricing. ETFs are continually priced during a trading day, just like stocks.

Which of the following correctly describes a disadvantage of cash and cash equivalents?

The answer is investors choosing to redeem their CDs prior to maturity may be subject to a substantial penalty. One of the advantages of money market deposit accounts is their liquidity. They may be used by investors as a source of funds to meet emergencies and other short-term obligations. Investments in money market mutual funds are not insured or guaranteed by the U.S. government. The rate of return on passbook savings accounts is relatively low when compared to higher risk alternatives such as government bonds.

Choose the stock value that is defined as the discounted present value based on future cash flows as determined by some form of a dividend discount model.

The answer is intrinsic value. A stock's intrinsic value is its discounted present value based on future cash flows as determined by some form of a dividend discount model.

Wendy is concerned that her investment's actual return will not equal its expected return. Point out the type of risk that she is concerned about regarding her investment.

The answer is investment risk. Investment risk is the uncertainty that an investment's actual, or realized, return will not equal its expected return.

Rossalyn is researching GFD stock to determine if she should add it to her extensive equity portfolio. This stock has a beta of 1.40 and a standard deviation of 6.85%. Assuming the market rate of return is 10%, and the current 90-day T-bill rate is 2%, what is the market and stock risk premiums for GFD stock?

The answer is market = 8.00%; stock = 11.20%. The market risk premium is calculated as follows: rm - rf = (10% - 2%). The stock risk premium is 11.20%, calculated as follows: (rm - rf)βi = (10% - 2%)1.40.

The exercise price of a put is $25 and the market price of the stock is $18. Which of the following statements is true?

The put is in the money. An investor who purchases a put option makes a profit only if the market price of the stock is lower than the exercise (strike) price of the option. Until the market price drops below the strike price, the option is said to be out of the money. It is in the money when the market price drops below the strike price. This put is in the money by $7.

If two stocks have positive covariance, which of these statements is CORRECT?

The rates of return tend to move in the same direction relative to their individual means. The answer is the rates of return tend to move in the same direction relative to their individual means. If one stock doubles in price, the other will also double in price is true if the correlation coefficient = 1. The two stocks need not be in the same industry.

Which of the following is both liquid and marketable?

U.S. Treasury bills

To have the right to the next dividend payment of a stock, an investor must purchase the stock

at least two days before the record date.

The risk associated with volatility in the price of securities due to shifts in the yield curve is

interest rate risk

Identify those risks that pertain to hedge funds. Overuse of leverage Excessive short selling Lack of transparency Lack of regulation

the answer is I, II, III, and IV. A hedge fund is subject to all these risks. In addition, hedge funds are usually subject to higher investment risk than other types of funds.

Rose purchased TRM stock for $40. A year later the stock paid a dividend of $4. At the end of the second year, Rose sold her TRM stock for $60 per share. What is the time-weighted return for TRM stock for the two-year period?

he answer is 27.58%. Time-weighted return is calculated as follows: CF0 = (40) CF1 = 4 CF2 = 60 IRR/YR = 27.58% LO 7.1.1

You are considering purchase of a stock that is currently selling for $23 and pays a dividend of $1.15 per share. The dividend is expected to grow at a rate of 15% per year for the next three years. After that, the dividend is expected to grow at a constant rate of 8%. Your required return is 13%. The maximum price you should pay for this stock is

$29.76. Explanation The value is calculated on the HP 10bII+ by solving for the NPV of uneven cash flows as follows: 13 I/YR 0 CF0 1.3225 CF1 1.5209 CF2 1.7490 + 37.7784 CF3 29.7559 SHIFT, NPV The 37.7784 is calculated from the constant growth DDM, starting at the end of the third year: [1.7490(1.08)] ÷ (0.13 - 0.08) = 37.7784

The yield curve graphically plots

A) yield on the y-axis and time on the x-axis.

Jennifer owns a state public purpose bond. She sells the bond and realizes a capital gain of $4,000. Prior to selling the bond, the total interest she had earned for the year was $99. Considering the sale and the interest amount, calculate the amount she must include in gross income.

$4,000 the interest on public purpose bonds is received tax-free by the holder. Only the capital gain realized on the sale is subject to income taxation

Tate is considering the purchase of a stock that just paid a dividend of $0.30 in the last quarter. That dividend has been steady for all of the past year. The company is expected to grow the dividend at 15% per year for the next three years, after which it is expected to grow at a constant rate of 8%. The required return is 11%. What is the maximum price Tate should pay for this stock? (Select the closest answer.)

$51.91 Using the three-step approach, first determine the dividend at the end of each of the first three years. The amount of $0.30 is a quarterly dividend and must be multiplied by 4 to get $1.20 for the current annual dividend. This amount is multiplied by 1.15 for each year in order to get: Year Dividend 1 1.38 2 1.587 3 1.8251 Next determine the value of the stock at the end of year 3 based on the dividend at the end of year 4: [1.8251(1.08)] ÷ (0.11 - 0.08) = 65.70 Using the CFj keys on the calculator (with dividends rounded), solve as follows: 11 I/YR 0 CF0 1.38 CF1 1.59 CF2 1.83 + 65.70 CF3 51.91 SHIFT, NPV

Given a required rate of return of 8%, a growth rate of 4%, a beta of 1.25, and a standard deviation of 2.5%, calculate the price-to-free-cash flow for this particular investment.

26.0 The answer is 26.0. The formula for price-to-free-cash flow: P/FCF = (1 + G) ÷ (r - g) = (1 + 0.04) ÷ (0.08 - 0.04) = 1.04 ÷ 0.04 = 26.0 After calculating this ratio, an investor would compare this P/FCF to other investments to see whether a purchase is warranted.

Galen has come to his financial planner with questions about dividends he received on some of his stock this year. He has received $1,000 in qualified dividends paid in cash. He also has received stock dividends of $4,000, but without a cash dividend option. How much will Galen have to report as dividend income for the current year? A) $4,000

1000

CPM stock is currently trading for $30 per share and has earnings of $1.50 per share. What is CPM's price-to-earnings (P/E) ratio?

20

A wash sale is deemed to have occurred within which of the following time frames?

61 days The answer is 61 days. 30 days before + date of sale + 30 days after = 61 total days.

Andy purchased a four-year bond with a coupon rate of 7.5% paid semiannually. The bond is trading for $1,025 in the secondary market. Calculate the bond's yield to maturity (YTM).

6.78% Explanation The answer is 6.78%. The bond's YTM is calculated as follows: END Mode, 2 P/YR PV = -1,025 FV = 1,000 PMT = 37.50 (1,000 × 7.5% ÷ 2) 4, DOWNSHIFT, N = 8 Solve for I/YR = 6.78%. LO 7.4.1

Bobby owns ABC stock that has mean return of 10.65%, a beta of 1.12, and a standard deviation of 9.05%. He decides to purchase MEJ stock that has a mean return of 11.5%, a beta of 0.98, and a standard deviation of 12.3%. Assume these stocks are weighted in the portfolio 70% for ABC and 30% for MEJ. Also, these stocks exhibit a covariance of 19.86. Calculate the standard deviation for this two-asset portfolio.

7.88% he answer is 7.88%. To determine the standard deviation of a two-asset portfolio, use this formula: [W2Aσ2A + W2Bσ2B + 2WAWB(COVAB)]1/2 [(0.72 × 9.052) + (0.32 × 12.32) + (2 × 0.7 × 0.3 × 19.86)]1/2 [(0.49 × 81.90) + (0.09 × 151.29) + (8.3412)]1/2 [40.1310 + 13.6161 + 8.3412]1/2 62.08831/2 = 7.8796, or 7.88% Note the standard deviation for the portfolio is lower than the standard deviations for each security. This result directly supports the low correlation between the returns of ABC and MEJ. LO 6.2.4

A stock fund had the following yearly returns: 20X514%20X67%20X7-3%20X818%20X99% What is the standard deviation of the returns?

7.97 calculation as follows: 14∑+∑+7∑+∑+3 +/-, ∑+∑+ 18∑+∑+9∑+∑+SHIFT, Sx,Sy

Which of these best describes a real estate mortgage conduit (REMIC)?

A REMIC is a self-liquidating, flow-through entity that invests exclusively in real estate mortgages or mortgage-backed securities. The answer is a REMIC is a self-liquidating, flow-through entity that invests exclusively in real estate mortgages or mortgage-backed securities. Typically, a REMIC issues debt securities to investors in the form of publicly traded REMIC bonds. A futures contract is an agreement between two parties to make or take delivery of a specified amount of a commodity or financial asset at a future time, place, and unit price.

Which of the following correctly describes a lifecycle fund?

A fund that bases and adjusts its asset allocation on a specific target date at which the investor will retire The answer is a fund that bases and adjusts its asset allocation on a specific target date at which the investor will retire. This is also known as a target-date fund

Jim is a paper maker who purchases lumber from tree farmers around his state. Which of these hedge positions should Jim consider if he is concerned with rising lumber prices?

A long hedge; Jim should buy lumber futures contracts to protect against rising lumber prices .The answer is a long hedge; Jim should buy lumber futures contracts to protect against rising lumber prices. A long hedge uses a long futures position to hedge a short position (purchasing lumber).

On December 18, 20X1, John sells some stock for a loss at $15 a share that he originally purchased for $40 per share. On January 9, 20X2, John repurchases the shares for $22 per share. What is his cost basis on the repurchased shares?

A) $47 The answer is $47. This is a wash sale because the shares were repurchased within 30 days of their sale. The loss is then disallowed for tax purposes, and the disallowed loss is added to the repurchase price to determine the new cost basis. $40 - $15 = $25 disallowed loss, so $25 + 22 = $47 new basis.

Which of the following statements concerning the purpose of an investment policy statement is CORRECT? An investment policy statement is a written document that establishes client objectives and sets limitations on the investment manager. The investment policy statement can be used as the basis to measure the manager's performance against the stated objectives and constraints. Possible investment strategies that should be pursued by an investment adviser on behalf of the client begin with the formulation of a complete and thorough investment policy statement. An allocation among asset classes and their respective weights is a part of any investment policy statement.

A) I and III The answer is I, II, III, and IV. All of these statements are correct. In addition, the investment policy statement can be provided to the portfolio manager to use in establishing and managing the characteristics of the client's portfolio.

Most fixed-income securities are subject to which of the following risks? Purchasing power risk Liquidity risk Default risk Reinvestment rate risk

A) I, II, III, and IV Fixed-income securities are subject to a number of risks including purchasing power, liquidity, default, and reinvestment rate risk.

Select the repayment period that subjects the investor to least amount of interest rate risk.

A) Tranche A The answer is Tranche A. Once all of the obligations of Tranche A are satisfied, all principal payments are made to the second tranche (Tranche B) and so on until all of the tranches are repaid. As a result, the holders of Tranche A have less interest rate risk than do the holders of all the other tranches, because the maturity date (or repayment period) of the mortgage principal obligation is the shortest.

An investor would consider converting a convertible bond into common stock if the bond's

A) market price is less than the conversion value.

Limited partnerships are distinguished by which of the following? The general partner controls the business activities of the partnership. The limited partners participate in the business venture with limited liability. The general partner determines when distributions are made to the limited partners. The limited partners may have difficulty selling their interests.

All of these statements are correct. Limited partnerships are characterized by a partnership entity that consists of a general partner and limited partners.

Calculate the expected rate of return for Stock A based on the following information: Risk-free rate of return: 3.85% FactorSensitivityRisk PremiumInflation1.24.00%Industrial Production0.95.75%PopulationGrowth0.52.60%

B) 15.13% The answer is 15.13%. The expected rate of return for Stock A based on the arbitrage pricing model (APT) is calculated as follows: ri = 0.0385 + (1.2 × 0.04) + (0.9 × 0.0575) + (0.5 × 0.0260) = 0.0385 + 0.048 + 0.0518 + 0.013 = 0.1513, or 15.13%. LO 8.2.1

Phil has computed a required return for the Pepsi stock he is considering purchasing. He believes that interest rates and inflation will change over the expected holding period. Therefore, he adjusted the required return for his projected changes in these factors. Which of the following stock market theories did Phil use?

Arbitrage pricing theory An investor using the APT starts with a required return for a security, possibly computed using the CAPM. The investor then adjusts the required return for a multitude of factors that may affect that particular security, such as interest rates, industrial production, and inflation.

he dollar has been declining against major world currencies for some time. A portion of your international equity holdings includes a sizable stake in Japanese stocks. As the dollar falls relative to the Japanese yen, which of these conditions would you expect to occur? U.S. investors in mutual funds that invest in Japanese securities increase their total returns. Companies that export products to Japan find that their products become more expensive to Japanese consumers. Japanese automobiles imported to the United States become more expensive to U.S. consumers. The U.S. currency experiences a devaluation with respect to the yen.

B) I, III, and IV The answer is I, III, and IV. II is incorrect because exporters benefit from a devaluation of the U.S. dollar. Foreign products become more costly and foreign purchasers of U.S. products use fewer yen to purchase products sold in U.S. dollar terms.

Jane considers herself to be a conservative investor. To generate additional income, she wants to add an investment-grade bond to her portfolio. She lives in a state that does not have an income tax and she is in the 35% federal income tax bracket. Select the best choice for her portfolio.

Bond A, AA rated municipal bond with a 3.5% coupon rate The answer is Bond A, AA rated municipal bond with a 3.5% coupon rate. Even though Bond C has the highest after-tax rate of return, this bond would not be appropriate for Jane based on her desire for an investment-grade bond. Therefore, Bond A would be the best choice. Calculations: Bond A: 3.5% Bond B: 4.75% × (1 - 0.35) = 3.0875% Bond C: 6% × (1 - 0.35) = 3.90% Bond D: 1.5% × (1 - 0.35) = 0.9750% LO 7.4.1

Identify which of these is NOT a source of systematic risk.

Business risk

Which of the following is a potential pitfall of mutual fund investing?

Buying a fund ranked number one if the fund is misclassified

Dividend reinvestment plans offer which of these advantages?

C) A convenient means to accumulate shares The answer is a convenient means to accumulate shares. The advantage to an investor is the saving of commissions by using a dividend reinvestment plan (DRIP). The advantage is to an investor, not to the company. The use of a DRIP has no effect on the stock's par value. A dividend reinvestment program has no effect on the company's ability to retain more earnings.

All of the following statements correctly describe certificates of deposit (CDs) except

CDs typically pay a variable interest rate. The answer is CDs typically pay a variable interest rate based on the term of the certificate. CDs typically pay a fixed interest rate, with higher interest rates offered for longer-term certificates.

Company A and Company B are in the same industry and have approximately the same dollar amount of assets and operating income. Company A has a return on equity (ROE) of 28% and Company B has an ROE of 12%. Which of the following statements best identifies the major difference causing the disparity in ROE between Company A and Company B?

Company A has more debt than Company B. Explanation Both depreciation expense and S, G & A expenses are used to obtain operating income, which is the same for both companies. Generally, the most significant factor in raising one company's ROE above another company's is the greater use of debt. The company having the greater percentage of debt, assuming the cost of the debt is less than the return earned from the debt proceeds, will have the highest ROE.

All of the following statements correctly describe a type of money market instrument except

Eurodollars are Eurodollar-denominated deposits maintained at banks within the United States.

hich of these is NOT a type of unsystematic risk?

Exchange rate risk The answer is exchange rate risk. Exchange rate risk is a type of systematic risk. Systematic risks are those risks that affect the entire market. Systematic risks include market risk, interest rate risk, purchasing power risk, reinvestment rate risk, and exchange rate risk.

Jill is in the 24% marginal tax bracket. She owns a $1,000 par value public purpose municipal bond that pays $35 interest semiannually. What pretax yield on corporate bonds would be comparable to the yield on Jill's current investment?

Explanation pretax yield=after tax yield(1 −marginal tax rate)pretax yield=after tax yield(1 −marginal tax rate) $35 × 2 = $70 annual interest = 7.0% coupon rate taxable equivalent yield (TEY) = 7.0% ÷ (1 - .24) = 7.0% ÷ 0.76 = 9.21%

Larry is looking to add a real estate investment to his portfolio that is publicly traded on the exchanges, thereby offering him diversification and marketability. He has decided that a real estate investment trust (REIT) is the best choice and asks his financial planner for information about the REITs available for purchase. Based on Larry's request, the financial planner explained the various investment choices. Which of these was incorrectly stated by his financial planner?

For the shareholders, income received is considered passive income. he answer is for the shareholders, income received is considered passive income. For REIT shareholders, income received is considered investment (not passive) income. Similar to a closed-end investment company, a REIT invests in real estate, short-term construction loans, and mortgages. Some REITs are publicly traded on the exchanges and may sell at a premium or discount to net asset value. As a result, the REIT investor achieves diversification and marketability.

Gordon, age 40, wants to invest in a mutual fund that will provide capital appreciation. He wants a fund that will do as well as the overall market and has a low expense ratio, but he does not want to assume a high risk to achieve his objective. He is considering purchasing one of the following mutual funds: Fund A: a growth mutual fund that has a beta of 1.10 and invests in medium- to high-grade common stock Fund B: an index mutual fund that has a beta of 1.00 and invests in common stock that mirrors the S&P 500 Index Which of these funds would best meet Gordon's objective?

Fund B, because it has a beta of 1.00, has low expenses, and is less risky Fund B can be expected to do as well as the overall market, will have a low expense ratio, and is less risk, as measured by beta, than Fund A.

Select the entity that issues guaranteed investment contracts (GICs).

The answer is insurance companies. GICs are issued by insurance companies. They are called guaranteed investment contracts because their rate of return is guaranteed by the insurance company for a fixed period. They are not guaranteed by the FDIC.

Which of the following are implications of the weak form of the efficient market hypothesis (EMH)? Stock prices fully reflect all historical price behavior. Consistently superior performance is common. Fundamental analysis may produce superior investment performance. Fundamental and technical analysis can produce superior investment performance.

I and III Technical analysis is not considered valuable under any of the forms of the EMH; fundamental analysis is considered valuable under the weak form only.

Which of these statements regarding investment risk is CORRECT? A firm's decision to buy back some of its own stock in the open market by borrowing funds through a new bond issue is an example of reinvestment rate risk. Rising inflation represents purchasing power risk. A decline in a firm's share price as a result of a 20% decline in the S&P 500 Index represents market risk. A reduction in the value of an international stock mutual fund because of a depreciation of the Euro is an example of exchange rate risk.

II, III, and IV Explanation The answer is II, III, and IV. Only statement I is incorrect. A firm's decision to buy back some of its own stock in the open market by borrowing funds through a new bond issue is an example of financial risk. LO 6.1.2

Which of the following steps are involved in the investment planning process? Establishment of a brokerage account Selection of assets for investment Implementation of investment plan Determination of ability to invest

II, III, and IV The answer is II, III, and IV. Establishing a brokerage account is not a necessary step in the investment process; however, it may be included in the implementation phase.

In order to do an effective job of investment counseling, which of the following should be analyzed and reviewed? Financial goals Client tax situation Client financial statements Client preferences, investment understanding, and experience

In order to do an effective job of investment counseling, which of the following should be analyzed and reviewed? Financial goals Client tax situation Client financial statements Client preferences, investment understanding, and experience

Jack sells short 200 shares of ABC stock at $38.50 with a 50% initial margin. ABC pays a dividend of $0.50 per share after he sells the stock. Jack then buys back the stock for $32. Calculate his percent gain or loss.

Jack had a total percent gain of 31.17%, calculated as follows: Jack's investment: 200 × $38.50 × 0.50 = $3,850 Proceeds: $7,700 Cost: (6,400) Gain: $1,300 Less dividend payment: (100) Net gain: $1,200 $1,200 ÷ $3,850 = 31.17% gain

Portfolio immunization is designed to mitigate which type of risk?

The answer is interest rate risk. A bond portfolio is immunized when the duration of the portfolio is equal to the time horizon of the investor, thereby mitigating both interest rate risk and reinvestment rate risk.

Which of these represents the best reason to include real estate as part of an investment portfolio?

Low correlation between real estate and equity investments The answer is low correlation between real estate and equity investments. Real estate's low correlation with equities provides investors with an additional way to diversify a portfolio. Real estate ownership may present an opportunity for capital appreciation. Investment in real estate generally requires substantial management costs and is not liquid.

Which one of these alternatives correctly outlines the importance of the portfolio perspective?

Market participants should analyze the risk-return trade-off of the portfolio, not the risk-return trade-off of the individual investments in a portfolio.

Income or dividends produced by which of the following securities is exempt from federal income tax?

Municipal bonds

If a mutual fund's beta and standard deviation are expected to decrease in the future, its average annual return and the market average annual return are expected to remain the same, and the risk-free rate is expected to remain constant, which of the following shows the real effect this would have on the following performance measures? OptionAlphaSharpe RatioAincreasedecreaseBdecreasedecreaseCdecreaseincreaseDincreaseincrease

Option D The answer is Option D. A decrease in the risk level decreases the denominator of the Sharpe ratio, while the numerator stays constant, thereby increasing the Sharpe ratio. The decreased risk level, as measured by beta, decreases the expected return for the fund, while the actual portfolio return remains constant, thereby increasing the alpha.

Which of the following regarding mutual fund performance is CORRECT? Past performance is a reliable predictor of future performance. Past performance offers some indication as to the competency of fund managers.

Past performance offers some indication as to the competency of fund managers.

Which one of these is CORRECT regarding preferred stock?

Preferred stock's value is based on prevailing interest rates. The answer is preferred stock's value is based on prevailing interest rates. The value for a preferred stock is its dividend divided by prevailing interest rates.

Which one of these risks is specifically related to mortgage pass-through securities?

Prepayment risk The answer is prepayment risk. The payments received from mortgage-backed securities consist of both interest and principal payments. When interest rates fall, homeowners refinance their homes, thereby paying off the principal on the original mortgages. This prepayment of principal is called prepayment risk.

What type of index is the Dow Jones Industrial Average (DJIA)?

Price-weighted Explanation The answer is price-weighted. The DJIA is price-weighted, meaning that higher-priced stocks will have more impact on the average than lower-priced stocks. Cap-weighted, which is the same as value-weighted, means the prices of stocks with the largest capitalization relative to the market capitalization of the entire index will have the greatest impact on the index. Equally weighted means that the price movement of each stock in an index has the same impact as that of any other stock in the index.

choose the risk that is attributable to cash and cash equivalents.

Purchasing power risk

Mutual fund QUE has a correlation coefficient with the market of 0.82, a beta of 1.05, and a standard deviation of 4%. The risk-free rate of return is 3.5%, and the return on the market is 12%. Mutual fund POI has a Sharpe ratio of 2.05, a Treynor ratio of 0.11, and an alpha of 0.70%. Decide which of the following a rational investor would select if the market's standard deviation is 2% and QUE realized a 13% return.

QUE over POI because QUE's Sharpe ratio is 2.38. The answer is QUE over POI because QUE's Sharpe ratio is 2.38. QUE's alpha = 13% - [3.5% + (12% - 3.5%) 1.05] = 0.58% QUE's Treynor ratio = (0.13 - 0.035) ÷ 1.05 = 0.09 QUE's Sharpe ratio = (0.13 - 0.035) ÷ 0.04 = 2.38 QUE's coefficient of variation = 4% ÷ 13% = 0.31 Because QUE's R2 equals 67% (0.82 × 0.82), alpha and Treynor are not appropriate performance measures for comparison purposes. Because we do not know POI's coefficient of variation, we must use the Sharpe ratio to select the better risk-adjusted return.

Mary owns a portfolio consisting of two stocks, FUR and STC. FUR stock has a beta of 0.90, a standard deviation of 5%, and an actual return of 10%. STC stock has a beta of 1.10, a standard deviation of 7%, and an actual return of 12%. Assume a risk-free rate of return of 3%. Using the Treynor ratio, evaluate which stock had the better risk-adjusted performance.

STC stock Explanation The answer is STC stock. The formula for the Treynor ratio is: Tp = (rp − rf ) ÷ βp Treynor for FUR stock = (0.10 − 0.03) ÷ 0.90 = 0.0777 Treynor for STC stock = (0.12 − 0.03) ÷ 1.10 = 0.0818 As a result, the Treynor ratio for STC stock is higher than FUR stock, indicating that STC has outperformed FUR on a risk-adjusted basis. LO 7.2.1

Which of the following terms is considered early-stage business funding for the purpose of research and development of an idea?

Seed financing

An investor who owns a sector fund that has substantial unsystematic risk and would like to know how a portfolio manager performed on a risk-adjusted basis would use which of the following indicators? A) Beta

Sharpe uses standard deviation and assumes the portfolio is not well diversified and measures total risk.

Which of the following statements best describes banker's acceptances?

Short-term drafts drawn by a private company on a major bank used to finance imports and exports

Which of the following statements regarding the efficient frontier is CORRECT?

Shows all portfolios that offer the highest expected returns for each level of risk The answer is shows all portfolios that offer the highest expected returns for each level of risk. Efficient frontiers take all of the possible combinations of assets that can be held in a portfolio into account in order to represent the set of portfolios that offer the highest expected return for a given amount of risk.

Which statement regarding single-country, closed-end funds is NOT correct?

Single-country funds trade at the fund's NAV. Single-country funds are closed-end funds and, like other closed-end funds, generally trade at a discount to NAV.

Ellen is an aggressive investor who is willing to take above-average risk to maximize capital appreciation. She is not interested in current income. The two mutual funds that she has under consideration are the following: Small-Cap Value Fund Small-Cap Growth Fund Current yield 1.1% 0.6% Five-year total return 14.2% 16.7% Beta 0.89 1.14 Sharpe ratio 1.12 0.93 Standard deviation 8.20 12.60 Alpha +2.70 +0.40 Which mutual fund is more appropriate for Ellen and why?

Small-cap value fund, because its risk-adjusted performance statistics are superior Perhaps a fund other than one with a lower-than-market beta fund should have been considered since she is an aggressive investor. However, no such alternative is shown. Total return alone is insufficient reason to select one fund over another. Risk-adjusted return is the appropriate measure. The value fund has a higher alpha, a higher Sharpe ratio, and a higher risk-adjusted performance when the total return is divided by beta. The current yield is relatively low for both funds.

Consider the following information for the CPM International Growth Fund: Average annual rate of return7.45%Average market rate of return8.50%Beta1.25Standard deviation4.55%Risk-free rate of return3.50% Select the statement that is NOT correct.

The Treynor ratio for the fund is 0.0400. e answer is the Treynor ratio for the fund is 0.0400. Based on the information provided, the Treynor ratio for the fund is 0.0316, calculated as follows: Sharpe ratio = (0.0745 - 0.0350) ÷ 0.0455 = 0.8681 Treynor ratio = (0.0745 - 0.0350) ÷ 1.25 = 0.0316 Jensen's alpha = 7.45% - [3.50% + (8.50% - 3.50%) 1.25] = -2.30% When Jensen's alpha is positive, the fund manager outperformed the overall market. In this case, alpha indicates that the fund manager has underperformed.

Which type of hedge should a wheat farmer select?

The answer is a short hedge—sell wheat futures contracts as a hedge against a decline in the price of wheat. Because the wheat farmer is long wheat, he would be interested in a short hedge to protect against a decline in the price of wheat.

According to Markowitz, an investor's optimal portfolio is determined when the investor's

The answer is highest indifference curve is tangent to the efficient frontier. The optimal portfolio for an investor is determined as the point when the investor's highest indifference curve is tangent to the efficient frontier.

Your client has just opened a margin account with your brokerage firm and purchased 500 shares of stock for $60 per share. The firm has a 55% initial margin and 35% maintenance margin policy. Calculate the stock price at which your client will receive a margin call.

The client will receive a margin call when the price of the stock drops to $41.54, calculated as follows: margin call = ($60 × 0.45) ÷ (1 - 0.35) margin call = $27.00 ÷ 0.65 = $41.5385, or $41.54

The conversion value of a convertible bond with a conversion ratio of 25, a conversion price of $40, and a market price of the underlying stock of $32 is

The conversion value is the value if converted and is determined by multiplying the market price of the underlying stock by the conversion ratio: $32 × 25 = $800

Taylor, a personal friend of yours, has been a practicing veterinarian for eight years. She is 35 years old and has a 3-year-old daughter. Taylor has a moderate risk tolerance, wants to save for retirement, and is considering increasing her investment in the following mutual fund. Taylor has asked you for your recommendation. Risk-free return 7.0% Return of market 12.5% Growth and Income FundNAV (beginning of year)$53.00NAV (end of year)$52.75Dividend$3.25Capital gains distributed$2.75Beta0.70Realized return10.85% Which of the following is CORRECT regarding the risk and return of the fund?

The fund has less risk and less return than the market.

Mary owns a put option with an exercise price of $20 per share. The option is currently trading for $0.53 and the underlying stock is currently trading for $19.67 per share in the secondary market. Based on this information, select the INCORRECT statement.

The option is out-of-the-money.

You are comparing two stocks based on the statistics below. Which one is the better investment based on the risk/return relationship? Stock AStock BAverage Return3.00%9.00%Standard Deviation3.9511.86

The two stocks have equal risk/reward profiles Explanation The answer is the two stocks have equal risk/reward profiles. The coefficient of variation is used to evaluate risk/return and is 3.95 ÷ 3.00 = 1.32 for stock A and 11.86 ÷ 9.00 = 1.32 for stock B, so both are equal in the amount of return relative to the risk. LO 6.2.3

A Japanese bank has decided to use some of its U.S. dollar reserves, resulting from the U.S. merchandise trade deficit with Japan, to invest in U.S. Treasury bonds. The U.S. Treasury securities pay approximately 6% interest, compared to 2% interest paid on Japanese bonds. For the Japanese bank to retain at least this differential in interest income, which of these situations in the foreign exchange market would have to occur?

The yen would have to depreciate or remain steady relative to the dollar during the holding period of the bond.

Which of the following statements best describes Eurodollars?

U.S. dollar-denominated deposits at banks outside the United States

Which of the following statements correctly distinguishes an investor who practices indexing? The investor purchases index mutual funds. The investor is practicing an active form of portfolio management. The investor is attempting to beat the market (i.e., S&P 500). Indexing and purchasing index funds tends to exhibit low administrative costs and a low turnover of existing assets.

Which of the following statements correctly distinguishes an investor who practices indexing? The investor purchases index mutual funds. The investor is practicing an active form of portfolio management. The investor is attempting to beat the market (i.e., S&P 500). Indexing and purchasing index funds tends to exhibit low administrative costs and a low turnover of existing assets.

rcy may add 100 shares of LKM corporation stock to her investment portfolio. The stock recently paid a dividend of $1.85 per share. The dividend is expected to grow at a constant rate of 2.25% per year. Her required rate of return is 7%. The stock is currently trading for $35.75 per share. Determine whether she should purchase the stock and why.

Yes, the stock is undervalued based on an intrinsic value of $39.82. The answer is yes, the stock is undervalued based on an intrinsic value of $39.82. Using the constant growth dividend discount model, the intrinsic value of the stock is $39.82. V = ($1.85 × 1.0225) ÷ (0.07 - 0.0225) V = 1.8916 ÷ 0.04750 V = 39.8232, or $39.82 Based on this value, the stock is undervalued relative to its price in the secondary market.

Choose a feature of zero-coupon bonds.

Zero-coupon bonds are sold at a deep discount from par value and have no coupon payments. They are redeemed for their face value at maturity. These bonds have maximum price volatility and respond sharply to interest rate changes. Zero-coupon bonds have durations equal to their term to maturity.

An investment, such as an option, whose value is based on that of another security is classified as

a derivative. Explanation The answer is a derivative. An investment whose value is based on that of another security is a derivative. Examples of derivatives include options, futures, and warrants.

An investor is interested in holding a diversified portfolio to reduce unsystematic risk. This can best be accomplished by buying stock in

companies with low correlation coefficients between them. Holding stocks that have a low correlation coefficient between them will result in a diversified portfolio that reduces and virtually eliminates the degree of unsystematic (business) risk in the portfolio. Buying stocks in international companies and stocks with low betas can help to reduce systematic risk, but only if they have low correlations with other stocks. Buying stocks in companies with strong revenue and earnings growth often results in acquiring significant company-specific risk that is attributable to the underlying business. LO 6.2.4

Robyn's bond has a current market value of $1,456.78 and a Macaulay duration of 12.5. Assuming the bond's yield to maturity (YTM) changes from 5.55% to 6.25%, calculate the estimated percent change in the price of the bond and the new expected market price of the bond (based on the percent change).

he answer is −8.29%, $1,336.01. The formula for determining the change in the price of the bond: ΔP/P = −D(Δy ÷ (1 + y)) ΔP/P = −12.5[(0.0625 − 0.0555) ÷ 1+0.0555] ΔP/P = −12.5(0.0066) = −0.08290, or −8.29% The bond's price should decrease by 8.29% and, therefore, sell for $1,336.01 in the secondary market.

All of these positions are used to create a zero-cost collar except

purchasing a call option on the stock .The answer is purchasing a call option on the stock. To create a zero-cost collar, the investor will not purchase a call option on the stock. Rather, the investor will use the premium received from writing the call option to subsequently purchase the put option on the underlying stock, thereby creating a cashless collar and protection from downside risk.

Which of the following bond strategies should not be recommended if an investor expects interest rates to increase?

sell low-duration bonds and buy longer duration bonds. The answer is sell low-duration bonds and buy longer duration bonds. Prices decline when interest rates rise. An investor expecting an increase in interest rates should sell more volatile bonds and purchase less volatile bonds. Bonds with large coupons and short durations are less price volatile than low-coupon, long-term, and long duration bonds.

An investor who makes the assumptions that security prices reflect all available information, that organized exchanges can execute trades rapidly, that security prices change rapidly in response to new information, and that security prices follow random patterns is a believer in

the efficient market hypothesis. Explanation In its purest form, the efficient market hypothesis suggests that investors are unable to outperform the market on a consistent basis. The fundamental assumption of the theory is that current stock prices reflect all available information for a company and that prices rapidly (or immediately) adjust to reflect any new information.

All of the following correctly identify features of limited partnerships except

the limited partners may participate in the management of the partnership. The answer is the limited partners may participate in the management of the partnership. Disadvantages of limited partnerships include the following: (1) they are generally riskier than bonds or exchange-traded equities; (2) they are generally illiquid; (3) limited partners cannot participate in the management; and (4) the sale of partnership interest may be restricted. In addition, the general partner has unlimited liability. LO 1.1.1

All of the following correctly describe disadvantages of cash and cash equivalents except

) an investor may quickly convert a money market deposit account to cash to meet short-term needs. The answer is an investor may quickly convert a money market deposit account to cash to meet short-term needs. One of the advantages of money market deposit accounts is their liquidity. They may be used by investors as a source of funds to meet emergencies and other short-term obligations.

Identify the yield-curve theory that relies on the laws of supply and demand for various maturities of borrowing and lending.

Market segmentation theory

A convertible bond has a par value of 1,000, a current market value of $1,200, and an investment value of $1,050. The bond is convertible into 25 shares of common stock. What is the investment premium of this bond?

The answer is $150. The investment premium is a measure of the downside risk of the bond and is the difference between the current market value and the investment value. $1,200 - $1,050 = $150 investment premium. LO 2.2.1

George Jones owns a convertible bond that has a conversion price of $50 per share and an annual coupon rate of 6.0%. Interest is paid semiannually. The current market price of the stock is $51 per share. The investment value of the bond is $890, and the bond currently sells for a market price of $1,080. What is the downside risk of this bond?

The answer is $190. The downside risk of a convertible bond is the dollar or percentage decline from the current market price of the convertible bond to the investment value of the bond: $1,080 - $890 = $190.

Nellie has accumulated $500,000 in a money market deposit account at ABC Bank and Trust. She is worried about the number of bank failures in the recent years and transfers $250,000 into a money market mutual fund paying a slightly higher return offered by her friend's investment firm. Determine the amount she has insured by the Federal Deposit Insurance Corporation (FDIC).

The answer is $250,000. Nellie's FDIC insured funds remain at $250,000. The money market deposit account is insured up to $250,000, but the money market mutual fund is not FDIC insured.

ordan has the following gains and losses from the previous year: $10,000 long-term capital gain $6,000 long-term capital loss $7,500 short-term capital gain $15,000 short-term capital loss What is the tax ramification of these transactions?

The answer is $3,000 deductible loss, $500 carryover loss. First, net the long-term gains and losses, and the short-term gains and losses. A $10,000 long-term capital gain with a $6,000 long-term capital loss equals a net long-term capital gain of $4,000. For short-term capital gains and losses, net the $7,500 short-term capital gain with the $15,000 short-term capital loss, which comes to a $7,500 short-term capital loss. Then, take the $4,000 long-term capital gain, and offset it against the $7,500 short-term capital loss, for a net short-term capital loss of $3,500. When Jordan files his tax return he will be able to deduct $3,000 of the $3,500 loss, and will carry forward to the next tax year the remaining $500 short-term loss.

Darla, a U.S. citizen and resident of Georgia, owns a 5% coupon corporate bond, a 4% coupon State of Georgia municipal bond, and a 3% coupon U.S. Treasury note. Darla's marginal state income tax rate is 6% and federal tax rate is 24%. If Darla invested equal amounts in each of the three bonds, what is her after-tax rate of return on the portfolio?

The answer is 3.26%. Because the corporate bond is taxable by the state and the federal government, its after-tax return is 3.5% [5% × (1 - 0.30)]. The State of Georgia municipal is not taxable by either government entity. The Treasury note is taxable by the federal government; therefore, its after-tax return is 2.28% [3% × (1 - 0.24)]. Averaging the three rates equals 3.26% [(3.5% + 4% + 2.28%) ÷ 3].

Which of these statements correctly describes differences between preferred stock and long-term bonds? Interest paid by firms is a tax-deductible expense; dividends paid on preferred stock are not tax deductible. Bonds usually have a finite maturity; preferred stock is usually perpetual. Bonds pay a fixed amount of interest; preferred stock pays a fluctuating dividend based on earnings. Interest on bonds and preferred stock dividends are legal obligations of a firm that must be paid.

The answer is I and II. Bonds pay a fixed periodic interest that is tax deductible by the firm; preferred stock dividends are fixed, but they are not tax deductible. Only the interest on bonds is a legal obligation that must be paid; dividends on preferred stock can be deferred (if there is a cumulative feature) or skipped.

Identify which of these statements regarding bonds is CORRECT. If a bond is issued in registered form, payments will be made to the owner of record. If a bond is issued in bearer form, payments will be made to whoever holds or possesses the bond. A bond acquired in the secondary market at a discount is called a market discount bond. The amount attributable to a market discount is always includable in income in the year of acquisition.

The answer is I, II and III. Only statement IV is incorrect. A bond is a debt security obligating the issuer to make periodic interest payments and to repay the principal at the time of maturity. The amount attributable to a market discount is generally not includable in income until sale or disposition of the bond, and then it is treated as interest income.

What are the advantages of a private placement to the issuing firm? Avoidance of disclosure requirements Reduction in the time needed to raise the capital Avoidance of some costs associated with publicly selling securities Limited to 25 unaccredited investors, but is available to an unlimited number of accredited investors

The answer is I, II, and III. A private placement is limited to 35 unaccredited investors but is available to an unlimited number of accredited investors and eliminates the need for full disclosure and SEC registration. In addition, costs associated with the sale of securities to the public are avoided. Because there is no need for SEC registration, the company can quickly raise the needed capital. However, a private placement usually means a higher, not lower, cost of capital to the issuing firm. LO 1.1.1

Identify which of these statements regarding zero-coupon bonds is NOT correct. Zero-coupon bonds are purchased at par and defer interest payments until maturity. Because there are no coupon payments for zero-coupon bonds, no current income is recognized. A zero-coupon bond is issued at a discount and pays semiannual interest payments. Corporations may favor zero-coupon bonds because they have an extended period to use the money that has been raised by the offering.

The answer is I, II, and III. Zero-coupon bonds are issued at a discount and pay only the par value at maturity; 'interest' is not paid during the term of the bond but 'interest/growth' are paid at the end. Even though no periodic interest payments are made, the bondholder must recognize the accrued interest each year for income tax purposes.

Dave and Pam Larson, ages 65 and 63, respectively, recently retired. They successfully saved for their retirement throughout their careers using a low-risk approach. They would like to restructure their investments to have current income now to travel in their leisure time. Which of the following investment alternatives would be appropriate for the Larsons' goal? Equity income mutual fund shares Aggressive growth mutual fund shares Newly issued U.S. government bonds GNMA fund shares

The answer is I, III, and IV. Aggressive growth mutual funds have a primary objective of capital appreciation and pay out little to no dividends. This investment would not provide the Larsons with their desired current income. The remaining investment alternatives all fit within the Larsons' risk tolerance.

Which of these describe similarities between preferred stock and long-term bonds? Both dividends and interest are tax-deductible expenses for the issuing corporations. Both generally pay a fixed periodic payment. Both preferred dividends and interest must be paid before common stock cash dividends are paid.

The answer is II and III. Both preferred stock and long-term bonds generally pay a fixed periodic payment, and both preferred dividends and interest must be paid before common stock cash dividends are paid.

Which of these statements regarding bond portfolio immunization is CORRECT? Immunization allows an investor to ensure that the value of his or her bond portfolio remains the same, regardless of whether interest rates increase or decrease. Immunization is accomplished by creating a portfolio whose duration is equal to the investor's investment time horizon. Immunization allows investors to earn a current yield that is equal to the yield to maturity. Immunization allows an investor to earn a specific rate of return, regardless of whether interest rates increase or decrease.

The answer is II and IV. Immunization attempts to protect the yield of a bond portfolio from changes in interest rates. An immunized portfolio is expected to provide a specific return over the investment time horizon. If interest rates change during the investment period, the capital losses are expected to be offset by the gains on reinvestment income. Immunization is accomplished by creating a portfolio whose duration is equal to the investor's investment time horizon. LO 2.3.1

Barbara, a Louisiana resident, is in the 35% marginal federal income tax bracket and the 6% marginal state income tax bracket. Select the bond that would provide Barbara with the highest after-tax rate of return.

The answer is Louisiana municipal bond with a coupon rate of 5.5%. U.S. Treasury bond (exempt from state income tax): 6% × (1 - 0.35) = 3.90% Corporate bond: 8% × [1 - (0.35 + 0.06)] = 4.72% Texas municipal bond (exempt from federal income tax): 5.8% × (1 - 0.06) = 5.45% Louisiana municipal bond (exempt from both federal and state income tax): 5.5%

All of the following statements concerning the types of orders used to buy and sell securities are correct except

The answer is a market order has the lowest priority and is subject to the fluctuations and timelines of the market. The market order, which is the most popular and has the highest priority, is subject to the fluctuations and timeliness of the market.

Which of these is NOT correct when defining an accredited investor under Rule 501 of Regulation D?

The answer is a natural person who has individual net worth, or joint net worth with the person's spouse, that exceeds $1 million at the time of the purchase, including the equity in a primary residence. The $1 million net worth requirement excludes the equity in the investor's primary residence.

An investor who carefully chooses a bond that has a duration that matches the investor's required holding period is practicing

The answer is an immunization strategy. An investor who chooses a bond that has a duration equal to the investor's desired holding period is practicing immunization. Because a bond's reinvestment rate risk and price risk tend to 'offset' each other, immunization can be used to cancel out interest rate risk.

An investor wants all of her bonds to mature in 10 years. She buys two bonds immediately, two bonds two years from now, and two more bonds four years from now. As a result, the bonds purchased immediately have a maturity of 10 years, the bonds purchased two years later have a maturity of eight years, and the bonds purchased four years later have a maturity of six years. Select the type of bond strategy she is using for her portfolio.

The answer is bond bullet. When using the bond bullet strategy, an investor purchases a series of bonds with similar maturities focused on one point in time. This strategy may be an effective method in matching duration to the cash needs of an investo

Which of the following statements regarding duration is CORRECT? Risk-averse investors should consider bonds with low durations. Aggressive investors should consider bonds with low durations when they anticipate that interest rates will rise.

The answer is both I and II. Risk-averse investors should consider bonds with low durations. Aggressive investors should consider bonds with high durations when they anticipate that interest rates will decline, and they should consider bonds with low durations when they anticipate that interest rates will rise.

Which of the following statements is CORRECT? If an investor expects a decline in market interest rates, she should attempt to construct a portfolio of long maturity bonds with low coupon rates. If the investor expects an increase in market interest rates, he should attempt to construct a portfolio of short maturity bonds with high coupon rates.

The answer is both I and II. The portfolio in Statement I will provide the investor with a portfolio that has the maximum interest rate sensitivity to take advantage of the capital gains experienced by bonds from the decrease in market interest rates. The portfolio in Statement II will provide the investor with a portfolio that has the minimum interest rate sensitivity to minimize the capital losses experienced by bonds from the increase in market interest rates.

If a bond is immunized against interest rate risk, a dollar decline in the bond's price, resulting from rising interest rates, will be approximately offset by a dollar increase in the

The answer is income from coupons reinvested over the investment horizon. By investing in bonds that have a duration equal to the investor's investment time horizon, any bond price/value changes caused by interest rate fluctuations will be approximately offset by changes in the interest earned on the reinvested coupons.

The yield curve theory that states current long-term interest rates contain an implicit prediction of future short-term interest rates is known as

The answer is unbiased expectations theory. The unbiased expectations theory states that long-term rates consist of many short-term rates and that long-term rates will be the average (or geometric mean) of short-term rates.

Which of the following is CORRECT with respect to convertible bonds and convertible preferred stock if the value of the common stock rises?

The value of convertible bonds and convertible preferred stock rises.

An individual with a short-term investment time horizon would choose what type of bonds when interest rates are expected to rise?

he answer is short-term bonds. Long-term bonds are affected by interest rate changes more than short-term bonds. If interest rates are expected to rise, an investor should invest in short-term bonds until rates peak. Risk of default of high-yield bonds increases when rates rise.

A client is considering the purchase of a $25 par preferred stock to add income to his portfolio. The stock has an 8% stated annual dividend rate and will never change. The investor's discount rate is 12%. What is the most the investor should pay for this stock?

16.66

A money market mutual fund manager recently purchased negotiable, short-term, unsecured promissory notes issued by a number of large corporations for the portfolio. Select the type of investment the money manager purchased.

Commercial paper: Commercial paper is usually issued in denominations of $100,000 or more and is a substitute for short-term bank financing. Commercial paper is normally sold at a discount and is rated for quality by a rating service.

Art and oriental rugs may not be sold through which of the following methods?

On the NYSE

Select the term that measures how far the actual outcomes of a probability distribution deviate from the arithmetic mean.

The answer is skewness. Skewness measures how far the median return is from the mean return in decimal terms.

An investor holding a Treasury bill as of the date of maturity includes

the amount of the discount as ordinary income.

Which of these risks is diversifiable?

the answer is default risk. Default risk is diversifiable (unsystematic) risk. The others are examples of systematic risk, or nondiversifiable risk.

What is the taxable equivalent yield on a municipal bond with an 8.75% return for an investor in the 24% marginal tax bracket?

The formula for solving this problem is 8.75% ÷ (1 - 0.24) = 8.75% ÷ 0.76 = 11.51%. LO 7.4.1

Zenith Mutual Fund has had the following annual returns: +12%, +18%, +22%, and -13%. What is the Zenith Fund's geometric mean return?

The simplest way to do this problem is to see how much $1 would have grown to over the four years, and then do a simple time value of money calculation. $1.00 × 1.12 × 1.18 × 1.22 × 0.87 = $1.4027. (1) PV, 1.4027 FV, 4 N, I/YR = 8.8281%.

What is the weighted average beta of a portfolio with 20% in Stock A with a beta of 0.9, 50% in Stock B with a beta of 1.2, and 30% in Stock C with a beta of 1.1?

) 1.11 The answer is 1.11. Keystrokes are 0.9, INPUT, 20, ∑+, 1.2, INPUT, 50, ∑+, 1.1, INPUT, 30, ∑+, SHIFT, 6 (alternate function is weighted average) = 1.11.

The random walk hypothesis is supported when future price changes are not correlated with past price changes. stock price changes are random but predictable. stock prices respond rapidly to new information. past information is not useful in predicting future price changes.

) I, III, and IV The answer is I, III, and IV. The random walk hypothesis assumes that stock price changes are essentially random, and therefore unpredictable; or that successive stock returns are independent of past returns. Stock prices react quickly to new information. Past information is not relevant when predicting future price changes, hence, future price changes are not correlated with past price changes. LO 8.1.1

LFM Corporation declared a record date of Wednesday, May 16, for its next quarterly cash dividend. Determine the last day an investor can purchase LFM stock and receive the current dividend.

) Monday, May 14

Your client owns a small-cap fund and wants to compare its performance to an appropriate benchmark. You would advise him to choose which benchmark?

) Russell 2000 The answer is Russell 2000. The Russell 2000 is a small-cap index.

All else remaining equal, if the dividend payout ratio decreases, the value of a company's common stock would

) increase because the company's dividend growth rate will increase. A decrease in the dividend payout ratio means that the earnings retention ratio (rr in the following formula) will increase. An increase in rr will cause an increase in g. When the higher g is inserted in the dividend discount model formula, the denominator decreases, thereby causing the value of the stock to increase. g = ROE × rr

A convertible bond's market value will NOT fall below its

) investment value.

The reason for using a ladder bond strategy is to

) lower interest rate risk For example, with a ladder bond strategy, instead of investing all money in a seven-year bond, an investor may divide the dollars among bonds with one, three, five, seven, and nine-year maturities. With this approach, instead of making a single bet on interest rates, the investor has both longer and shorter maturities, so that regardless of which way interest rates move the investor will not experience either great losses or great gains.

Stock XYZ has an average return of 18% with a standard deviation of 21. Within what range could an investor expect a return to fall 68% of the time?

-3% to 39% By definition, an investment's return will be within one standard deviation of the mean return 68% of the time. The mean return of 18% plus or minus one standard deviation is 18% - 21% (-3%) and 18% + 21% (39%).

The Mountain Fund has a standard deviation of 22, a mean return of 15%, and a correlation coefficient with the S&P 400 Mid-Cap Index of 0.85. Mountain Fund is subject to how much systematic risk?

72% R-squared gives us the amount of systematic risk, and we have been given R (correlation coefficient). So, we square 0.85 to come up with an R-squared of 0.7225, or 72%.

The Finite Mutual Fund has a correlation coefficient of 0.90 with the S&P 500 Index. How much of the price movement of the Finite Mutual Fund is explained by the S&P 500 Index?

81% R-squared gives us the amount of systematic risk, and we have been given R (correlation coefficient). So, we square 0.90 to come up with an R-squared of 0.81, or 81%.

he use of P/E ratios to select stocks suggests which of these?

A stock should be purchased if it is selling near its historic low P/E. The answer is a stock should be purchased if it is selling near its historic low P/E. While purchasing stocks near their historically high P/E ratio could continue to represent value, a much better time would be to purchase stocks that are at their historic low ratios. LO 4.3.2

The weak form of the efficient market hypothesis reinforces the value of technical analysis. implies that technical analysis is not worthwhile. implies that fundamental analysis is not worthwhile. implies that inside traders cannot earn superior risk-adjusted returns.

B) II only The answer is II only. The weak form implies that information contained in historical stock prices is fully incorporated into current stock prices; therefore, technical analysis (the study of historical prices and volume) is not worthwhile in predicting future prices. This form neither refutes fundamental analysis nor implies that traders using insider information cannot earn superior profits.

Which of the following combinations of risks is associated with art and other collectibles?

B) Liquidity risk and market risk

Which one of the following statements CORRECTLY matches a technical indicator to the information it provides in signaling a change from a bear to a bull market?

Barron's Confidence Index indicates that the yield differential between low-quality bonds and high-quality bonds is decreasing.

LTD Inc., a candy manufacturer, is considering the use of sugar futures contracts because sugar is a major ingredient in the manufacturing process. What type of hedge position should LTD take in the sugar futures market, and why?

C) A long hedge; the company should purchase sugar futures contracts because it is hedging against higher sugar prices. The answer is a long hedge; the company should purchase sugar futures contracts because it is hedging against higher sugar prices. An investor or producer who enters into a short hedge sells a futures contract; one who enters into a long hedge purchases a futures contract. A grower owns crops in the field and enters into a short hedge to hedge against declining prices. A user of the commodity, like LTD, is concerned about rising prices and, therefore, enters into a long hedge to lock in the current price of the commodity.

Which of the following is a general characteristic of hedge funds?

Charge both a management fee and a carried interest fee

Daniel has several investment company products within his retirement portfolio. One of these investments trades on an exchange, may trade at a premium or discount to its net asset value, and has a fixed capital structure. These features illustrate which of these investments?

Closed-end investment company The answer is closed-end investment company. A closed-end investment company (closed-end fund) is a type of company whose shares trade in the secondary market.

Which one of these is a measure of a security's risk-adjusted return?

Coefficient of variation

Which one of these factors has the greatest impact on the standard deviation of a two-asset portfolio?

Covariance The answer is covariance. Covariance is the most important variable in minimizing the standard deviation of a portfolio. The weight and standard deviation are not as critical as the covariance of the two securities. Beta is not used in the formula to compute a portfolio's standard deviation.

fter suffering an unexpected job loss and losing a $125,000 annual salary, your client, Joe, comes to you to review his asset allocation. In addition to possibly having a prolonged unemployment period, Joe, age 62, would like to retire within eight years. He is concerned about the allocation of the Section 401(k) plan at his prior employer which is 100% invested in an S&P 500 Index fund. The balance of the account is $2,500,000. Based on this information, what should you do next?

Discuss Joe's risk tolerance and make appropriate changes to his plan's asset allocation.

You are about to recommend the purchase of an additional mutual fund to add to a client's portfolio, with the objective of reducing the portfolio's total risk. Upon analysis of several funds, you determine that the standard deviations of the current portfolio and each of the potential new funds are equal, but that the correlation coefficients of these funds with the current portfolio are as shown in the answer choices below. Which of the funds should you recommend?

Fund D: correlation coefficient = -0.08 According to modern portfolio theory, total portfolio risk, as measured by standard deviation, is lowered by combining securities in a portfolio so that individual securities have negative (or low positive) correlations between each other's rates of return.

Which of the following are advantages of investing internationally? Some international markets are less efficient than U.S. markets. International mutual funds do not have the exchange rate risks of individual foreign stocks. Due to lower correlations with U.S. stocks, foreign stocks can lower total portfolio risk. Investors in foreign securities avoid U.S. tax on realized capital gains.

I and III Foreign markets have far fewer analysts following their stocks than do U.S. markets, and the information available from many foreign companies is less readily available to investors, making these markets less efficient than U.S. markets. Portfolio standard deviation is highly dependent on the covariance of returns. The covariance of foreign stocks with U.S. stocks is relatively low, serving to lower the standard deviation of the portfolio into which such stocks are placed. Exchange rate risk is a constant threat to a foreign stock or bond investment in any form, including mutual funds. Foreign security income is taxed first in the country of origin and then again in the United States. U.S. taxpayers can take a foreign tax credit on their U.S. return for the taxes paid to other countries. LO 5.5.1

Which of the following statements concerning a knowledge of the risk/return relationship is CORRECT? Future risk/return relationships are not guaranteed to match past risk/return relationships. Chances are that past relative relationships will not continue into the future. A reduction in risk also means a reduction in the possible return on the investment. The smaller the dispersion of returns, the greater the risk associated with a particular investment.

I and III Chances are that past relative relationships will continue into the future. The smaller the dispersion of returns, the lower the risk associated with a particular investment.

Which of the following are characteristics of collectibles investments that distinguish them from financial investments? The market is relatively inefficient. The capital gain is taxed at a maximum rate of 20%. There are large spreads between bid and ask prices. There is little liquidity risk.

I and III Collectibles are taxed at a maximum capital gain rate of 28%, compared to the 20% maximum rate on financial assets. Liquidity is a major risk in collectibles, since there is no active market for the assets. LO 5.4.1

Assume that the economic forecast for the coming year is expected to be one of increasing inflation and interest rates. The GDP is expected to be strong. Which of the following types of investments would be advisable for the coming year and why? Liquid investments, such as money market funds and short-term securities, to allow the investor flexibility to reinvest as rates increase Long-term debt, such as 20-year government bonds, to lock in current interest rates Stock in public utilities and durable goods firms, becausethey benefit from a rising interest rate environment Tangible assets, such as gold, to keep pace with the rate of inflation

I and IV Long-term bonds decrease in value in a rising interest rate environment. Stock in public utilities and durable goods firms does not benefit from a rising interest rate environment.

Mutual fund I has a standard deviation of 4% and an expected return of 10%. Mutual fund J has a standard deviation of 8% and an expected return of 13%. If I and J have a correlation coefficient of -1.0, which of the following statements is CORRECT?

I and J are perfectly negatively correlated. J's coefficient of variation is 8% ÷ 13% = 0.615. I's coefficient of variation = 4% ÷ 10% = 0.40. I is less risky, on a risk-adjusted basis, than J. Because I and J are perfectly negatively correlated (correlation coefficient of -1.0), there exists a combination of I and J such that the standard deviation is zero. The expected return of a portfolio is the weighted average, which cannot be less than the lowest expected return of the portfolio components.

You are about to recommend international mutual funds to your clients. Which of the following are characteristics of investing internationally? International markets are less efficient than U.S. markets. International mutual funds have the exchange rate risks of individual foreign stocks. Due to lower correlations with U.S. stocks, foreign stocks can lower total portfolio risk. Investors in foreign securities avoid U.S. tax on realized capital gains.

I, II, and III Explanation Foreign markets have fewer analysts following stocks than do U.S. markets, and the information available from many foreign companies is minimal, making these markets less efficient than U.S. markets. The covariance of foreign stocks, especially small-cap and emerging market stocks, with U.S. stocks is relatively low, serving to lower the standard deviation of the portfolio in which such stocks are placed. Exchange rate risk is a systematic risk with foreign investments. Foreign income is taxed first in the country of origin and then again in the United States; U.S. taxpayers can take a foreign tax credit for taxes paid to other countries. LO 5.5.1

Which of the following are characteristics of closed-end funds? A closed-end fund stands ready to redeem shares from investors. Closed-end funds trade like common stocks. Closed-end fund shares can be bought on margin or sold short. Closed-end funds will always trade at net asset value.

II and III Similar to common stocks, closed-end funds trade on the exchanges. Also, closed-end funds can be bought on margin and sold short.

Which of the following statements regarding the various performance measures are CORRECT? A positive alpha indicates that the manager consistently underperformed the market on a risk-adjusted basis. Jensen's alpha indicates how much the realized return differs from the expected return, as per the capital asset pricing model (CAPM). The Sharpe ratio is not useful for evaluating the performance of nondiversified portfolios. The Treynor ratio does not indicate whether a portfolio manager outperformed or underperformed the market portfolio.

II and IV The answer is II and IV. Statements I and III are incorrect. A positive alpha indicates that the manager outperformed the market on a risk-adjusted basis. The Sharpe ratio uses total risk, as measured by standard deviation, and is useful for evaluating the performance of both nondiversified and well-diversified portfolios.

Which of the following are factors used in industry analysis for investment purposes? Financial leverage Government rules and regulations Labor conditions Technological advances

II, III, and IV

Which of these are nondiversifiable risks? Business risk Interest rate risk Market risk Purchasing power risk

II, III, and IV

Which of the following are characteristics of exchange-traded funds (ETFs)? They may not be sold short. They are generally tax-efficient. Large investors known as authorized participants buy or sell shares on an "in-kind" basis. They usually trade at or near their net asset value.

II, III, and IV ETFs trade like stock and can be sold short. They are tax-efficient, generally low cost, and large investors conduct trades by making in-kind exchanges, whereby they give or receive shares of stock that are in the fund. Generally ETFs trade near net asset value (NAV), if not at NAV.

Which of the following statements regarding the variables in the Black/Scholes European call option pricing model is CORRECT? An increase in the price of the underlying stock decreases the value of the European call option. An increase in the variability of the price of the underlying stock decreases the value of the European call option. An increase in the risk-free rate increases the value of the European call option. An increase in the time to expiration decreases the value of the European call option.

III Only An increase in the price of the underlying stock and/or the variability in the price of the underlying stock increases the value of the European call option. An increase in the time to expiry also increases the value of the European call option.

JEM stock has a beta coefficient of 1.35 and the market has a rate of return of 8.50%. The 90-day U.S. Treasury bill rate of return is 1.75%. Based on the information provided, choose the CORRECT statements. The stock risk premium is 6.75%. The market risk premium is 9.11%. The expected rate of return is 10.86%.

III only The answer is III only. The expected rate of return based on the capital asset pricing model is 10.86%, calculated as follows: ri = 0.0175 + (0.085 - 0.0175)1.35 = 10.86, or 10.86% The stock risk premium is 9.11%, (0.085 - 0.0175)1.35. The market risk premium is 6.75%, (0.085 - 0.0175). LO 8.2.1

Identify the entity that issues guaranteed investment contracts (GICs).

Insurance companies

Immunization offsets which two risks in a bond portfolio?

Interest rate risk and reinvestment rate risk

Your client is concerned that the stock market is overvalued and may experience a large market correction within the next year. The client is 45 years old, has significant retirement savings, little debt, and no dependents. The current retirement portfolio mix is 80% stock/20% fixed-income. What is the best course of action for your client to take regarding this concern?

Maintain a long-term perspective and consider keeping the current portfolio allocation.

A client has $12,000 of capital gains and $15,000 of capital losses. How much unused loss is carried forward to the following tax year?

The answer is $0. After netting capital gain and losses, the client has a net capital loss of $3,000. Because $3,000 of net losses can be deducted during any one tax year, there is no carryforward.

Select the arrangement that is commonly used by dealers in government securities to satisfy short-term liquidity needs.

Repurchase agreement Dealers in government securities use repurchase agreements, or repos, to satisfy short-term liquidity needs.

Equity investments made for the launch, early development, or expansion of a business are known as

The answer is venture capital. Equity financing associated with the early development of a business is called venture capital. Mezzanine financing is provided for expansion and new products. Leveraged buyout financing is provided to allow management to buy all or part of a business; often used when a public company divests a division that it feels is no longer part of its long-term plans. Distressed debt is distressed debt investing in the debt of companies that are in trouble or failing.

An investor might use a stock index option instead of an individual stock option if the investor

The answer is wants to avoid business risk. Buying an index option means the risk associated with any one company (business risk) is avoided. An index option can be used to protect against market or sector swings, and the investor need not be concerned with individual companies.

Which of the following forms of the efficient market hypothesis suggests that fundamental analysis and insider information may produce above-market returns?

The answer is weak. The weak form holds that current stock prices reflect all historical market data and that historical price trends are, therefore, of no value in predicting future prices. However, this form holds that credible fundamental analysis and insider information may produce above-market returns.

CAL stock has a current annual dividend of $1.25 that has been growing at a constant rate of 4.5% per year. Assuming the stock is currently selling for $40, and your required rate of return is 7.5%, should you buy the stock at today's price?

The answer is yes, because the stock is undervalued. On the basis of the constant growth dividend discount model, the intrinsic value of CAL stock is $43.54, calculated as follows: V = [$1.25 × (1 + 0.045)] ÷ (0.075 - 0.045) V = $1.30625 ÷ 0.03 V = $43.54 Because CAL stock is currently selling for $40 per share, it is undervalued in the secondary market and worthy of a purchase.

Shannon is evaluating the absolute performance of the Shining Star mutual fund. The return of the fund for the past year was 13%, beta is 1.10, and standard deviation is 23. The market return is 9.5%, and the risk-free rate is 4.5%. Which of the following statements is true?

The fund's alpha is +3, meaning that the fund manager achieved a higher return than required for the risk taken. The answer is the fund's alpha is +3, meaning that the fund manager achieved a higher return than required for the amount of risk taken. Alpha is an absolute measure that is simply the difference between the return of the portfolio and the required return (CAPM). The formula is rp - [rf + (rm - rf)β. 13 - [4.5 + (9.5 - 4.5)1.1] = 3. Treynor and Sharpe are comparative or relative measures, and you would choose the investment with the highest number. Alpha is an absolute measure, giving you the actual return above the required return.

If the intrinsic value of a call option is $3, which of these statements is CORRECT?

The option is in-the-money.

Louis owns an investment that is an unmanaged portfolio in which the money manager initially selects the securities to be included in the portfolio and then holds those securities until they mature or the investment portfolio terminates. This statement best describes which type of investment?

Unit investment trust The answer is unit investment trust. A unit investment trust (UIT) is an investment company whose units are sold in the secondary market and is generally unmanaged, or passively managed as the money manager initially selects the securities to be included in the portfolio and then holds those securities until they mature or the UIT terminates.

You have narrowed your choice down to these investments with the following characteristics: JJJ LLL NNN YYY Mean return 10 18 7 11 Standard deviation 17 25 10 19 Which fund has the least risk per unit of return?

Using the coefficient of variation (CV). JJJ Fund: 17 ÷ 10 = 1.70 LLL Fund: 25 ÷ 18 = 1.39 NNN Fund 10 ÷ 7 = 1.43 YYY Fund 19 ÷ 11 = 1.73 The stock with the lowest CV has the least amount of total risk per unit of expected return. LO 6.2.3

Richard has become very interested in the stock market and enjoys spending his spare time researching companies in the medical field. He believes studying and analyzing the industry, combined with his advanced exposure to trends and new innovations in medicine, will give him an advantage in achieving superior performance in medical stock investment opportunities. Choose the form of the efficient market hypothesis (EMH), if any, that Richard is considered subscribing to.

Weak form Richard is subscribing to the weak form of the EMH because he believes fundamental analysis and insider information will yield superior performance.

In analyzing the position of a portfolio in terms of risk/return on the capital market line (CML), superior performance exists if the fund's position is ________ the CML, inferior performance exists if the fund's position is ________ the CML, and equilibrium position exists if it is ________ the CML.

above; below; on According to modern portfolio theory, the CML defines performance of a portfolio. All portfolios should plot on the CML in proportion to the risk of the portfolio. The Y axis is return and the X axis is the risk level. If a fund has superior performance, its return will be above the return level for the given risk level; if inferior, its return will be below the return level for the given risk level; and if it is in equilibrium, then it will plot exactly on the CML. LO 8.2.1

One implication of the efficient market hypothesis is that

although security markets are efficient, they are not necessarily equally efficient. Markets are efficient when many analysts follow a company, when information about a company is rapidly disseminated, and when no individual investor or analyst has information not readily available to any other investor or analyst. Some markets (e.g., foreign markets) may be less efficient, as may the markets for companies where few analysts follow a company, such as a small company that has few shares available for purchase by large institutional investors.

All of these correctly express risks associated with an investment in undeveloped land except

an investment in undeveloped land always guarantees a short-term profit. The answer is that an investment in undeveloped land always guarantees a short-term profit. An investment in undeveloped land is made with the expectation that the land will eventually provide a significant, future capital return over the long run.

The anticipation of inflation suggests that the investor should

anticipate higher interest rates. Real assets, which includes gold and real estate, should do well in inflationary times. Bonds do poorly because interest rates will increase to fight inflation, and increases in interest rates cause bond prices to fall.

Assets with expected returns that lie above the security market line (SML)

are undervalued. The answer is are undervalued. Assets that lie above the security market line (SML) are undervalued because their expected returns are higher than the required return represented by the SML.

What is the internal rate of return (IRR) on an investment that was purchased for $10,000, generated income at the end of Year 1 of $600, required an additional expenditure at the end of Year 2 of $300, and was sold at the end of Year 3 for $13,000?

he answer is 10.24%. Using the HP 10bII+: CF0 = (10,000) CF1 = 600 CF2 = (300) CF3 = 13,000 Solve for IRR/YR = 10.2432, or 10.24%.

call option with an exercise price of $105 is selling in the open market for $4.25 when the market price of the underlying stock is $102. What is the intrinsic value of this option?

he answer is $0. This call option is out-of-the-money; thus, its intrinsic value is zero.

Assume that Zephyr stock pays a dividend in the current year of $1.75 per share and that the dividend is expected to grow by 2% per year. Calculate the price of the stock assuming an investor has a required rate of return of 8%.

he answer is $29.75. The formula for the constant growth dividend discount model: V = D1 ÷ (r - g) Therefore the intrinsic value of ADM stock equals $29.75 [(1.75 × 1.02) ÷ (0.08 - 0.02)].

Jose owns a 30-year corporate bond, with 22 years remaining until maturity, featuring a coupon rate of 6.25% (paid semiannually). Assuming the comparable yield for this quality bond is currently 7%, calculate the intrinsic value of his bond.

he answer is $916.44. Jose's bond has an intrinsic value of $916.44, calculated as follows: END Mode, 2 P/YR FV = 1,000 PMT = 31.25 (6.25% × 1,000 ÷ 2) 22, DOWNSHIFT, N = 44 I/YR =7 Solve for PV = -916.4395, or $916.44 LO 4.1.1

Chris owns a bond that is convertible into common stock at $38 per share and has a coupon of 6.0%. Interest is paid semiannually. The current market price of the stock is $42 per share. The investment value of the bond is $1,050, and the bond currently sells for a market price of $1,225. Which one of these percentages is closest to the downside risk of this bond?

he answer is 14.29%. The downside risk of the bond is $175 ($1,225 current market price - $1,050 investment value). This translates into 14.29% ($175 downside risk ÷ $1,225 current market value). LO 2.2.1

Your client has established a balanced portfolio with various amounts allocated to different asset classes, and periodically she rebalances the portfolio to keep the same approximate percentages in these asset classes. Her approach is

he answer is strategic asset allocation. Strategic asset allocation involves re-balancing back to the original allocation and adjusting the allocation based on changing client circumstances. Tactical asset allocation involves choosing various sectors that you believe will do best, and changing as you believe is necessary. Dynamic asset allocation changes the allocation amounts as the market changes, typically used by institutional investors. Core-satellite asset allocation is a combination of strategic and tactical.

Wendy traveled to France and converted U.S. dollars into euros when the exchange rate was USD 1.42 for each euro. When Wendy returned from France, she had some euros left over and converted them back into U.S. dollars. At that time, one dollar was worth .77 euros. Wendy wants to know if she made or lost money on the euros she exchanged back into U.S. dollars. You inform her that

he answer is the dollar has strengthened, and she lost money. When she returned, she would have preferred the euro to strengthen against the dollar; that way, she could have received more dollars.

All of these statements correctly explain warrants except

issuing a bond with an attached warrant may permit the corporation to increase the coupon rate to entice investors to make the investment. The answer is issuing a bond with an attached warrant may permit the corporation to increase the coupon rate to entice investors to make the investment. Warrants give the bond purchaser a sweetener, which makes the issue more attractive. Issuing a bond with a warrant will typically allow the corporation to lower the coupon rate necessary to entice the investor to make the investment.

A convertible bond's market value will NOT fall below

its investment value.

A distribution that is more peaked than normal is

leptokurtic. Explanation The answer is leptokurtic. A distribution that is more peaked than normal is leptokurtic. A distribution that is flatter than normal is platykurtic.

Which of these statements about closed-end funds is CORRECT?

nvestors in closed-end shares can profit from changes in the discount to NAV. The answer is investors in closed-end shares can profit from changes in the discount to NAV. When the discount shrinks, an investor can profit even if the NAV of the underlying portfolio does not change.

If information is generated randomly and information announcements are independent

prices will change quickly in response to the new random information. The answer is prices will change quickly in response to the new random information. If information is generated randomly and information announcements are independent, prices will tend to fully reflect all available information. In addition, price changes will be independent of one another and tend to occur in random patterns.

If information is generated randomly and information announcements are independent,

prices will change very quickly in response to the new random information. If information is generated randomly and information announcements are independent, prices will tend to fully reflect all available information. In addition, price changes will be independent of one another and tend to occur random patterns.

The Dow Jones Utility Average has recently dropped 30% from its high, and you decide to recommend a utility sector fund to your clients. If they invest in the fund, your clients will be exposed to which of these risks? Interest rate risk Business risk Default risk Financial risk

sector funds are subject to the unsystematic (diversifiable) risks of business risk and financial risk; utility sector funds are also subject to the nondiversifiable interest rate risk because of their high debt to total capital percentage. Stocks are not subject to default risk. LO 6.1.2

A wheat farmer would hedge by _____________ wheat futures; a bread manufacturer would hedge by ________________ wheat futures; a speculator would _________ a wheat futures contract if he believes the price of wheat may rise; and a speculator would __________ a wheat futures contract if he believes the price of wheat may fall.

shorting; buying; buy; short Explanation The answer is shorting; buying; buy; short. A wheat farmer would hedge by shorting wheat futures; a bread manufacturer would hedge by buying wheat futures; a speculator would buy a wheat futures contract if he believes the price of wheat may rise; and a speculator would short a wheat futures contract if he believes the price of wheat may fall. LO 5.3.1

Unsystematic (unique) risk can be reduced by buying

stocks in numerous unrelated companies. The answer is stocks in numerous unrelated companies. Owning stock in unrelated companies results in holding stocks that have a low correlation coefficient between them. If a portfolio has numerous diversified issues of stocks, an investor can reduce and virtually eliminate the degree of unsystematic (unique) risk in the portfolio. Buying stocks in international companies can help to reduce systematic risk, because those stocks trade in different markets.

All of the following should be found in an investment policy statement (IPS) except

the actual investments. Explanation The answer is the actual investments. Risk tolerance, time frame, and liquidity needs should all be found in the IPS; investments are not found in the IPS. The IPS is a roadmap and provides guidance for the adviser and client as to the type of investments that will be used (such as 15%-20% in large-cap U.S. stocks), but the actual investments are not specified in the IPS.

Equity income funds may hold which of these types of securities? Income-producing common stocks Convertible bonds Convertible preferred stocks

the answer is I, II, and III. All three types of securities may be in an equity income fund.

Which of the following types of federal income tax treatment generally apply to municipal bonds? Ordinary income Tax-free income Capital gains or losses Tax-deferred income

the answer is II and III. Municipal bond interest is received federal income tax free by the bondholder. In addition, when a municipal bond is sold by the investor, the net proceeds above/below basis may be subject to capital gain/loss.

Choose the CORRECT statements concerning the futures market. A futures contract is a standardized, transferable agreement providing for the deferred delivery of either a designated commodity or financial instrument (or its cash value). Although a buy or sell commitment in futures trading is binding, a buyer or seller can eliminate the commitment by taking an opposite position in the same commodity or financial instrument for the same futures month.

the answer is both I and II. Both statements I and II are correct.

According to the unbiased expectations theory of interest rates,

the current long-term rate is the average of today's short-term rate and expected future short-term rates.

Open interest is

the number of futures contracts outstanding for a commodity on any given trading day.


संबंधित स्टडी सेट्स

Block 10 Module 9 Iggy Questions: ED/Trauma/Burns

View Set

Micro Chapter 13: Viruses, Viroids, and Prions

View Set

Business Computer Applications: 1305

View Set

Chapter 11 Pain Management Prep U 40 questions

View Set

Chapter 15: Respiratory Emergencies

View Set